Você está na página 1de 174

Apostila Digital

Matemática
A
Análise Combinatória
Introdução à Análise Combinatória
Análise Combinatória é um conjunto de procedimentos que possibilita a construção de grupos diferentes
formados por um número finito de elementos de um conjunto sob certas circunstâncias.
Na maior parte das vezes, tomaremos conjuntos Z com m elementos e os grupos formados com elementos
de Z terão p elementos, isto é, p será a taxa do agrupamento, com p<m.
Arranjos, Permutações ou Combinações, são os três tipos principais de agrupamentos, sendo que eles
podem ser simples, com repetição ou circulares. Apresentaremos alguns detalhes de tais agrupamentos.
Observação: É comum encontrarmos na literatura termos como: arranjar, combinar ou permutar, mas todo o
cuidado é pouco com os mesmos, que às vezes são utilizados em concursos em uma forma dúbia!

Arranjos
São agrupamentos formados com p elementos, (p<m) de forma que os p elementos sejam distintos entre sí
pela ordem ou pela espécie. Os arranjos podem ser simples ou com repetição.
Arranjo simples: Não ocorre a repetição de qualquer elemento em cada grupo de p elementos.
Fórmula: As(m,p) = m!/(m-p)!
Cálculo para o exemplo: As(4,2) = 4!/2!=24/2=12.
Exemplo: Seja Z={A,B,C,D}, m=4 e p=2. Os arranjos simples desses 4 elementos tomados 2 a 2 são 12
grupos que não podem ter a repetição de qualquer elemento mas que podem aparecer na ordem trocada.
Todos os agrupamentos estão no conjunto:
As={AB,AC,AD,BA,BC,BD,CA,CB,CD,DA,DB,DC}
Arranjo com repetição: Todos os elementos podem aparecer repetidos em cada grupo de p elementos.
Fórmula: Ar(m,p) = mp.
Cálculo para o exemplo: Ar(4,2) = 42=16.
Exemplo: Seja C={A,B,C,D}, m=4 e p=2. Os arranjos com repetição desses 4 elementos tomados 2 a 2 são
16 grupos que onde aparecem elementos repetidos em cada grupo. Todos os agrupamentos estão no
conjunto:
Ar={AA,AB,AC,AD,BA,BB,BC,BD,CA,CB,CC,CD,DA,DB,DC,DD}
Arranjo condicional: Todos os elementos aparecem em cada grupo de p elementos, mas existe uma
condição que deve ser satisfeita acerca de alguns elementos.
Fórmula: N=A(m1,p1).A(m-m1,p-p1)
Cálculo para o exemplo: N=A(3,2).A(7-3,4-2)=A(3,2).A(4,2)=6×12=72.
Exemplo: Quantos arranjos com 4 elementos do conjunto {A,B,C,D,E,F,G}, começam com duas letras
escolhidas no subconjunto {A,B,C}?
Aqui temos um total de m=7 letras, a taxa é p=4, o subconjunto escolhido tem m1=3 elementos e a taxa que
este subconjunto será formado é p1=2. Com as letras A,B e C, tomadas 2 a 2, temos 6 grupos que estão no
conjunto:
PABC = {AB,BA,AC,CA,BC,CB}
Com as letras D,E,F e G tomadas 2 a 2, temos 12 grupos que estão no conjunto:
PDEFG = {DE,DF,DG,ED,EF,EG,FD,FE,FG,GD,GE,GF}
Usando a regra do produto, teremos 72 possibilidades obtidas pela junção de um elemento do conjunto
PABC com um elemento do conjunto PDEFG. Um típico arranjo para esta situação é CAFG.

Permutações
Quando formamos agrupamentos com m elementos, de forma que os m elementos sejam distintos entre sí
pela ordem. As permutações podem ser simples, com repetição ou circulares.
Permutação simples: São agrupamentos com todos os m elementos distintos.
Fórmula: Ps(m) = m!.
Cálculo para o exemplo: Ps(3) = 3!=6.
Exemplo: Seja C={A,B,C} e m=3. As permutações simples desses 3 elementos são 6 agrupamentos que
não podem ter a repetição de qualquer elemento em cada grupo mas podem aparecer na ordem trocada.
Todos os agrupamentos estão no conjunto:
Ps={ABC,ACB,BAC,BCA,CAB,CBA}
Permutação com repetição: Dentre os m elementos do conjunto C={x1,x2,x3,...,xn}, faremos a suposição
que existem m1 iguais a x1, m2 iguais a x2, m3 iguais a x3, ... , mn iguais a xn, de modo que
m1+m2+m3+...+mn=m.
Fórmula: Se m=m1+m2+m3+...+mn, então
Pr(m)=C(m,m1).C(m-m1,m2).C(m-m1-m2,m3) ... C(mn,mn)
Anagrama: Um anagrama é uma (outra) palavra construída com as mesmas letras da palavra original
trocadas de posição.
Cálculo para o exemplo: m1=4, m2=2, m3=1, m4=1 e m=6, logo: Pr(6)=C(6,4).C(6-4,2).C(6-4-
1,1)=C(6,4).C(2,2).C(1,1)=15.
Exemplo: Quantos anagramas podemos formar com as 6 letras da palavra ARARAT. A letra A ocorre 3
vezes, a letra R ocorre 2 vezes e a letra T ocorre 1 vez. As permutações com repetição desses 3 elementos
do conjunto C={A,R,T} em agrupamentos de 6 elementos são 15 grupos que contêm a repetição de todos
os elementos de C aparecendo também na ordem trocada. Todos os agrupamentos estão no conjunto:
Pr={AAARRT,AAATRR,AAARTR,AARRTA,AARTTA,
AATRRA,AARRTA,ARAART,ARARAT,ARARTA,
ARAATR,ARAART,ARAATR,ATAARA,ATARAR}
Permutação circular: Situação que ocorre quando temos grupos com m elementos distintos formando uma
circunferência de círculo.
Fórmula: Pc(m)=(m-1)!
Cálculo para o exemplo: P(4)=3!=6
Exemplo: Seja um conjunto com 4 pessoas K={A,B,C,D}. De quantos modos distintos estas pessoas
poderão sentar-se junto a uma mesa circular (pode ser retangular) para realizar o jantar sem que haja
repetição das posições?
Se considerássemos todas as permutações simples possíveis com estas 4 pessoas, teriamos 24 grupos,
apresentados no conjunto:
Pc={ABCD,ABDC,ACBD,ACDB,ADBC,ADCB,BACD,BADC,
BCAD,BCDA,BDAC,BDCA,CABD,CADB,CBAD,CBDA,
CDAB,CDBA, DABC,DACB,DBAC,DBCA,DCAB,DCBA}
Acontece que junto a uma mesa "circular" temos que:
ABCD=BCDA=CDAB=DABC
ABDC=BDCA=DCAB=CABD
ACBD=CBDA=BDAC=DACB
ACDB=CDBA=DBAC=BACD
ADBC=DBCA=BCAD=CADB
ADCB=DCBA=CBAD=BADC
Existem somente 6 grupos distintos, dados por:
Pc={ABCD,ABDC,ACBD,ACDB,ADBC,ADCB}

Combinações
Quando formamos agrupamentos com p elementos, (p<m) de forma que os p elementos sejam distintos
entre sí apenas pela espécie.
Combinação simples: Não ocorre a repetição de qualquer elemento em cada grupo de p elementos.
Fórmula: C(m,p) = m!/[(m-p)! p!]
Cálculo para o exemplo: C(4,2)=4!/[2!2!]=24/4=6
Exemplo: Seja C={A,B,C,D}, m=4 e p=2. As combinações simples desses 4 elementos tomados 2 a 2 são 6
grupos que não podem ter a repetição de qualquer elemento nem podem aparecer na ordem trocada. Todos
os agrupamentos estão no conjunto:
Cs={AB,AC,AD,BC,BD,CD}
Combinação com repetição: Todos os elementos podem aparecer repetidos em cada grupo até p vezes.
Fórmula: Cr(m,p)=C(m+p-1,p)
Cálculo para o exemplo: Cr(4,2)=C(4+2-1,2)=C(5,2)=5!/[2!3!]=10
Exemplo: Seja C={A,B,C,D}, m=4 e p=2. As combinações com repetição desses 4 elementos tomados 2 a 2
são 10 grupos que têm todas as repetições possíveis de elementos em grupos de 2 elementos não podendo
aparecer o mesmo grupo com a ordem trocada. De um modo geral neste caso, todos os agrupamentos com
2 elementos formam um conjunto com 16 elementos:
Cr={AA,AB,AC,AD,BA,BB,BC,BD,CA,CB,CC,CD,DA,DB,DC,DD}
mas para obter as combinações com repetição, deveremos excluir deste conjunto os 6 grupos que já
apareceram antes, pois AB=BA, AC=CA, AD=DA, BC=CB, BD=DB e CD=DC, assim as combinações com
repetição dos elementos de C tomados 2 a 2, são:
Cr={AA,AB,AC,AD,BB,BC,BD,CC,CD,DD}

Regras gerais sobre a Análise Combinatória


Problemas de Análise Combinatória normalmente são muito difíceis mas eles podem ser resolvidos através
de duas regras básicas: a regra da soma e a regra do produto.
Regra da soma: A regra da soma nos diz que se um elemento pode ser escolhido de m formas e um outro
elemento pode ser escolhido de n formas, então a escolha de um ou outro elemento se realizará de m+n
formas, desde que tais escolhas sejam independentes, isto é, nenhuma das escolhas de um elemento pode
coincidir com uma escolha do outro.
Regra do Produto: A regra do produto diz que se um elemento H pode ser escolhido de m formas diferentes
e se depois de cada uma dessas escolhas, um outro elemento M pode ser escolhido de n formas diferentes,
a escolha do par (H,M) nesta ordem poderá ser realizada de m.n formas.
Exemplo: Consideremos duas retas paralelas ou concorrentes sem que os pontos sob análise estejam em
ambas, sendo que a primeira r contem m pontos distintos marcados por r1, r2, r3, ..., rm e a
segunda s contem n outros pontos distintos marcados por s1, s2, s3, ..., sn. De quantas maneiras podemos
traçar segmentos de retas com uma extremidade numa reta e a outra extremidade na outra reta?

É fácil ver isto ligando r1 a todos os pontos de s e assim teremos n segmentos, depois ligando r 2 a todos os
pontos de s e assim teremos n segmentos, e continuamos até o último ponto para obter também n
segmentos. Como existem m pontos em r e n pontos em s, teremos m.n segmentos possíveis.

Número de Arranjos simples


Seja C um conjunto com m elementos. De quantas maneiras diferentes poderemos escolher p elementos
(p<m) deste conjunto? Cada uma dessas escolhas será chamada um arranjo de m elementos tomados p a
p. Construiremos uma sequência com os m elementos de C.
c1, c2, c3, c4, c5, ..., cm-2, cm-1, cm
Cada vez que um elemento for retirado, indicaremos esta operação com a mudança da cor do elemento
para a cor vermelha.
Para escolher o primeiro elemento do conjunto C que possui m elementos, temos m possibilidades. Vamos
supor que a escolha tenha caído sobre o m-ésimo elemento de C.
c1, c2, c3, c4, c5, ..., cm-2, cm-1, cm
Para escolher o segundo elemento, devemos observar o que sobrou no conjunto e constatamos que agora
existem apenas m-1 elementos. Suponhamos que tenha sido retirado o último elemento dentre os que
sobraram no conjunto C. O elemento retirado na segunda fase é o (m-1)-ésimo.
c1, c2, c3, c4, c5, ..., cm-2, cm-1, cm
Após a segunda retirada, sobraram m-2 possibilidades para a próxima retirada. Do que sobrou, se
retirarmos o terceiro elemento como sendo o de ordem (m-2), teremos algo que pode ser visualizado como:
c1, c2, c3, c4, c5, ..., cm-2, cm-1, cm
Se continuarmos o processo de retirada, cada vez teremos 1 elemento a menos do que na fase anterior.
Para retirar o p-ésimo elemento, restarão m-p+1 possibilidades de escolha.
Para saber o número total de arranjos possíveis de m elementos tomados p a p, basta multiplicar os
números que aparecem na segunda coluna da tabela abaixo:

Retirada Número de possibilidades


1 m
2 m-1
3 m-2
... ...
p m-p+1
No.de arranjos m(m-1)(m-2)...(m-p+1)
Denotaremos o número de arranjos de m elementos tomados p a p, por A(m,p) e a expressão para seu
cálculo será dada por:
A(m,p) = m(m-1)(m-2)...(m-p+1)
Exemplo: Consideremos as 5 vogais de nosso alfabeto. Quais e quantas são as possibilidades de dispor
estas 5 vogais em grupos de 2 elementos diferentes? O conjunto solução é:
{AE,AI,AO,AU,EA,EI,EO,EU,IA,IE,
IO,IU,OA,OE,OI,OU,UA,UE,UI,UO}
A solução numérica é A(5,2)=5×4=20.
Exemplo: Consideremos as 5 vogais de nosso alfabeto. Quais e quantas são as possibilidades de dispor
estas 5 vogais em grupos de 2 elementos (não necessariamente diferentes)?
Sugestão: Construir uma reta com as 5 vogais e outra reta paralela à anterior com as 5 vogais, usar a regra
do produto para concluir que há 5x5=25 possibilidades.
O conjunto solução é:
{AA,AE,AI,AO,AU,EA,EE,EI,EO,EU,IA,IE,II,
IO,IU,OA,OE,OI,OO,OU,UA,UE,UI,UO,UU}
Exemplo: Quantas placas de carros podem existir no atual sistema brasileiro de trânsito que permite 3 letras
iniciais e 4 algarismos no final?
XYZ-1234
Sugestão: Considere que existem 26 letras em nosso alfabeto que podem ser dispostas 3 a 3 e 10
algarismos que podem ser dispostos 4 a 4 e em seguida utilize a regra do produto.
Número de Permutações simples
Este é um caso particular de arranjo em que p=m. Para obter o número de permutações com m elementos
distintos de um conjunto C, basta escolher os m elementos em uma determinada ordem. A tabela de
arranjos com todas as linhas até a ordem p=m, permitirá obter o número de permutações de m elementos:
Retirada Número de possibilidades
1 m
2 m-1
... ...
p m-p+1
... ...
m-2 3
m-1 2
m 1
No.de permutações m(m-1)(m-2)...(m-p+1)...4.3.2.1
Denotaremos o número de permutações de m elementos, por P(m) e a expressão para seu cálculo será
dada por:
P(m) = m(m-1)(m-2) ... (m-p+1) ... 3 . 2 . 1
Em função da forma como construímos o processo, podemos escrever:
A(m,m) = P(m)
Como o uso de permutações é muito intenso em Matemática e nas ciências em geral, costuma-se
simplificar a permutação de m elementos e escrever simplesmente:
P(m) = m!
Este símbolo de exclamação posto junto ao número m é lido como: fatorial de m, onde m é um número
natural.
Embora zero não seja um número natural no sentido que tenha tido origem nas coisas da natureza, procura-
se dar sentido para a definição de fatorial de m de uma forma mais ampla, incluindo m=0 e para isto
podemos escrever:
0!=1
Em contextos mais avançados, existe a função gama que generaliza o conceito de fatorial de um número
real, excluindo os inteiros negativos e com estas informações pode-se demonstrar que 0!=1.
O fatorial de um número inteiro não negativo pode ser definido de uma forma recursiva através da função
P=P(m) ou com o uso do sinal de exclamação:
(m+1)! = (m+1).m!, 0! = 1
Exemplo: De quantos modos podemos colocar juntos 3 livros A, B e C diferentes em uma estante? O
número de arranjos é P(3)=6 e o conjunto solução é:
P={ABC,ACB,BAC,BCA,CAB,CBA}
Exemplo: Quantos anagramas são possíveis com as letras da palavra AMOR? O número de arranjos é
P(4)=24 e o conjunto solução é:
P={AMOR,AMRO,AROM,ARMO,AORM,AOMR,MARO,MAOR,
MROA,MRAO,MORA,MOAR,OAMR,OARM,ORMA,ORAM,
OMAR,OMRA,RAMO,RAOM,RMOA,RMAO,ROAM,ROMA}

Número de Combinações simples


Seja C um conjunto com m elementos distintos. No estudo de arranjos, já vimos antes que é possível
escolher p elementos de A, mas quando realizamos tais escolhas pode acontecer que duas coleções com p
elementos tenham os mesmos elementos em ordens trocadas. Uma situação típica é a escolha de um casal
(H,M). Quando se fala casal, não tem importância a ordem da posição (H,M) ou (M,H), assim não há a
necessidade de escolher duas vezes as mesmas pessoas para formar o referido casal. Para evitar a
repetição de elementos em grupos com a mesma quantidade p de elementos, introduziremos o conceito de
combinação.
Diremos que uma coleção de p elementos de um conjunto C com m elementos é uma combinação de m
elementos tomados p a p, se as coleções com p elementos não tem os mesmos elementos que já
apareceram em outras coleções com o mesmo número p de elementos.
Aqui temos outra situação particular de arranjo, mas não pode acontecer a repetição do mesmo grupo de
elementos em uma ordem diferente.
Isto significa que dentre todos os A(m,p) arranjos com p elementos, existem p! desses arranjos com
os mesmos elementos, assim, para obter a combinação de m elementos tomados p a p, deveremos dividir
o número A(m,p) por m! para obter apenas o número de arranjos que contem conjuntos distintos, ou seja:
C(m,p) = A(m,p) / p!
Como
A(m,p) = m.(m-1).(m-2)...(m-p+1)
então:
C(m,p) = [ m.(m-1).(m-2). ... .(m-p+1)] / p!
que pode ser reescrito
C(m,p)=[m.(m-1).(m-2)...(m-p+1)]/[(1.2.3.4....(p-1)p]
Multiplicando o numerador e o denominador desta fração por
(m-p)(m-p-1)(m-p-2)...3.2.1
que é o mesmo que multiplicar por (m-p)!, o numerador da fração ficará:
m.(m-1).(m-2).....(m-p+1)(m-p)(m-p-1)...3.2.1 = m!
e o denominador ficará:
p! (m-p)!
Assim, a expressão simplificada para a combinação de m elementos tomados p a p, será uma das
seguintes:

Número de arranjos com repetição


Seja C um conjunto com m elementos distintos e considere p elementos escolhidos neste conjunto em uma
ordem determinada. Cada uma de tais escolhas é denominada um arranjo com repetição de m elementos
tomados p a p. Acontece que existem m possibilidades para a colocação de cada elemento, logo, o número
total de arranjos com repetição de m elementos escolhidos p a p é dado por mp. Indicamos isto por:
Arep(m,p) = mp

Número de permutações com repetição


Consideremos 3 bolas vermelhas, 2 bolas azuis e 5 bolas amarelas. Coloque estas bolas em uma ordem
determinada. Iremos obter o número de permutações com repetição dessas bolas. Tomemos 10
compartimentos numerados onde serão colocadas as bolas. Primeiro coloque as 3 bolas vermelhas em 3
compartimentos, o que dá C(10,3) possibilidades. Agora coloque as 2 bolas azuis nos compartimentos
restantes para obter C(10-3,2) possibilidades e finalmente coloque as 5 bolas amarelas. As possibilidades
são C(10-3-2,5).
O número total de possibilidades pode ser calculado como:

Tal metodologia pode ser generalizada.


Número de combinações com repetição
Considere m elementos distintos e ordenados. Escolha p elementos um após o outro e ordene estes
elementos na mesma ordem que os elementos dados. O resultado é chamado uma combinação com
repetição de m elementos tomados p a p. Denotamos o número destas combinações por Crep(m,p). Aqui a
taxa p poderá ser maior do que o número m de elementos.
Seja o conjunto A=(a,b,c,d,e) e p=6. As coleções (a,a,b,d,d,d), (b,b,b,c,d,e) e (c,c,c,c,c,c) são exemplos de
combinações com repetição de 5 elementos escolhidos 6 a 6.
Podemos representar tais combinações por meio de símbolos # e vazios Ø onde cada ponto # é repetido (e
colocado junto) tantas vezes quantas vezes aparece uma escolha do mesmo tipo, enquanto o vazio Ø serve
para separar os objetos em função das suas diferenças
(a,a,b,d,d,d) equivale a ##Ø#ØØ###Ø
(b,b,b,c,d,e) equivale a Ø###Ø#Ø#Ø#
(c,c,c,c,c,c) equivale a ØØ######ØØ

Cada símbolo possui 10 lugares com exatamente 6# e 4Ø. Para cada combinação existe uma
correspondência biunívoca com um símbolo e reciprocamente. Podemos construir um símbolo pondo
exatamente 6 pontos em 10 lugares. Após isto, os espaços vazios são prenchidos com barras. Isto pode ser
feito de C(10,6) modos. Assim:
Crep(5,6) = C(5+6-1,6)
Generalizando isto, podemos mostrar que:
Crep(m,p) = C(m+p-1,p)

Propriedades das combinações


O segundo número, indicado logo acima por p é conhecido como a taxa que define a quantidade de
elementos de cada escolha.
Taxas complementares
C(m,p)=C(m,m-p)
Exemplo: C(12,10) = C(12,2)=66.

Relação do triângulo de Pascal


C(m,p)=C(m-1,p)+C(m-1,p-1)
Exemplo: C(12,10)=C(11,10)+C(11,9)=605

Número Binomial
O número de combinações de m elementos tomados p a p, indicado antes por C(m,p) é chamado
Coeficiente Binomial ou número binomial, denotado na literatura científica como:

Exemplo: C(8,2)=28.
Extensão: Existe uma importante extensão do conceito de número binomial ao conjunto dos números reais
e podemos calcular o número binomial de qualquer número real r que seja diferente de um número inteiro
negativo, tomado a uma taxa inteira p, somente que, neste caso, não podemos mais utilizar a notação de
combinação C(m,p) pois esta somente tem sentido quando m e p são números inteiros não negativos.
Como Pi=3,1415926535..., então:
A função envolvida com este contexto é a função gama. Tais cálculos são úteis em Probabilidade e
Estatística.

Teorema Binomial
Se m é um número natural, para simplificar um pouco as notações, escreveremos mp no lugar de C(m,p).
Então:
(a+b)m = am+m1am-1b+m2am-2b2+m3am-3b3+...+mmbm
Alguns casos particulares com m=2, 3, 4 e 5, são:
(a+b)2 = a2 + 2ab + b2
(a+b)3 = a3 + 3 a2b + 3 ab2 + b3
(a+b)4 = a4 + 4 a3b + 6 a2b2 + 4 ab3 + b4
(a+b)5 = a5 + 5 a4b + 10 a3b2 + 10 a2b3 + 5 ab4 + b5
A demonstração segue pelo Princípio da Indução Matemática.
Iremos considerar a Proposição P(m) de ordem m, dada por:
P(m): (a+b)m=am+m1am-1b+m2am-2b2+m3am-3b3+...+mmbm
P(1) é verdadeira pois (a+b)1 = a + b
Vamos considerar verdadeira a proposição P(k), com k>1:
P(k): (a+b)k=ak+k1ak-1b+k2ak-2b2+k3ak-3b3+...+kkbk
para provar a propriedade P(k+1).
Para que a proposição P(k+1) seja verdadeira, deveremos chegar à conclusão que:
(a+b)k+1=ak+1+(k+1)1akb+(k+1)2ak-1b2+...+(k+1)(k+1)bk+1
(a+b)k+1= (a+b).(a+b)k
= (a+b).[ak+k1ak-1b+k2ak-2b2+k3ak-3b3+...+kkbk]
a.[ak+k1ak-1b+k2ak-2 b2+k3ak-3b3+...+kkbk]
=
+b.[ak+k1ak-1b+k2ak-2b2+k3ak-3b3+...+kk bk]
ak+1+k1akb+k2ak-1b2+k3ak-2b3+...+kkabk
=
+akb+k1ak-1b2+k2ak-2 b3+k3ak-3b4+...+kkbk+1
ak+1+[k1+1]akb+[k2+k1]ak-1b2+[k3+k2]ak-2b3
=
+[k4+k3] ak-3b4+...+[kk-1+kk-2]a2bk-1+[kk+kk-1]abk+kkbk+1
ak+1+[k1+k0] akb+[k2+k1]ak-1b2+[k3+k2]ak-2b3
=
+[k4+k3]ak-3b4+...+[kk-1+kk-2]a2bk-1+[kk+kk-1]abk+kkbk+1
Pelas propriedades das combinações, temos:
k1+k0=C(k,1)+C(k,0)=C(k+1,1)=(k+1)1
k2+k1=C(k,2)+C(k,1)=C(k+1,2)=(k+1)2
k3+k2=C(k,3)+C(k,2)=C(k+1,3)=(k+1)3
k4+k3=C(k,4)+C(k,3)=C(k+1,4)=(k+1)4
... ... ... ...
kk-1+kk-2=C(k,k-1)+C(k,k-2)=C(k+1,k-1)=(k+1)k-1
kk+kk-1=C(k,k)+C(k,k-1)=C(k+1,k)=(k+1)k
E assim podemos escrever:
ak+1+(k+1)1akb + (k+1)2ak-1b2 + (k+1)3ak-2b3
(a+b)k+1=
+(k+1)4ak-3b4 +...+ (k+1)k-1a2bk-1 + (k+1)kabk + kkbk+1
que é o resultado desejado.
                             CONTEÚDOS  E  MÉTODOS  PARA  O  ENSINO  DA  MATEMÁTICA   1  
 

AS QUATRO “OPERAÇÕES FUNDAMENTAIS”

Cada uma das quatro operações tem mais de uma ideia ou mais de um uso na
resolução de problemas. Para quem já está acostumado a lidar com situações-problema
que envolvam essas operações, às vezes é difícil perceber as diferentes ideias implicadas
em cada operação. Entretanto, para o aluno, essas diferenças constituem muitas vezes
grandes obstáculos. Por isso, é muito importante, no trabalho com as quatro operações,
que o professor explore conscientemente suas diferentes ações. A compreensão do
significado das operações e seu uso na resolução de problemas são um dos objetivos mais
importantes do bloco de conteúdos Números e Operações, definido nos Parâmetros
Curriculares Nacionais (BRASIL. PCN, 1997) de Matemática.

1. Os conceitos de adição e subtração

A conceituação da operação de adição serve de base para boa parte de


aprendizagens futuras em Matemática. A criança deve passar por várias experiências
concretas envolvendo o conceito da adição para que ela possa interiorizá-lo e transferi-lo
para a aprendizagem do algoritmo, que vem a ser um mecanismo de cálculo. A
conceituação da operação de subtração deve ser feita paralelamente, já que em atividades
concretas a exploração dos dois tipos de conceitos é muito natural. Além disso, não
podemos deixar escapar a oportunidade que o aluno tem de ver, na prática, que a
subtração e a adição são operações inversas. Por exemplo, quando reúne objetos para
desenvolver o significado da adição, a criança sente que pode também separá-los. Assim,
ela vê que se 4 + 2 = 6, vale também que 6 – 2 = 4.
Quando desenvolve o conceito de número, a criança verifica, por exemplo, que
pode arrumar cinco palitos como “quatro e um” ou “três e dois”. Tais experiências devem
ser enriquecidas, para que a criança possa registrá-las mais tarde, em linguagem
matemática como: 4 + 1 = 5 e 3 + 2 = 5. A professora ou o professor terá de oferecer
inúmeras oportunidades concretas para que a criança comece a exprimir experiências em
linguagem matemática. Assim, quando ela escreve 4 + 3 = 7, esta ação deve refletir uma
experiência e não uma simples informação transmitida pela professora ou pelo professor.

1.1 Ações associadas às operações de adição e subtração

A adição corresponde sempre a dois tipos básicos de ação: juntar (ou reunir) ou
então acrescentar, enquanto a subtração corresponde às ações de: retirar, comparar ou
completar. É muito importante que as crianças vivenciem experiências envolvendo todos
estes tipos de ação. A dificuldade que os alunos sentem na resolução de problemas,
expressada muitas vezes pela pergunta “que conta devo fazer?”, é causada,
principalmente, pela falta de experiências concretas variadas.

Conteúdos  e  Métodos  para  o  Ensino  da  Matemática  


Pedagogia  –  FAC-­‐UNILAGOS  
 
                             CONTEÚDOS  E  MÉTODOS  PARA  O  ENSINO  DA  MATEMÁTICA   2  
 

Atividades que envolvem a ação de juntar

1) Utilize materiais concretos como chapinhas palitos, botões, grãos e pedrinhas e uma
folha de papel para cada aluno, na qual estão desenhados três círculos de cores diferentes
(azul, vermelho e verde, por exemplo). Peça às crianças que coloquem 3 lápis no círculo
vermelho e 2 no círculo azul. Feito isto, peça que juntem todos os lápis no círculo verde e
pergunte: “quantos lápis estão reunidos no círculo verde?”.

2) Explore atividades lúdicas, como por exemplo, o “jogo de esconder”. Neste jogo,
distribua um certo número de objetos do mesmo tipo para cada dupla de alunos (podem
ser 9 no primeiro momento, e mais tarde uma quantidade maior). Diga às crianças que o
jogo tem as seguintes regras:

a) Um aluno apresenta ao seu colega certa quantidade de fichas (ou do objeto que estiver
sendo utilizando) arrumadas em dois grupos – as fichas não utilizadas permanecem
escondidas da vista do outro jogador.
b) Depois que o colega observar, junta as fichas e cobre-as com uma folha de papel.
c) O outro aluno que joga deve dizer o total de fichas que ficou embaixo da folha.
d) Em seguida, os dois alunos levantam a folha e conferem o resultado. Para cada
resultado correto será marcado um ponto para o jogador.
e) A turma faz 10 jogadas, revezando sempre o aluno jogador. Depois os pontos são
contados para se determinar o vencedor da partida.

Atividade que envolve a ação de acrescentar

1) Uma forma interessante de se trabalhar é contar histórias, usando, por exemplo,


flanelogravuras. Por exemplo: “Havia 5 patinhos no lago”. Peça que um aluno venha à
frente e prenda cinco patinhos no flanelógrafo, de forma que as outras crianças
acompanhem a tarefa. Continue contando: “Chegaram mais dois patinhos”. Outro aluno
deve fazer a ação de acrescentar os novos patinhos ao flanelógrafo. Pergunte então, no
final: “quantos patinhos estão agora no lago?”.

Ações de acrescentar são também bastante comuns em situações que ocorrem no cotidiano
da sala de aula. A professora ou o professor atento pode registrar estas ocorrências e fazer
perguntas.

Atividades que envolvem a ação de retirar

1) Usando o mesmo tipo de material adotado em atividades anteriores, proponha que um


aluno “coloque 5 borrachas dentro da caixa”. Depois, peça que ele “retire 3” e que, ao
final, “verifique quantas ficaram na caixa”.

2) Forme, na frente da turma, uma fila de crianças (até 9). Peça a uma criança, que não
esteja na fila, que observe a quantidade de crianças na fila e depois vire de costas. Sem

Conteúdos  e  Métodos  para  o  Ensino  da  Matemática  


Pedagogia  –  FAC-­‐UNILAGOS  
 
                             CONTEÚDOS  E  MÉTODOS  PARA  O  ENSINO  DA  MATEMÁTICA   3  
 

falar, retire alguns alunos da fila e diga à criança de costas que se vire. Em seguida,
pergunte:

- “Quantos alunos havia na fila?”


- “Quantos alunos ainda ficaram?”
- “Quantos saíram?”

Repita a atividade com outros alunos, sempre mudando o número de alunos da fila.

Atividades que envolvem a ação de comparar

A ação de comparar não é do mesmo tipo que a ação de retirar. Considerando o


grupo original dado, na ação de retirar uma parte era subtraída para se encontrar o resto.
No entanto, numa ação comparativa como “Marcos tem 5 lápis e 2 canetas. Quantos lápis ele
tem a mais do que canetas?”, as duas canetas não podem ser retiradas do conjunto de 5 lápis.

A forma de criar situações para que a criança perceba que a operação de subtração é
a que deve ser associada à comparação é o emparelhamento de objetos. Colocando os
elementos dos dois conjuntos, lado a lado, até que todos os elementos de um dos
conjuntos tenham sido utilizados, a criança verá que a resposta (quantos a mais) é a
quantidade de elementos que ficaram sem par. A ação concreta necessária para encontrar
esta resposta é separar ou retirar os elementos do conjunto maior, que tiveram elementos
correspondentes no conjunto menor. Assim, ele estará determinando o número de
elementos do resto, e esta ação corresponde à determinação de quantos elementos a mais
existem.
Dessa forma, estaremos sempre subtraindo elementos de um mesmo conjunto. Do
total de 3 lápis (conjunto maior), retiramos 2 deles, que foram emparelhados com as 2
borrachas. Sobra 1 lápis. Este resultado diz “quantos a mais” há no conjunto maior.
Utilize materiais diferenciados e proporcione muitas atividades de emparelhar
objetos. Somente quando você perceber que a relação da ação de comparação com a
subtração foi compreendida e está sendo corretamente utilizada, é que você poderá partir
para generalizações, trabalhando com comparações nas quais os alunos não possam dispor
os elementos dos dois conjuntos lado a lado.

Atividades que envolvem a ação de completar

Para a criança, a utilização da subtração em situações de completar é ainda mais


difícil. Quando precisamos descobrir quantos elementos faltam para completar um

Conteúdos  e  Métodos  para  o  Ensino  da  Matemática  


Pedagogia  –  FAC-­‐UNILAGOS  
 
                             CONTEÚDOS  E  MÉTODOS  PARA  O  ENSINO  DA  MATEMÁTICA   4  
 

conjunto de objetos, a ação de completar está intimamente relacionada à ação de


acrescentar. No entanto, a operação realizada é a subtração, e as crianças devem ser
ajudadas a compreender POR QUE se usa a subtração para resolver esse tipo de situação,
à qual uma idéia aditiva está associada.
Aqui, para compreender que a subtração resolve esse tipo de situação-problema, o
aluno deve ser levado a visualizar a quantidade total necessária e a retirada do que já tem
deste total. Separando o conjunto de objetos disponíveis do total necessário, o aluno verá
porque subtrai para encontrar a resposta.
Coloque no flanelógrafo (ou sobre uma mesa, ou em um mural) 2 agrupamentos de
figuras, sendo que em um dos conjuntos faltam algumas figuras que estão no outro.
Peça a um aluno que complete o segundo grupo, levando-o a responder à seguinte
questão: “Quantas figuras você precisou colocar para que as quantidades ficassem
iguais?”.
A ação de completar pode ser explorada em atividades nas quais os alunos tenham
de completar uma tarefa já iniciada. Podemos utilizar folhas com desenhos para colorir ou
completar: Veja:

Maria tem 4 vasos.


- “Quantos estão com plantas?”
- “Quantos estão vazios?”
- “Complete o trabalho de Maria, desenhando flores nos
vasos vazios”.

1.2 O Algoritmo da Adição

Você já teve a oportunidade de analisar atividades que preparam o aluno para


adicionar corretamente, incluindo aquelas voltadas para a compreensão do sistema de
numeração – ou seja, estamos propondo adiar um pouco a introdução do algoritmo.
Agora, vamos discutir brevemente nossos motivos para propor que você considere esta
forma de trabalhar.
Em primeiro lugar, a habilidade de utilizar o algoritmo corretamente não se adquire
de uma só vez, pois requer tempo e prática. Por isso, o algoritmo da adição só deve ser
apresentado às crianças quando elas já dominarem, com certa segurança, o conceito da
operação, os fatos básicos e o sistema de numeração.
É importante ainda ficar claro que não estamos fazendo um bom uso do algoritmo
quando solicitamos a uma criança, um “arme e efetue” em adições como “5+2=” ou
“8+7=”. Os resultados destas adições são fatos básicos e o algoritmo da adição não ajuda a
criança a efetuar a operação. Nesses casos, é mais adequada a resolução por meio do
cálculo mental (iniciando o processo de memorização com o auxílio de materiais de
contagem). Na verdade, para que a criança utilize bem o algoritmo quando for operar com
as representações dos números dispostas em colunas, ela precisará de boas estratégias
mentais para determinar os resultados das adições de números de um algarismo.
Finalmente, consideramos que no processo de construção do algoritmo da adição, é
recomendável que os primeiros exemplos já envolvam adições com “reservas”, ou seja,

Conteúdos  e  Métodos  para  o  Ensino  da  Matemática  


Pedagogia  –  FAC-­‐UNILAGOS  
 
                             CONTEÚDOS  E  MÉTODOS  PARA  O  ENSINO  DA  MATEMÁTICA   5  
 

aquelas em que a soma das unidades isoladas é maior que nove, sendo necessário fazer
um agrupamento para a casa das dezenas. Trabalhando com “reserva” desde o início, o
aluno compreende porque é necessário começar a operar pelas unidades, isto é, da direita
para a esquerda, o que contraria seus hábitos de leitura. Por outro lado, ao trabalharmos
os primeiros exemplos sem reservas, o resultado da operação será o mesmo se operarmos
da esquerda para direita ou vice-versa. Tal estratégia não permite ao aluno perceber que,
na utilização do algoritmo, há uma nítida vantagem em se iniciar o processo pela ordem
das unidades.

A figura ao lado mostra a utilização de materiais


concretos e do QVL para registro do algoritmo da adição.

(a) Discutam e escrevam um roteiro explicativo das


três etapas realizadas com os palitos.
(b) Descrevam a relação das etapas realizadas com o
material concreto e o registro do algoritmo formal.

A figura abaixo mostra o material dourado e o QVL,


usados de forma integrada, para adicionar 87 a 161.

(a) Discutam e descrevam o que está sendo


representado em cada uma das quatro linhas do
quadro.
(b) Que dificuldade da compreensão do algoritmo
este tipo de trabalho pode ajudar a superar? Por
quê?

1.3 Introduzindo o Algoritmo da Subtração

O algoritmo da subtração tem finalidade similar ao da adição, ou seja, sistematizar e


facilitar o processo de cálculo. Ele deve ser apresentado quando as crianças já dominarem,
com certa segurança, os conceitos associados à subtração, o sistema de numeração, os fatos
básicos da subtração e o algoritmo da adição. Novamente chamamos sua atenção para o
fato de que a habilidade de utilizar o algoritmo corretamente requer tempo e prática,
sendo necessárias diversas experiências preparatórias, variando-se bastante os valores
numéricos.

Para facilitar a discussão das sugestões de


atividades, vamos apresentar desde já a
nomenclatura associada ao algoritmo da
subtração, lembrando que não há sentido em
pedir aos alunos que memorizem estes termos.

Conteúdos  e  Métodos  para  o  Ensino  da  Matemática  


Pedagogia  –  FAC-­‐UNILAGOS  
 
                             CONTEÚDOS  E  MÉTODOS  PARA  O  ENSINO  DA  MATEMÁTICA   6  
 

De um modo geral, o uso correto da linguagem matemática não deve ser o foco
principal. Os alunos precisam compreender que os termos desta linguagem nos ajudam a
conversar, comunicar e defender nossos pensamentos e nossa forma de resolver
problemas e cálculos. No entanto, você, professora ou professor, deve utilizar a linguagem
matemática corretamente. Deve ainda estimular o debate e o registro, pois essas atitudes
farão com que os alunos assimilem, aos poucos, o vocabulário que for relevante a cada
momento de sua aprendizagem.

1.3.1 O algoritmo da subtração e a ação de retirar

Ao iniciarmos o algoritmo da subtração, devemos usar, como na adição, materiais


de contagem e o QVL. Lembramos que, dentre as ações associadas à subtração, a mais
natural para a criança é a de retirar e, por isso, vale a pena iniciar o estudo do algoritmo da
subtração usando esta idéia.
Para representar com material concreto a idéia de retirar, a criança deve separar, de
seu material de contagem, apenas a quantidade que representa o minuendo. A seguir, ela
deve retirar deste grupo de objetos a quantidade que corresponde ao subtraendo. A ação
de retirar, da coleção de objetos que representa o minuendo, uma quantidade
correspondente ao valor do subtraendo só faz sentido quando trabalhamos com apenas
uma mesma coleção de objetos. Retiramos algo daquilo que temos!
Por meio de exemplos, vamos estudar como atividades que exploram a ação de
retirar podem ser desenvolvidas concretamente.

Exemplo 1:
Enuncie, oralmente, uma situação–problema
envolvendo a ação de retirar. Como exemplo
vamos retirar 13 de 25.

Peça aos alunos que arrumem 25 palitos em


um QVL, como na figura ao lado. Você pode
construir em papel pardo, por exemplo,
quadros com apenas duas linhas para que os
alunos, ou grupos de alunos, trabalhem
independentemente.

Diga aos alunos:


- “Agora vamos resolver o nosso problema, ou seja, tirar 13 palitos dos 25 palitos”.
- “Mude para a linha debaixo os palitos que representam a quantidade que você precisa tirar”.
- “Quantos palitos permaneceram na primeira linha?”
- “Na primeira linha fica a quantidade de palitos que sobrou de 25 depois de tirarmos 13 (ou seja, o
resto!)”.

Conteúdos  e  Métodos  para  o  Ensino  da  Matemática  


Pedagogia  –  FAC-­‐UNILAGOS  
 
                             CONTEÚDOS  E  MÉTODOS  PARA  O  ENSINO  DA  MATEMÁTICA   7  
 

Por meio de conversas como a que exemplificamos,


mostre às crianças que a quantidade de palitos da
segunda linha representa o que foi retirado
(subtraendo), e que a quantidade que sobrou na
primeira linha é o resultado da operação. Logo: 25–
13=12.

Trabalhando com material concreto você pode propor diversas situações. Isto vai
ajudar seu aluno a perceber a seqüência de ações que compõe o algoritmo. A
representação, no caderno, dos passos realizados com material concreto também é
importante para que o aluno, aos poucos, compreenda a relação entre estes passos e o
registro formal do algoritmo.
Usando o exemplo anterior, veja como você pode estimular esta associação entre
concreto e a representação escrita.

Após a representação do minuendo:


- “Vamos representar este número no caderno?”
- “Façam um QVL e anotem esta quantidade de palitos”

Após a retirada dos 13 palitos (o subtraendo):


- “Vamos anotar agora, abaixo do número 25, a quantidade de palitos que foi
retirada.”

E para finalizar:
- “Agora vamos fazer um traço para separar o resultado final e anotar quantos
palitos sobraram depois da retirada.”

Conteúdos  e  Métodos  para  o  Ensino  da  Matemática  


Pedagogia  –  FAC-­‐UNILAGOS  
 
                             CONTEÚDOS  E  MÉTODOS  PARA  O  ENSINO  DA  MATEMÁTICA   8  
 

Exemplo 2:

É possível usar estas idéias em uma subtração


na qual é preciso desfazer as dezenas
rearrumando o minuendo. Crie uma situação
problema para os alunos subtraírem 5 de 32.
Iniciamos por arrumar o minuendo na tabela.
Explique aos alunos que eles só possuem 2
unidades não agrupadas e por isso não podem retirar 5 unidades. No entanto, é
importante que eles percebam que o número 32 possui trinta e duas unidades, e o que
“atrapalha” a realização concreta da retirada é apenas a forma como os objetos estão
organizados.

Assim, os alunos devem concluir que será


preciso desfazer uma das dezenas (que
contém10 unidades). Após desamarrarem uma
dezena e a passarem para a casa das unidades,
os palitos ficarão com a seguinte disposição:

Esse é um bom momento para ajudá-los a perceber que o número representado continua
sendo o mesmo (32). A decomposição é que mudou: a forma inicial (3 dezenas e 2
unidades) foi alterada para: 2 dezenas e doze unidades.

Pergunte aos alunos:


- “O número mudou?” (não) “Então, o que mudou?” (a forma de decompor)
- “Quantas unidades estão agora registradas na primeira ordem?” (12)
- “E agora, podemos tirar 5 unidades de 12 unidades?” (sim)
- “Com quantas unidades ainda ficamos?” (7)
- “Com quantas dezenas ainda ficamos?” (2)

Bem, agora é possível retirar 5 palitos dos que ficaram na ordem das unidades e o material
fica com a disposição mostrada no quadro abaixo.

Observe que o registro escrito dos passos da


operação pode ou não incluir a passagem na
qual uma dezena foi desagrupada em 10
unidades.

Varie os materiais de contagem, pois isto ajuda o aluno a compreender o processo sem
se fixar no material, o que possibilitará a necessária abstração.

Conteúdos  e  Métodos  para  o  Ensino  da  Matemática  


Pedagogia  –  FAC-­‐UNILAGOS  
 
                             CONTEÚDOS  E  MÉTODOS  PARA  O  ENSINO  DA  MATEMÁTICA   9  
 

 Para ilustrar o uso de um outro material, vamos subtrair 17 de 35.


Faça você as etapas, utilizando o QVL e, por exemplo, o material dourado.

O uso de material concreto facilita bastante à compreensão dos algoritmos e ajuda a


consolidar a aprendizagem das características de nosso sistema de numeração. Numa
etapa seguinte, você pode propor exemplos nos quais o zero aparece na casa das dezenas,
como tirar 25 de 208.
Você poderá verificar como o uso de material concreto ajuda em situações como
esta que costuma ser considerada difícil na operação de subtração.

 Faça você mesmo as etapas da subtração 208–25, usando o QVL e uma representação
de material concreto.

Destacamos que a professora ou o professor deve, sempre que possível, conhecer e


apresentar aos alunos mais de um procedimento. Possibilitar ao aluno a chance de
experimentar diferentes ações é fundamental para que ele desenvolva o senso crítico e
tenha o direito de escolher a estratégia com a qual mais se identifica, ou aquela que
possibilita compreender melhor o que está fazendo. Muitas vezes, uma criança com
dificuldade de compreender um procedimento ou conceito, resolve este obstáculo inicial
quando é apresentada a outros caminhos ou formas de raciocinar.
Assim, sugerimos que você pesquise sobre como as ações de comparar e completar
podem auxiliar o desenvolvimento de estratégias de cálculo para efetuar uma subtração.

O olhar dos alunos

Ao lado, apresentamos o registro de Bruno para efetuar a operação 920 –


709
 Expliquem o pensamento de Bruno. O que ele acerta? O que ele erra?

A ORDEM DAS PARCELAS ALTERA O RESULTADO?

Propriedade Comutativa

Complete a tabela a seguir efetuando as operações de adição.

Conteúdos  e  Métodos  para  o  Ensino  da  Matemática  


Pedagogia  –  FAC-­‐UNILAGOS  
 
                             CONTEÚDOS  E  MÉTODOS  PARA  O  ENSINO  DA  MATEMÁTICA   10  
 

• que você pode observar de regularidade entre os resultados das 3ª e 4ª colunas?

• Que conjectura você faria sobre essa observação?


Nos Anos Iniciais, as propriedades devem ser abordadas por meio de seus diferentes
usos. Quando uma criança tem que operar 8 + 21, e ela “guarda na cabeça” o 21 (vinte e
um) e conta nos dedos, acrescentando o 8 (oito), fazendo 22, 23, 24, 25, 26, 27, 28, 29, ela
está fazendo uso da propriedade comutativa da adição.

Podemos entender propriedades de uma operação como verdades que se verificam


com todos os elementos de um conjunto que no caso é o conjunto dos números naturais.

A adição é comutativa, ou seja, para quaisquer dois números naturais a e b, temos:


a+b=b+a

Comutar significar trocar, por isso adicionar 7 e 5 pode ser feito de duas diferentes formas.

A seguir, temos uma tabela de dupla entrada (linha e colunas):

Faça as adições completando a tabela:

• Identifique regularidades na tabela.

• Uma das regularidades que podemos observar é que alguns resultados se repetem.
Por que isso acontece?

Conteúdos  e  Métodos  para  o  Ensino  da  Matemática  


Pedagogia  –  FAC-­‐UNILAGOS  
 
                             CONTEÚDOS  E  MÉTODOS  PARA  O  ENSINO  DA  MATEMÁTICA   11  
 

Faça a subtração entre a coluna e a linha.

• Foi possível encontrar todos os resultados?

• Existe algum resultado que se repete?

Pense na subtração e nos números 7 e 5. Considerando as duas idéias envolvidas na


subtração, tirar e verificar quanto falta, se fizermos 7 – 5 = 2, faz sentido tirar 5 de 7, ou
pensar em, quanto falta ao 5 para completar 7. Se trocarmos a ordem e fizermos 5 – 7, as
duas idéias que envolvem a subtração no conjunto dos números naturais não fazem
sentido, ou seja, não podemos tirar 7 de 5, nem verificar quanto falta ao 7 para completar 5.
Assim, podemos verificar que a subtração não é comutativa.

O fato de 5 – 7 ser igual a –2, que não é um número natural, nos remete a outra
propriedade, o fechamento.

RELEMBRANDO O FECHAMENTO...

Propriedade do Fechamento
O que é fechamento? As teorias matemáticas trazem afirmações que por vezes nos
parecem evidentes, mas essas verdades são necessárias para a lógica interna das teorias.
Quando tomamos o conjunto dos números naturais e a operação adição, é sempre
possível adicionar dois números, em qualquer ordem, e encontrarmos como resultado um
número natural. Por isso, podemos afirmar que o conjunto dos números naturais é
fechado em relação à operação adição.

A adição possui a propriedade do fechamento, ou seja, para quaisquer dois


números naturais a e b, temos:
a + b é um número natural.

Conteúdos  e  Métodos  para  o  Ensino  da  Matemática  


Pedagogia  –  FAC-­‐UNILAGOS  
 
                             CONTEÚDOS  E  MÉTODOS  PARA  O  ENSINO  DA  MATEMÁTICA   12  
 

O mesmo não acontece quando tomamos o conjunto dos números naturais e a


operação subtração. Como vimos no item anterior, em relação ao fato de a subtração não
ser comutativa (5 – 7 = –2), embora os números 5 e 7 sejam naturais, o resultado –2 não é
um número natural, por isso, podemos afirmar que o conjunto dos números naturais não é
fechado em relação à operação subtração.

Propriedade Associativa

Complete a tabela a seguir associando as parcelas e resolvendo as operações


conforme indicado na primeira linha.

Se adicionarmos os números 7, 5 e 13, podemos fazer (7 + 5) + 13 ou 7 + (5 + 13), em


ambos os casos o resultado é o mesmo, 25. Por isso, podemos afirmar que a adição é
associativa. A propriedade associativa é usada em especial em cálculos mentais com
muitas parcelas, em que procuramos associar valores mais fáceis de serem operados.
Nesse exemplo, quando associamos 5 + 13 encontramos 18; adicionado ao 7 temos 25.

Enquanto na adição podemos associar quaisquer dois números e adicioná-los,


quando se trata da operação subtração isso já não acontece. Vamos explorar uma tabela e
verificar o que ocorre.

Observe os resultados das 4a e 5a colunas, eles são diferentes, por isso, podemos afirmar
que a subtração não é associativa.

Conteúdos  e  Métodos  para  o  Ensino  da  Matemática  


Pedagogia  –  FAC-­‐UNILAGOS  
 
                             CONTEÚDOS  E  MÉTODOS  PARA  O  ENSINO  DA  MATEMÁTICA   13  
 

O ELEMENTO NEUTRO DA ADIÇÃO

Propriedade do Elemento Neutro


Uma importante regularidade que pode ser observada é quando adicionamos
qualquer número natural a zero. O resultado dessa adição será sempre o próprio número.
Por isso dizemos que o zero (0) é o elemento neutro da adição.

REFERÊNCIAS:

BARBOSA, Andréia Carvalho Maciel; SILVA, Ana Lúcia Vaz da. Sistema de Numeração
Decimal. In: Matemática na Educação 1, v.1. Rio de Janeiro: CECIERJ, 2010.

BRASIL, Ministério da Educação. Pró-Letramento Matemática: Programa de formação


continuada de professores dos anos/séries iniciais do Ensino Fundamental: Matemática.
– edição revista e ampliada, incluindo SAEB / Prova Brasil matriz de referência /
Secretaria de Educação Básica. Brasília: Ministério da Educação/ Secretaria de Educação
Básica, 2007.

MOÇO, Anderson. Diagnóstico em Matemática: você sabe o que eles já sabem? Nova
Escola. São Paulo, 2010. Disponível em: <http://revistaescola.abril.com.br/matematica/
pratica-pedagogica/diagnostico-incial-o-que-eles-ja-sabem-528156.shtml?page=0>
Acessado em: 20 ago. 2014.

Conteúdos  e  Métodos  para  o  Ensino  da  Matemática  


Pedagogia  –  FAC-­‐UNILAGOS  
 
Conjuntos Numéricos

Definição de Conjunto: Conjunto é o agrupamento de elementos que possuem características semelhantes. Os


Conjuntos numéricos especificamente são compostos por números.
Exemplo: conjunto dos números pares positivos: P = {2,4,6,8,10,12, ... }. Esta forma de representar um conjunto, pela
enumeração dos seus elementos, chama-se forma de listagem.

O mesmo conjunto também poderia ser representado por uma propriedade dos seus elementos ou seja, sendo x um
elemento qualquer do conjunto P acima, poderíamos escrever: P = { x | x é par e positivo } = { 2,4,6, ... } Relação de
pertinência Sendo x um elemento do conjunto numérico A , escrevemos x 0 A , onde o símbolo 0significa "pertence a".
Sendo y um elemento que não pertence ao conjunto A , indicamos esse fato com a notação y A.

O conjunto que não possui elementos , é denominado conjunto vazio e representado por φ . Com o mesmo raciocínio, e
opostamente ao conjunto vazio, define-se o conjunto ao qual pertencem todos os elementos, denominado conjunto
universo, representado pelo símbolo U. Assim é que, pode-se escrever como exemplos: i= { x; x ≠ x} e U = {x; x = x}.

Subconjunto
Se todo elemento de um conjunto A também pertence a um conjunto B, então dizemos que A é subconjunto de B e
indicamos isto por A d B. Notas: a) todo conjunto numérico é subconjunto de si próprio. ( A d A ) b) o conjunto vazio é
subconjunto de qualquer conjunto. (id A) c) se um conjunto A possui m elementos então ele possui 2m subconjuntos. d)
o conjunto formado por todos os subconjuntos de um conjunto A é denominado conjunto das partes de A e é indicado
por P(A). Assim, se A = {c, d} , o conjunto das partes de A é dado por P(A) = {φ , {c}, {d}, {c,d}} e) um subconjunto de A é
também denominado parte de A.

Conjuntos numéricos fundamentais


Entendemos por conjunto numérico, qualquer conjunto cujos elementos são números. Existem infinitos conjuntos
numéricos, entre os quais, os chamados conjuntos numéricos fundamentais, a saber: Conjunto dos números naturais N
= {0,1,2,3,4,5,6,... } Conjunto dos números inteiros Z = {..., -4,-3,-2,-1,0,1,2,3,... } Obs: é evidente que N d Z.

Conjunto dos números racionais


Q = {x; x = p/q com p 0 Z , q 0 Z e q … 0 }. Temos então que número racional é aquele que pode ser escrito na forma de
uma fração p/q onde p e q são números inteiros, com o denominador diferente de zero. Lembre-se que não existe
divisão por zero! São exemplos de números racionais: 2/3, -3/7, 0,001=1/1000, 0,75=3/4, 0,333... = 1/3, 7 = 7/1, etc.
Notas: a) é evidente que N d Z d Q. b) toda dízima periódica é um número racional, pois é sempre possível escrever
uma dízima periódica na forma de uma fração. Exemplo: 0,4444... = 4/9 _

Conjunto dos números irracionais


I = {x; x é uma dízima não periódica}. Exemplos de números irracionais: Π = 3,1415926... (número pi = razão entre o
comprimento de qualquer circunferência e o seu diâmetro) 2,01001000100001... (dízima não periódica) √ 3 =
1,732050807... (raiz não exata).

Conjunto dos números reais


R = { x; x é racional ou x é irracional}. Notas: a) é óbvio que N d Z d Q d R b) I d R c) I cQ = R d) um número real é
racional ou irracional, não existe outra hipótese!

Intervalos numéricos
Dados dois números reais p e q, chama-se intervalo a todo conjunto de todos números reais compreendidos entre p e
q , podendo inclusive incluir p e q. Os números p e q são os limites do intervalo, sendo a diferença p - q , chamada
amplitude do intervalo. Se o intervalo incluir p e q , o intervalo é fechado e caso contrário, o intervalo é dito aberto. A
tabela abaixo, define os diversos tipos de intervalos.
INTRODUÇÃO ............................................................................. 2
PRINCÍPIO FUNDAMENTAL DE CONTAGEM............................ 3
FATORIAL .................................................................................... 8
AGRUPAMENTOS ..................................................................... 10
ARRANJOS ................................................................................ 10
PERMUTAÇÕES........................................................................ 17
PERMUTAÇÃO COM ELEMENTOS REPETIDOS .................... 23
COMBINAÇÕES ........................................................................ 28
BINÔMIO DE NEWTON ............................................................. 36
PROBABILIDADES .................................................................... 46
PROBABILIDADE DA UNIÃO DE DOIS EVENTOS .................. 55
PROBABILIDADE CONDICIONAL............................................. 55
RESPOSTAS ............................................................................. 64
REFERÊNCIA BIBLIOGRÁFICA ................................................ 66

No final das séries de exercícios podem aparecer


sugestões de atividades complementares. Estas
sugestões referem-se a exercícios do livro
“Matemática” de Manoel Paiva fornecido pelo FNDE
e adotado pelo IFMG – Campus Ouro Preto durante
o triênio 2015-2017.

Todos os exercícios sugeridos nesta apostila se


referem ao volume 2.

O fato de apenas alguns exercícios estarem


indicados não significa que os demais devam ser
ignorados. Ao contrário, quanto mais exercícios você
fizer mais hábil você pode ficar.

MATEMÁTICA III 1 ANÁLISE COMBINATÓRIA, BINÔMIO DE NEWTON E PROBABILIDADES


INTRODUÇÃO poucos elementos, mas quando
trabalhamos com conjuntos muito
As duas figuras abaixo mostram numerosos, os métodos tornam-se
placas de veículos automotores. A primeira indispensáveis.
mostra o modelo atualmente utilizado no
Brasil e a segunda apresenta o modelo de Vamos ver alguns exemplos abaixo e
placas unificadas do Mercosul que deveria tentar preencher os espaços com a
ser utilizada a partir de janeiro de 2016. quantidade de elementos de cada conjunto:

Ex.1: Conjunto formado por todos os


números de dois algarismos distintos
formados, exclusivamente, pelos dígitos 1,
2 e 3:
A = (12, 13, 21, 23, 31, 32}, logo o conjunto
Por meio do endereço <www.vid A tem _____ elementos.
igal.ouropreto.ifmg.edu.br/placasveiculos>
ou utilizando o qr-code Ex.2: Conjunto formado
ao lado, você consegue pelas diagonais de um
acessar uma página que hexágono:
aponta para quatro
outras que trazem D = {AC, AD, AF, BD, BE, BF,
reportagens sobre a CE, CF, DF}, logo o conjunto
mudança no padrão das D tem _____ elementos.
placas. Um fato comum
em todas as reportagens
é que a nova placa permitirá obter mais de Ex.3: Conjunto formado
450 milhões de combinações diferentes. A pelas diagonais de um
placa antiga permitia menos de 18 milhões heptágono:
de combinações.
H=
O assunto que vamos estudar agora
nos permite encontrar números como estes
e a entender situações como a que Quantos elementos tem o conjunto H? ____
recentemente vivemos quando, aos
números dos nossos celulares, foi Ex.4: Conjunto das sequências de letras
acrescentado um dígito. que se obtém mudando a ordem das letras
da palavra ROMA (anagramas da palavra
A Análise Combinatória, visa ROMA)
desenvolver métodos que permitem contar P=
o número de elementos de um conjunto,
sendo estes elementos, agrupamentos
formados sob determinadas condições.

Estes métodos de contagem podem Quantos elementos tem o conjunto P?


parecer pouco necessários e realmente o
são quando lidamos com conjuntos que tem
CÁSSIO VIDIGAL 2 IFMG – CAMPUS OURO PRETO
Ex.5: K é o conjunto formado pelos números C1 → C2 C2 → C3 C1 → C3
de três algarismos distintos formados a 1 A 1A 1

partir dos dígitos 1, 2, 3, 4, 5, 6. 7 e 8. 1 B 1B 2

Quantos elementos tem K? 1 C 1C 3


1 D 1D 4
2 A 2A 5
2 B 2B 6
2 C 2C 7
2 D 2D 8
_______________ 3 A 3A 9
3 B 3B 10

Observe que neste caso, é bastante 3 C 3C 11

trabalhoso obter todos os elementos do 3 D 3D 12


4 A 4A 13
conjunto K para depois contá-los e 4 B 4B 14
notadamente sabemos quais as 4 C 4C 15

dificuldades. Usando as técnicas que 4 D 4D 16

estudaremos nas próximas páginas desta 5 A 5A 17


5 B 5B 18
apostila, veremos que o conjunto K tem 336 5 C 5C 19
elementos. 5 D 5D 20

Para cada uma das 5 estradas que


ligam C1 a C2, podemos escolher uma das
PRINCÍPIO FUNDAMENTAL DE 4 que ligam C2 a C3, assim, são 5 ∙ 4 = 20
CONTAGEM maneiras diferentes de viaja.

Acompanhe a seguir a resolução de alguns


problemas: Ex.2: Ao lançarmos uma
moeda e um dado,
quantos resultados
obtidos a partir da
combinação Cara ou
Ex.1: Uma pessoa quer viajar da Cidade C1 Coroa (da moeda) com o
para a Cidade C3 passando pela cidade C2. número (do dado) temos?
Sabendo que existem 5 estradas que ligam
C1 a C2 e outras 4 estradas que ligam C2 a Resolução:
C3, de quantas maneiras diferentes a Observe o diagrama a seguir:
pessoa poderá viajar?

Resolução:

Para facilitar, observe o diagrama:

MATEMÁTICA III 3 ANÁLISE COMBINATÓRIA, BINÔMIO DE NEWTON E PROBABILIDADES


Note que o evento “lançar moeda e
dado” tem duas etapas independentes com 1)
2 possibilidades na primeira e 6 Uma loja de roupas femininas vende cinco
possibilidades na segunda totalizando 12 modelos de calças Jeans (Pantalona, Flare,
possibilidades (2 ∙ 6 = 12). Legging, Sarouel e Skinny) e cada calça
pode ter uma de três cores (Preta, Marrom
Assim, de modo geral, podemos e Azul).
dizer que: a) Escreva todas as possíveis combinações
possíveis de calças nesta loja.
Se um evento é composto por m
etapas diferentes sucessivas e
independentes de tal maneira que
a etapa 1 tenha n1 possibilidades,
que a etapa 2 tenha n2
possibilidades, ..., que tenha nm
possibilidades, então o número
total de possibilidades de o evento
ocorrer é dado pelo produto 𝑛1 ∙
𝑛2 ∙ ⋯ ∙ 𝑛𝑚 . Esse é o Princípio b) Quantas opções de escolha terá uma
Fundamental de Contagem. consumidora interessada em uma calça
desta loja?

Vamos agora fazer alguns exercícios


envolvendo diversas situações que
permitem a aplicação do conceito do
Princípio Fundamental de Contagem.

2) Quantos números naturais de 3


algarismos podem ser escritos com os
algarismos 2, 3, 4, 5, e 6?

CÁSSIO VIDIGAL 4 IFMG – CAMPUS OURO PRETO


3) Quantos números naturais de 3 5) Aline e Bárbara
algarismos distintos podem ser escritos praticam natação. As
com os algarismos 2, 3, 4, 5, e 6? duas amigas resolvem
disputar uma corrida
na piscina. Quantos
resultados diferentes podemos obter no
final da disputa? Liste todos os possíveis.

4) Em um ginásio de esportes, os lugares 6) Aline, Bárbara e


destinados aos espectadores são Camila praticam
separados em quatro setores com a mesma natação. As três
quantidade de cadeiras em cada um deles: amigas resolvem
setor Azul, laranja, amarelo e verde. Em disputar uma corrida na piscina. Quantos
cada setor existem 26 filas de cadeiras resultados diferentes podemos obter no
identificadas pelas letras do alfabeto. Em final da disputa? Liste todos os possíveis.
cada fila estão 45 cadeiras numeradas de 1
a 45. O ingresso para o ginásio apresenta
uma sequência com uma cor, uma letra e
um número (Ex.: Azul J 25 indica a cadeira
25 da fila J do setor Azul). Qual o total de
cadeiras neste ginásio?

MATEMÁTICA III 5 ANÁLISE COMBINATÓRIA, BINÔMIO DE NEWTON E PROBABILIDADES


7) Aline, Bárbara, 9) Um edifício tem 8 portas. De quantas
Camila e Daniela formas uma pessoa ode entrar por uma
praticam natação. porta e sair por outra, diferente daquela que
As quatro amigas ele entrou?
resolvem disputar uma corrida na piscina.
Quantos resultados diferentes podemos
obter no final da disputa?

10) Uma prova possui 2 questões do tipo


“VERDADEIRO ou FALSO”. Liste todas as
maneiras diferentes que uma pessoa pode
8) Uma bandeira com a marcar, aleatoriamente, as questões.
forma ao lado, vai ser Quantas maneiras existem?
pintada utilizando duas das
três cores: vermelho, verde e
azul.

Utilize os modelos abaixo e


liste todas as possíveis
bandeiras. Quantas são elas?

11) Uma prova possui 3 questões do tipo


“VERDADEIRO ou FALSO”. Liste todas as
maneiras diferentes que uma pessoa pode
marcar, aleatoriamente, as questões.
Quantas maneiras existem?

CÁSSIO VIDIGAL 6 IFMG – CAMPUS OURO PRETO


12) Uma prova possui 4 questões do tipo 15) Se seis teclas literais (são 27) de um
“VERDADEIRO ou FALSO”. De quantas computador forem pressionadas,
maneiras diferentes uma pessoa pode sucessivamente, ao acaso, quantos
marcar, aleatoriamente, as questões? anagramas diferentes podem ser
formados?

13) Uma prova possui 10 questões do tipo


“VERDADEIRO ou FALSO”. De quantas
maneiras diferentes uma pessoa pode
marcar, aleatoriamente, as questões?
16) Se seis teclas literais (são 27) de um
computador forem pressionadas,
sucessivamente, ao acaso, quantos
anagramas formado por letras distintas
podem ser formados?

14) Uma prova possui 20 questões de cinco


alternativas (A, B, C, D e E). De quantas
maneiras diferentes uma pessoa pode
marcar, aleatoriamente, o gabarito?

______________________
ATIVIDADES COMPLEMENTARES
Pág. 147 – Exercícios 04 a 10

MATEMÁTICA III 7 ANÁLISE COMBINATÓRIA, BINÔMIO DE NEWTON E PROBABILIDADES


FATORIAL
12!
O fatorial de um número consiste em Ex.1:Vamos calcular o valor de .
8!
um relevante mecanismo nos estudos
envolvendo análise combinatória, pois a Resolução:
multiplicação de números naturais Desenvolvendo 12! no numerador da
consecutivos é muito utilizada nos fração, podemos reescrevê-la assim:
processos de contagem.
12! 12 ∙ 11 ∙ 10 ∙ 9 ∙ 8!
Fatorial de um número consiste em =
8! 8!
multiplicar o número por todos os seus
antecessores até o número 1. simplificando numerador e denominador por
8!, temos:
Representamos o fatorial de um 12! 12 ∙ 11 ∙ 10 ∙ 9 ∙ 8!
= = 12 ∙ 11 ∙ 10 ∙ 9 = 11880
número natural por n! e o desenvolvimento 8! 8!
de n! é dado por:

𝑛! = 𝑛 ∙ (𝑛 − 1) ∙ (𝑛 − 2) ∙ ⋯ ∙ 3 ∙ 2 ∙ 1
7!+8!
Observação: Ex.2: Simplificar a expressão .
9!
A definição acima é válida para 𝑛 ≥ 2. Para
n = 1 e n = 0, definimos que: Resolução:
1! = 1 e 0! = 1
7! + 8! 7! + 8 ∙ 7! 7! (1 + 8)
= = =
9! 9! 9!
7! ∙ 9 1
2! = 2 ∙ 1 = 2 = =
9 ∙ 8 ∙ 7! 8
3! = 3 ∙ 2 ∙ 1 = 6
4! = 4 ∙ 3 ∙ 2 ∙ 1 = 24
5! = 5 ∙ 4 ∙ 3 ∙ 2 ∙ 1 = 120
6! = 6 ∙ 5 ∙ 4 ∙ 3 ∙ 2 ∙ 1 = 720 (𝑛+3)!
Ex.3: Simplificar a expressão (𝑛+1)!.
7! = 7 ∙ 6 ∙ 5 ∙ 4 ∙ 3 ∙ 2 ∙ 1 = 5040
8! = 8 ∙ 7 ∙ 6 ∙ 5 ∙ 4 ∙ 3 ∙ 2 ∙ 1 = 40320
9! = 9 ∙ 8 ∙ 7 ∙ 6 ∙ 5 ∙ 4 ∙ 3 ∙ 2 ∙ 1 = 362880 Resolução:
10! = 10 ∙ 9 ∙ 8 ∙ 7 ∙ 6 ∙ 5 ∙ 4 ∙ 3 ∙ 2 ∙ 1 = 3628800
(𝑛 + 3)! (𝑛 + 3)(𝑛 + 2)(𝑛 + 1)!
= =
Observação: (𝑛 + 1)! (𝑛 + 1)!
Observe a situação abaixo = (𝑛 + 3)(𝑛 + 2) = 𝑛2 + 5𝑛 + 6
8! = 8 ∙ ⏟
7∙6∙5∙4∙3∙2∙1
𝑛!
7! Ex.4: Calcule 𝑛 tal que (𝑛−1)! = 4.
8! = 8 ∙ 7!

Isso vale para qualquer fatorial, Resolução:


assim, 𝑛! = 𝑛 ∙ (𝑛 − 1)! 𝑛! 𝑛 ∙ (𝑛 − 1)!
=4→ =4→𝑛=4
(𝑛 − 1)! (𝑛 − 1)!
Alguns cálculos envolvendo fatorial
exigem algumas técnicas de simplificação e
fatoração. Observe os exemplos a seguir
CÁSSIO VIDIGAL 8 IFMG – CAMPUS OURO PRETO
𝑛! 10!∙4!
Ex.5: Calcule 𝑛 tal que (𝑛−2)! = 42. c)
8!∙6!
𝑛! 𝑛 ∙ (𝑛 − 1)(𝑛 − 2)!
= 42 → = 42
(𝑛 − 2)! (𝑛 − 2)!

𝑛 ∙ (𝑛 − 1) = 42 → 𝑛2 − 𝑛 − 42 = 0
𝑛!
resolvendo a equação do 2º grau, d) (𝑛−2)!
encontramos 𝑛1 = 7 e 𝑛2 = −6. Como
fatorial só é definido para números naturais,
então 𝑛 = 7.

(𝑛−3)!
e) (𝑛−1)!

18) Resolver a equação:


(𝑛 + 1)!
= 20
(𝑛 − 1)!

17) Simplificar as frações:


10!
a) 9!

8!
b) 10!

______________________
ATIVIDADES COMPLEMENTARES
Pág. 152 – Exercícios 17 a 22

MATEMÁTICA III 9 ANÁLISE COMBINATÓRIA, BINÔMIO DE NEWTON E PROBABILIDADES


AGRUPAMENTOS a duas. Esses agrupamentos são
chamados de combinações porque a ordem
O Princípio Fundamental da com que são misturadas as duas cores
Contagem (PFC) é a principal técnica para primárias não altera a mistura:
resolução de problemas de contagem,
VERMELHO + AZUL = AZUL + VERMELHO
muitas vezes porém, se utilizarmos apenas
o PFC a resolução desses problemas pode
Nas próximas sessões desta apostila
se tornar trabalhosa.
estudaremos estes tipos de agrupamentos
e peculiaridades em cada caso.
Em nosso cotidiano, formamos
agrupamentos em várias situações. Por
exemplo ao escolher colegas para um
ARRANJOS
trabalho escolar em grupo, formamos
agrupamentos de pessoas. Ao discutir
sobre os possíveis quatro primeiros
colocados do Campeonato Brasileiro de
Futebol, formamos agrupamentos de Cinco amigos: André, Beto, Carlos, Daniel e
clubes de futebol etc. Edio disputam uma corrida. Os dois
primeiros serão premiados. De quantas
A análise combinatória identifica dois formas diferentes podemos entregar estes
tipos básicos de agrupamentos: os arranjos prêmios?
e as combinações.
Resolução:
Os arranjos são agrupamentos em
que se considera a ordem dos Na tabela abaixo, indicamos uma
elementos; qualquer mudança da ordem representação de todas as possibilidades:
dos elementos altera o agrupamento. Por
exemplo, ao formar números naturais de (André, Beto) (Carlos, Daniel)
três algarismos distintos escolhidos dentre (André, Carlos) (Carlos, Edio)
os algarismos 2, 4, 6, 7 e 8, estaremos (André, Daniel) (Daniel, André)
arranjando esses cinco algarismos três a (André, Edio) (Daniel, Beto)
três. Esses números são chamados de (Beto, André) (Daniel, Carlos)
arranjos de algarismos porque, mudando a (Beto, Carlos) (Daniel, Edio)
ordem dos algarismos em um desses (Beto, Daniel) (Edio, André)
números, obtemos outro número, por (Beto, Edio) (Edio, Beto)
exemplo: (Carlos, André) (Edio, Carlos)
(Carlos, Beto) (Edio, Daniel)
246 ≠ 462
Observe que cada possibilidade
Já as combinações são representada na tabela corresponde a um
agrupamentos em que não se considera a agrupamento ordenado que duas pessoas
ordem dos elementos; portanto, escolhidas entre os cinco amigos.
mudanças na ordem dos elementos não
alteram o agrupamento. Um pintor ao Note, por exemplo, que o par
produzir uma mistura com duas cores ordenado (André, Beto) é diferente do par
primárias distintas escolhidas entre ordenado (Beto, André) pois na primeira
vermelho, azul e amarelo, estará situação André é vencedor e Beto vice
combinando essas três cores tomadas duas
CÁSSIO VIDIGAL 10 IFMG – CAMPUS OURO PRETO
enquanto na segunda situação Beto é o pois já fizemos a escolha anterior e não
vencedor e André fica em segundo lugar. há repetição de elementos.

Dizemos que cada resultado da  feitas as duas primeiras escolhas, há


corrida corresponde a um arranjo de 5 𝑛 – 2 maneiras diferentes de escolher o
elementos (amigos) tomados dois a dois terceiro elemento da sequência, pois
(isto é, escolhidos dois entre os cinco para não pode haver repetição.
formar o agrupamento ordenado).

Vamos, por meio do PFC, contar o
número total de arranjos possíveis e  para escolher o p-ésimo elemento, a
indicaremos por A5,2 (Lemos: arranjo de 5 partir de 𝑝 – 1 escolhas anteriores ,
elementos tomados dois a dois) sobram 𝑛 – (𝑝 – 1) opções1.
1) Para ocupar a primeira posição há cinco Assim, pelo PFC, a quantidade de
possibilidades; e
2) definido o primeiro lugar, sobram 4 opções para arranjos possíveis (indicadas por An,p) é:
preencherem a segunda posição.
𝐴𝑛,𝑝 = 𝑛 ∙ (𝑛 − 1) ∙ (𝑛 − 2) ∙ ⋯ ∙ (𝑛 − 𝑝 + 1)
Assim:
1ª posição 2ª posição
5 x 4 = 20 Para obter uma expressão
equivalente a esta acima usando o fatorial,
Então, vamos multiplica-la e dividi-la por um
𝐴5,2 = 5 ∙ 4 = 20 mesmo número (𝑛 − 𝑝)!
______________________
𝑛 ∙ (𝑛 − 1) ∙ ⋯ ∙ (𝑛 − 𝑝 + 1) ∙ (𝑛 − 𝑝)!
𝐴𝑛,𝑝 = =
Assim, definimos por arranjo de n (𝑛 − 𝑝)!
𝑛 ∙ (𝑛 − 1) ∙ ⋯ ∙ (𝑛 − 𝑝 + 1) ∙ (𝑛 − 𝑝) ∙ (𝑛 − 𝑝 − 1) ∙ ⋯ ∙ 1
elementos tomados p a p como qualquer =
(𝑛 − 𝑝)!
agrupamento ordenado de p elementos
Assim:
escolhidos entre os n existentes.

Fórmula do Arranjo 𝒏!
𝑨𝒏,𝒑 =
Dados n elementos distintos, vamos (𝒏 − 𝒑)!
indicar por An,p o número de arranjos
desses elementos tomados p a p.
Vamos usar o PFC.
Ex.1: Aline,
 O primeiro elemento da sequência pode Bárbara, Camila,
ser escolhido de 𝑛 formas possíveis. Daniela e Eduarda
praticam natação.
 O segundo elemento da sequência pode As cinco amigas resolvem disputar uma
ser escolhido de 𝑛 – 1 maneiras distintas, corrida na piscina. De quantas formas
diferentes pode ser preenchido um pódio de
3 lugares?

1Como 𝑛 − (𝑝 − 1) = 𝑛 − 𝑝 + 1 então optaremos por


esta segunda notação.
MATEMÁTICA III 11 ANÁLISE COMBINATÓRIA, BINÔMIO DE NEWTON E PROBABILIDADES
Resolução 1: Ex.3: Uma família é composta por seis
Aplicando o PFC, montamos o seguinte pessoas (pai, mãe e quatro filhos) que
esquema: nasceram em meses diferentes do ano.
Calcule as sequências dos possíveis meses
1ª pos. 2ª pos 3ª pos de nascimento dos membros dessa família.
5 x 4 x 3 = 60
Resolução: Sabemos que 1 ano é composto
Resolução 2: de 12 meses, então devemos determinar o
Aplicando o conceito de Arranjo: número de sequência através do arranjo de
𝑛! 12, tomados 6 a 6.
𝐴𝑛,𝑝 =
(𝑛 − 𝑝)!
5! 5 ∙ 4 ∙ 3 ∙ 2! 12!
𝐴5,3 = = = 60 𝐴12,6 = = 665.280
(5 − 3)! 2! (12 − 6)!

Logo são 60 possibilidades diferentes de


se compor o pódio.

19) Para ocupar os cargos de Presidente e


Ex.2: De quantas formas diferentes é vice-presidente do Grêmio de um colégio,
possível organizar 6 passageiros num candidataram-se dez alunos. De quantos
ônibus de 48 lugares? modos distintos pode ser feita essa
escolha?
Resolução:
48! 48!
𝐴48,6 = =
(48 − 6)! 42!

48!
Resposta:
42!
(ou, efetuando a operação, 8.835.488.640 de formas diferentes.)

Ex.3: Em uma urna de sorteio de prêmios


existem dez bolas enumeradas de 0 a 9.
Determine o número de possibilidades
existentes num sorteio cujo prêmio é
formado por uma sequência de 6
algarismos.

Resolução 1:
10! 10!
𝐴10,6 = = = 151.200
(10 − 6)! 4!

Resolução 2:

10 x 9 x 8 x 7 x 6 x 5 = 151200

Logo, são 151 200 possibilidades


diferentes.

CÁSSIO VIDIGAL 12 IFMG – CAMPUS OURO PRETO


20) A senha de acesso a uma rede de 22) Em uma pesquisa encomendada por
computadores é formada por uma uma operadora turística com o objetivo de
sequência de quatro letras distintas seguida descobrir os destinos nacionais mais
por dois algarismos distintos: (considere as cobiçados pelos brasileiros, o entrevistado
26 letras do alfabeto) deve escolher, em ordem de preferência,
a) quantas são as possíveis senhas de três destinos entre os dez apresentados
acesso? pelo entrevistador. Um dos destinos
apresentados é a cidade de Ouro Preto.
a) quantas respostas diferentes podem ser
obtidas?

b) quantas senhas apresentam


simultaneamente apenas consoantes e
algarismos maiores que 5?

b) Quantas respostas possíveis apresentam


a cidade de Ouro Preto como a preferida?

21) Calcule
a) A7,3

c) Quantas respostas possíveis não contém


b) A11,2 Ouro Preto entre os destinos mencionados?

c) A5,1

d) A5,5

MATEMÁTICA III 13 ANÁLISE COMBINATÓRIA, BINÔMIO DE NEWTON E PROBABILIDADES


23) Responda 24) Para eleição do corpo dirigente de uma
a) Quantos números de três algarismos empresa, oito pessoas são pré-
distintos podem ser formados dispondo-se selecionadas. de quantas maneiras
dos algarismos de 1 a 9? distintas poderão ser escolhidos presidente,
vice-presidente e diretor financeiro?

b) Quantos números de 3 algarismos


podem ser Dispondo-se dos algarismos de
1 a 9?
25) A primeira fase de um torneio de futebol
é disputada por 15 equipes no sistema de
turno e returno ( a equipe A, por exemplo,
joga com a equipe B duas vezes: uma em
seu campo e outra do campo adversário)
quantas partidas são disputadas ao todo?

c) Quantos números de 3 algarismos


podem ser formados dispondo-se dos
algarismos de 1 a 9 em que há a repetição
de ao menos um algarismo?

CÁSSIO VIDIGAL 14 IFMG – CAMPUS OURO PRETO


26) Resolva a equação An,2 = 110 b) Em quantos resultados só aparecem
atletas europeus nas três primeiras
posições?

c) Em quantos resultados o atleta brasileiro


recebe medalha?

27) Em um torneio
internacional de natação
participaram cinco atletas
europeus, dois americanos
e um brasileiro.

a) De quantos modos distintos poderão ser


distribuídas as medalhas de ouro, prata e
bronze?

d) Supondo que o atleta brasileiro não


recebeu medalha, determine o número de
resultados em que há mais atletas europeus
do que americanos no pódio.

MATEMÁTICA III 15 ANÁLISE COMBINATÓRIA, BINÔMIO DE NEWTON E PROBABILIDADES


28) O logotipo de uma empresa é 29) Seis amigos
representado pelos três círculos a seguir participam de uma
brincadeira de
futebol que consiste
na cobrança de
pênaltis. Cada um
escolhe, de todas as
formas possíveis, um colega para bater o
ainda não foram escolhidas as cores que pênalti e outro para tentar defendê-lo.
serão usadas para colorir cada círculo. o
departamento de marketing sugeriu o uso a) Quantas cobranças de pênalti são feitas
de azul, laranja, verde, branco, Vermelho nessa brincadeira?
ou gelo. sabendo que cada círculo será
pintado de uma cor diferente, determine:
a) o número de maneiras de colorir o
logotipo.

b) Quantas cobranças haveria se o grupo


resolvesse convidar um sétimo amigo para
que ele escolhesse, de todas as formas
possíveis, o cobrador e o defensor dos
pênaltis?

b) o número de maneiras de colorir o


logotipo incluindo obrigatoriamente a cor
laranja.

CÁSSIO VIDIGAL 16 IFMG – CAMPUS OURO PRETO


PERMUTAÇÕES
Na análise combinatória, as
Ex.1: Oito carros participam de uma corrida.
permutações dos elementos de uma
De quantas formas diferentes eles podem
sequência são um tipo particular de arranjo,
chegar ao final?
como mostra a situação abaixo.
Resolução:
Ao formar os números naturais de
três algarismos distintos com os algarismos
2, 5 e 8, estamos formando os arranjos 𝑃8 = 8! = 8 ∙ 7 ∙ 6 ∙ 5 ∙ 4 ∙ 3 ∙ 2 ∙ 1 = 40320
simples desses três algarismos tomados
três a três. Observe: Ex.2: Dez CDs diferentes sendo
seis de música clássica e quatro de
258 285 música popular devem ser
528 582 colocados lado a lado no porta
825 852 CDs.
De quantas formas diferentes estes
Dois quaisquer desses arranjos se discos podem ser dispostos de modo que os de
diferenciam apenas pela ordem dos música clássica fiquem juntos e os de música
elementos componentes, e não pela popular também fiquem juntos?
natureza dos elementos, já que todos esses
arranjos possui os mesmos elementos: 2, 5 Resolução: Como os CDs de mesmo estilo
e 8. por isso dizemos que cada um desses devem ficar juntos, temos duas opções:
arranjos é uma permutação simples dos Primeira Opção:
algarismos 2, 5 e 8. Música Clássica M. Popular
Dados os elementos distintos do
conjunto K = {a1, a2, a3, ..., an}, ↓ ↓ ↓ ↓ ↓ ↓ ↓ ↓ ↓ ↓
chama-se permutação simples 6 5 4 3 2 1 4 3 2 1
dos n elementos de K todo arranjo
simples desses n elementos Pelo princípio multiplicativo:
tomados n a n. 6 ∙ 5 ∙ 4 ∙ 3 ∙ 2 ∙ 1 ∙ 4 ∙ 3 ∙ 2 ∙ 1 = 6! ∙ 4!
(ou 𝑃6 ∙ 𝑃4 )
Fórmula da Permutação Segunda Opção
M. Popular Música Clássica
Conforme consta na definição do
quadro acima, a permutação de n ↓ ↓ ↓ ↓ ↓ ↓ ↓ ↓ ↓ ↓
elementos de um conjunto é o arranjo dos n 4 3 2 1 6 5 4 3 2 1
elementos tomados n a n. Assim:
Pelo princípio multiplicativo:
𝑛! 𝑛! 𝑛! 4 ∙ 3 ∙ 2 ∙ 1 ∙ 6 ∙ 5 ∙ 4 ∙ 3 ∙ 2 ∙ 1 = 4! ∙ 6!
𝑃𝑛 = 𝐴𝑛,𝑛 = = = = 𝑛!
(𝑛 − 𝑛)! 0! 1 3 ∙ 2 ∙ 1 ∙ 4 ∙ 3 ∙ 2 ∙ 1 = 6! ∙ 4!
(ou 𝑃4 ∙ 𝑃6 )
Assim:
Para sabermos o total de possibilidades,
somamos os resultados
𝑷𝒏 = 𝒏! 6! ∙ 4! + 6! ∙ 4! = 17280 + 17280 = 34 560

R: Os CDs podem ser guardados de 34 560


maneiras diferentes.
MATEMÁTICA III 17 ANÁLISE COMBINATÓRIA, BINÔMIO DE NEWTON E PROBABILIDADES
Ex.2: Considerando a palavra NUMEROS: c)
Para encontrar a quantidade de anagramas
a) Quantos anagramas podemos formar? começados pela letra N e terminados em S,
fixamos estas letras e fazemos as
b) Quantos anagramas começam por N? permutações de todas as demais.
N S
c) Quantos anagramas começam por N e
terminam com S?
𝑃5 = 5! = 120
d) Quantos anagramas começam com uma
vogal? R: 120 anagramas

e) Quantos anagramas terminam com uma


consoante?
d)
f) Quantos anagramas começam por uma Neste caso, existem três possibilidades
vogal e terminam com uma consoante? para a primeira posição: E, O ou U. Para
cada vogal fixada na primeira posição,
g) Quantos anagramas apresentam as sobram seis letras para permutar.
letras N, U e M juntas e nesta ordem?
.
h) Quantos anagramas apresentam as
letras N, U e M juntas em qualquer ordem?

Resolução 3 𝑃6

3 ∙ 𝑃6 = 3 ∙ 720 = 2160
a)
Os anagramas da palavra NUMEROS são R: 2160 anagramas
a própria palavra ou qualquer outro
agrupamento que se obtém trocando a
ordem de suas letras como por exemplo:
ERMNUSO. Assim, o número de
e)
anagramas da palavra NUMEROS é o
Neste caso, existem quatro possibilidades
número total de permutações simples de para a última posição: M. N. R ou S. Para
sete letras distintas, isto é: cada consoante fixada na última posição,
𝑃7 = 7! = 5040 sobram seis letras para permutar.
R: 5040 anagramas
.
b)
Para encontrar a quantidade de anagramas
começados pela letra N, fixamos esta letra
e fazemos as permutações de todas as 𝑃6 4
demais.
N 𝑃6 ∙ 4 = 720 ∙ 4 = 2880

R: 2160 anagramas
𝑃6 = 6! = 720

R: 720 anagramas
CÁSSIO VIDIGAL 18 IFMG – CAMPUS OURO PRETO
f)
Agora, existem três possibilidades de
preenchimento da primeira posição, quatro
possibilidades para a última posição e 30) Determine o número de anagramas
sobram cinco letras para as outras cinco formados a partir de:
posições: a) LUA

3 𝑃5 4 b) GATO

3 ∙ 𝑃5 ∙ 4 = 3 ∙ 120 ∙ 4 = 1440

R: 1440 anagramas

c) ESCOLA

g)
Já que as letras N, U e M devem aparecer
juntas e nesta ordem, vamos “transformá-
las” num único bloco.
NUM E R O S
d) REPÚBLICA
Assim, podemos responder a questão
encontrando a quantidade de permutações
de 5 elementos: NUM, E, R, O e S.

𝑃5 = 5! = 120

R: 120 anagramas
e) FESTA

h)
Tomando como base o item anterior,
devemos considerar que para cada uma
das 120 permutação em que as letras N, U
e M aparecem juntas, existem 𝑃3 diferentes f) PERNAMBUCO
então fazemos:
𝑃5 ∙ 𝑃3 = 120 ∙ 6 = 720

R: 120 anagramas

MATEMÁTICA III 19 ANÁLISE COMBINATÓRIA, BINÔMIO DE NEWTON E PROBABILIDADES


31) Um dado foi lançado quatro vezes 33) Considere os anagramas formados a
sucessivamente e as Faces obtidas foram partir da palavra CONQUISTA:
2, 3, 5 e 6, não necessariamente nesta a) Quantos são?
ordem. De quantas formas distintas pode
ter ocorrido a sequência de resultados?

b) Quantos começam por vogal?

c) Quantos começam e terminam por


32) Calcule: consoante?
a) 𝑃5

b) 𝑃7

d) Quantos tem as letras CON juntas e


nessa ordem?

c) 𝑃3 + 𝑃2

e) Quantos apresentam a letra C antes da


letra A?
𝑃8
d)
𝑃10

CÁSSIO VIDIGAL 20 IFMG – CAMPUS OURO PRETO


34) Uma vez por ano, Dona Fátima, que 36) De quantos modos distintos seis
mora no Recife, visita parentes em Caruaru, homens e seis mulheres podem ser
João Pessoa, Petrolina, Maceió e colocados em fila indiana:
Garanhuns. a) em qualquer ordem?
a) De quantas formas distintas ela pode
escolher a sequência de cidades a visitar?

b) iniciando com homem e terminando com


mulher?

b) De quantos modos diferentes a ordem


das cidades pode ser definida se Dona
Fátima pretende encerrar as visitas em
Petrolina?

c) se os homens devem aparecer juntos e o


mesmo ocorrendo com as mulheres?

35) Uma estante tem dez livros distintos d) de modo que apareçam, do início para o
sendo cinco de álgebra, três de geometria e final da fila, dois homens, duas mulheres,
dois de trigonometria. De quantos modos três homens, três mulheres, um homem e
podemos arrumar esses livros na estante se uma mulher?
desejamos que os livros de mesmo assunto
permaneçam juntos?

MATEMÁTICA III 21 ANÁLISE COMBINATÓRIA, BINÔMIO DE NEWTON E PROBABILIDADES


37) Em quantos anagramas da palavra 40) Permutando se as letras T, R, A, P, O
QUEIJO as vogais não aparecem juntas? e S, são formados 720 anagramas. Esses
anagramas são colocados em ordem
alfabética. Qual a posição correspondente
a PRATOS.

38) Dona Lola tem três filhos: Pedro, Paulo


e Pérsio. Os três casaram-se e tem,
respectivamente, 1, 3 e 2 filhos. um
domingo e dona Lolla recebeu para o
almoço seus três filhos acompanhados das
respectivas esposas além de todos os
netos. Como recordação ela fotografou
todos os familiares, lado a lado, mas pediu
que cada filho aparecesse Junto de sua
família. De quantas formas distintas a foto
poderia ter sido feita?
41) Considerando os anagramas da palavra
BRASIL, responda:
a) quantos começam por B?

b) quantos começam por B e terminam por


L?
39) Resolva as seguintes equações
a) 𝑃𝑛 = 24

c) quantos começam por B ou terminam por


L?

𝑃𝑛
b) = 506
𝑃(𝑛−2)

d) quantos começam por K?

CÁSSIO VIDIGAL 22 IFMG – CAMPUS OURO PRETO


42) Formados e dispostos em ordem PERMUTAÇÃO COM ELEMENTOS
crescente todos os números que se obtém REPETIDOS
permutando se os algarismos 1, 2, 4, 6 e 8,
qual a posição ocupada pelo número 68
Em vários cálculos combinatórios,
412?
temos que calcular o número de
permutações de n elementos, nem todos
distintos.

Consideremos n o número de
elementos entre os quais o elemento a1
aparece n1 vezes, o elemento a2 aparece n2
vezes ... o elemento ak aparece nk vezes:
𝑛 𝑒𝑙𝑒𝑚𝑒𝑛𝑡𝑜𝑠

𝑎1 , 𝑎1 , . . . , 𝑎1 ⏟
⏟ 𝑎2 , 𝑎2 , . . . , 𝑎2 , . . . , ⏟
𝑎𝑘 , 𝑎𝑘 , . . . , 𝑎𝑘
𝑛1 𝑒𝑙𝑒𝑚𝑒𝑛𝑡𝑜𝑠 𝑛2 𝑒𝑙𝑒𝑚𝑒𝑛𝑡𝑜𝑠 𝑛𝑘 𝑒𝑙𝑒𝑚𝑒𝑛𝑡𝑜𝑠
𝑖𝑔𝑢𝑎𝑖𝑠 𝑎 𝑎1 𝑖𝑔𝑢𝑎𝑖𝑠 𝑎 𝑎2 𝑖𝑔𝑢𝑎𝑖𝑠 𝑎 𝑎𝑘

Sendo a1, a2, ... , ak distintos entre si


e n1 + n2 + ... nk = n.
O número de permutações desses n
elementos será dada por:
(𝑛1 ,𝑛2 ,…,𝑛𝑘) 𝑛!
𝑃𝑛 =
𝑛1 ! ∙ 𝑛2 ! ∙ … ∙ 𝑛𝑘 !

Para entender esse tipo de cálculo,


convém analisar os exemplos a seguir.

Ex.1: Quantos são os anagramas da


palavra CASA?

Resolução
Já estudamos que, se a palavra em
questão fosse formada por letras distintas,
faríamos 4! = 24 permutações. Para
encontrar a quantidade correta de
anagramas distintos da palavra CASA,
vamos, em princípio, considerar as letras A
como distintas utilizando cores para
diferenciá-las
CASA ACSA SCAA ACAS
CAAS ACAS SCAA ACSA
CSAA ASCA SACA AACS
______________________ CSAA ASAC SAAC AASC
ATIVIDADES COMPLEMENTARES CAAS AACS SACA ASCA
Pág. 166 – Exercícios 8 a 12 CASA AASC SAAC ASAC

MATEMÁTICA III 23 ANÁLISE COMBINATÓRIA, BINÔMIO DE NEWTON E PROBABILIDADES


Agora, vamos agrupar os anagramas Ex.3: Quantos são os anagramas da
iguais desconsiderando a diferenciação de palavra ITATIAIA?
cores:
CASA CASA SCAA SCAA Total de letras: 8
CAAS CAAS SACA SACA Repetições: I → 3 vezes
CSAA CSAA SAAC SAAC T → 2 vezes
A → 3 vezes
ASAC ASAC AACS AACS
ASCA ASCA AASC AASC
ITATIAIA
ACAS ACAS ACSA ACSA 8! 8 ∙ 7 ∙ 6 ∙ 5 ∙ 4 ∙ 3!
= = ⋯ = 560
3! ∙ 2! ∙ 3! 3 ∙ 2 ∙ 1 ∙ 2 ∙ 1 ∙ 3!
É possível observar que, para cada
anagrama que apresenta o A antes do A, Assim, concluímos que a palavra ITATIAIA
existe um correspondente com essas letras tem 560 anagramas.
em posição (de cores) inversa.

Neste caso, para encontrar a Ex.4: Quantos são os anagramas da


quantidade de anagramas da palavra palavra PANTANAL?
CASA, devemos calcular todos os
anagramas considerando as todas as letras Total de letras: 8
como distintas e, em seguida, “eliminar” as Repetições: A→ 3 vezes
repetições. Para tal, dividimos a quantidade N → 2 vezes
de permutações considerando todas as
letras distintas pelas permutações das PANTANAL
8! 8 ∙ 7 ∙ 6 ∙ 5 ∙ 4 ∙ 3!
letras que se repetem, ou, em linguagem = = ⋯ = 3360
matemática: 3! ∙ 2! 3! ∙ 2 ∙ 1

CASA
4! 4 ∙ 3 ∙ 2! Assim, concluímos que a palavra
= = 12 PANTANAL tem 3360 anagramas.
2! 2!

Assim, concluímos que a palavra CASA tem


12 anagramas. Ex.5: De quantas formas diferentes
podemos permutar os elementos de um
Ex.2: Quantos são os anagramas da conjunto formado por 3 letras A e 10 letras
palavra PALAVRA? B {A, A, A, B, B, B, B, B, B, B, B, B, B}?
Total de letras: 7
Repetições: A → 3 vezes Total de letras: 13
Repetições: A→ 3 vezes
PALAVRA B → 10 vezes
7! 7 ∙ 6 ∙ 5 ∙ 4 ∙ 3!
= = 7 ∙ 6 ∙ 5 ∙ 4 = 840 13! 13 ∙ 12 ∙ 11 ∙ 10!
3! 3! = = ⋯ = 286
3! ∙ 10! 3 ∙ 2 ∙ 1 ∙ 10!
Assim, concluímos que a palavra PALAVRA
tem 840 anagramas.

CÁSSIO VIDIGAL 24 IFMG – CAMPUS OURO PRETO


Ex.6: A figura abaixo representa quarteirões
de uma cidade (as partes em cinza são as
ruas) e dois pontos: A e B. 43) Quantos são os anagramas da palavra
MATEMATICA?

Andando sempre para norte ou para leste,


de quantas formas diferentes é possível
seguir saindo de A até chegar em B?
44) Em uma prova composta de 20
questões envolvendo V ou F, de quantas
Resolução: maneiras distintas teremos doze respostas
Para se deslocar de A para B, deve-se V e oito respostas F?
seguir 3 quarteirões para Norte e 5 para
Leste. As figuras abaixo mostram dois
possíveis caminhos:

Indicando por L os deslocamentos a Leste


e por N os deslocamentos ao Norte, o nosso 45) Quantos números de 6 algarismos
trabalho será permutar os elementos do podemos escrever utilizando os algarismos
conjunto {L, L, L, L, L, N, N, N}: 2, 2, 3, 3, 3 e 4?

Total de elementos: 8
Repetições: L→ 5 vezes
N → 3 vezes

8! 8 ∙ 7 ∙ 6 ∙ 5!
= = ⋯ = 56
3! ∙ 5! 3 ∙ 2 ∙ 1 ∙ 5!

Logo, Existem 56 maneiras


diferentes de ir de A até B nesta situação.

MATEMÁTICA III 25 ANÁLISE COMBINATÓRIA, BINÔMIO DE NEWTON E PROBABILIDADES


46) Dos números distintos que são 48) Pedro tem 9 bolinhas de gude, todas
formados com todos os algarismos do iguais, e deseja acondicioná-las em 4
número 333669, quantos desses são gavetas numeradas de 1 a 4. Sabendo que
ímpares? as gavetas podem conter até nove bolinhas
de gude cada uma, de quantas formas
diferentes Pedro pode guardar as 9
bolinhas de gude nas quatro gavetas?

47) Quantas são as soluções da equação: 49) Vô Pedro tem 10 moedas de R$1,00 e
x + y + z = 15, quer distribuí-las entre seus 4 netos. De
formadas, exclusivamente, por números quantas maneiras diferentes ele pode fazer
naturais: essa divisão?

CÁSSIO VIDIGAL 26 IFMG – CAMPUS OURO PRETO


50) Um casal de 5 filhos sendo 3 meninas e 52) Com duas letras iguais a A e n letras
2 meninos. De quantas formas diferentes iguais a B, podem ser formados 21
pode ter ocorrido a ordem de nascimentos anagramas. Qual o valor de n?
das crianças?

53) Sobre a malha


ao lado estão
marcados os
51) Sobre a palavra PIRATARIA: pontos A e B. De
a) Quantos são os seus anagramas? quantas formas
diferentes você
pode fazer uma linha contínua ligando o
ponto A ao ponto B seguindo
exclusivamente sobre as linhas da malha
para a esquerda e para baixo?

b) Quantos deles começam por A?

54) Em um plano cartesiano são marcados


dois pontos: A(37, -11) e B(46, -16). De
quantas formas diferentes você pode fazer
c) Quantos começam por vogal? uma linha contínua com traços horizontais
para a direita e verticais para baixo, sobre a
malha de unidades, de forma a ligar os dois
pontos?

MATEMÁTICA III 27 ANÁLISE COMBINATÓRIA, BINÔMIO DE NEWTON E PROBABILIDADES


55) Seguindo apenas para cima e para a COMBINAÇÕES
direita, de quantas formas diferentes é
possível ir de A até C passando por B? Em todos os casos estudados até
agora, a ordem dos elementos era relevante
para diferenciar dois agrupamentos, porém
existem situações em que a ordem com que
os elementos são tomados, não diferencia
dois agrupamentos. Para exemplificar,
tomemos dois casos: Numa primeira
situação, se tivermos que eleger dentre os
alunos da sua sala dois colegas para serem
representante e vice representante da
turma junto à pedagogia, a ordem de
escolha seria relevante. Porém, se tivermos
que escolher agora, dois colegas para irem
à pedagogia com a finalidade de tratar de
um assunto de interesse da turma, neste
caso não faria diferença a ordem com que
os colegas fossem escolhidos. Situações
como esta, em que a ordem de escolha não
diferencia dois agrupamentos, são
chamadas de combinação.

Ex.: Todos os dias, chegar do treino, Vítor


56) No mesmo esquema acima, de quantas faz uma vitamina de três frutas dentre seis
formas é possível ir de A até C sem passar disponíveis (Abacate, Banana, Maçã,
por B? Mamão, Morango e Amora). Quantos tipos
diferentes de vitamina ele pode fazer?

Resolução:
Sabemos que a ordem com que ele escolhe
as frutas não vai diferenciar o resultado final
da vitamina então calcularemos a
quantidade de formas diferentes de
selecionar as 3 frutas dentre as 6
disponíveis e, em seguida, eliminaremos as
repetições.

Para calcular a quantidade de formas


diferentes de selecionar as 3 frutas dentre
as 6 disponíveis, faremos 𝐴6,3 :

6! 6 ∙ 5 ∙ 4 ∙ 3!
______________________ 𝐴6,3 = = = 120
ATIVIDADES COMPLEMENTARES 3! 3!
Pág. 169 – Exercícios 13 a 16

CÁSSIO VIDIGAL 28 IFMG – CAMPUS OURO PRETO


Uma das 120 formas de escolher essas três
frutas é, por exemplo, Abacate, Mamão e
Morango mas já sabemos que se forem Ex.1: Dentre os 26 alunos de uma turma,
escolhidas nesta ordem ou em outra, o pretende-se formar uma comissão de 3
resultado da vitamina será o mesmo. Então estudantes para representar a turma numa
vamos contar todas as permutações reunião junto ao setor pedagógico da
possíveis em grupos de três frutas e escola. De quantas formas esta comissão
eliminar essas repetições. pode ser formada?

Já vimos que para calcular a quantidade de Resolução:


permutações entre as três frutas, devemos
fazer 𝑃3 e: n!
Cn,p =
𝑃3 = 3! = 6 (n − p)! p!

Agora, dividindo 120 (𝐴6,3 ) por 6 (𝑃3 ) 26! 26 ∙ 25 ∙ 24 ∙ 23!


C26,3 = = =
encontramos a quantidade procurada: (26 − 3)! 3! 23! ∙ 3 ∙ 2 ∙ 1

𝐴6,3 120 26 ∙ 25 ∙ 24
𝐶6,3 = = = 20 = = 26 ∙ 25 ∙ 4 = 2600
𝑃3 6 3∙2

Assim, Vitor pode fazer 20 tipos de R: A comissão ode ser formada de 2600
vitaminas diferentes. manieras diferentes.

Fórmula da Combinação Ex.2: Em uma academia trabalham 7


professores de musculação e 10 de
Como acabamos de ver no exemplo ginástica aeróbica. Quantas equipes de 2
acima, para calcular a quantidade de professores de musculação e 2 de ginástica
combinações de n elementos tomados p a aeróbica podem ser formadas?
p, fazemos
An,p n! 1 n! Resolução:
Cn,p = = ∙ = Primeira vamos escolher os professores de
Pp (n − p)! p! (n − p)! p!
musculação:
Essa é a “fórmula da combinação” 7! 7 ∙ 6 ∙ 5!
C7,2 = = = 21
(7 − 2)! 2! 5! ∙ 2 ∙ 1
𝒏! Agora vamos escolher os professores de
𝑪𝒏,𝒑 = ginástica aeróbica:
(𝒏 − 𝒑)! 𝒑! 10! 10 ∙ 9 ∙ 8!
C7,2 = = = 45
(Lemos: Combinação de n elementos tomados p a (10 − 2)! 2! 8! ∙ 2 ∙ 1
p é igual a n fatorial sobre n menos p fatorial vezes
p fatorial) A cada uma das 21 equipes de professores
de musculação, podemos associar uma das
Observação: É comum encontrarmos a 45 equipes de professores de Ginástica
𝑛
representação (𝑝) para Cn,p . Aeróbica. Então o resultado procurado é:
21 ∙ 45 = 945

R: 945 equipes

MATEMÁTICA III 29 ANÁLISE COMBINATÓRIA, BINÔMIO DE NEWTON E PROBABILIDADES


Ex.3: Sobre uma circunferência marcam-se
oito pontos distintos. Quantos triângulos
podem ser construídos com vértices em três 57) De quantos modos distintos Eduardo
desses pontos? pode escolher quatro entre as nove
Resolução: camisetas regata que possui para levar em
Observe a figura ao uma viagem?
lado. Nela estão
destacados oitos
pontos e um
triângulo. Podemos
notar que a ordem
com que os pontos
tomados, desde
que tomemos os mesmos pontos, não
diferencia dois triângulos. Assim, para
encontrar a quantidade de triângulos
possíveis de serem formados, devemos
fazer 𝐶8,3.
8! 8 ∙ 7 ∙ 6 ∙ 5! 58) Um curso de idiomas oferece turmas
𝐶8,3 = = = 56 para iniciantes em inglês, espanhol,
(8 − 3)! 3! 5! ∙ 3 ∙ 2 ∙ 1
alemão, italiano e japonês.
R: Podemos formar 56 triângulos diferentes. a) De quantas formas distintas um
estudante pode matricular-se em três
desses cursos?
Ex.4: Uma locadora de automóveis têm à
disposição de seus clientes uma frota de 16
carros nacionais e quatro carros
importados, todos distintos. De quantas
formas uma empresa poderá alugar três
carros de modo que pelo menos um carro
nacional seja escolhido?

Resolução: b) De quantas formas distintas ele poderá


A forma de a empresa alugar 3 carros matricular-se em três desses cursos,
quaisquer é 𝐶20,3 incluindo Obrigatoriamente o de inglês?
20!
𝐶20,3 = = 1140
17! ∙ 3!
A forma de a empresa alugar apenas carros
importados 𝐶4,3
4!
𝐶4,3 = =4
1! ∙ 3!

Assim, fazendo 1140 − 4 temos a


quantidade de formas de alugar ao menos
um carro nacional.

R: São 1136 formas de alugar ao menos um


carro nacional.
CÁSSIO VIDIGAL 30 IFMG – CAMPUS OURO PRETO
59) Calcule 60) Sobre uma circunferência marcam-se
a) 𝐶11,3 dez pontos.
a) Qual o número de segmentos de reta que
podemos traçar com extremidades em dois
desses pontos?

b) 𝐶9,6

c) 𝐶6,3 b) Quantos triângulos podemos construir


com vértices em três desses pontos?

d) 𝐶17,7 − 𝐶17,10

61) Uma junta médica deverá ser formada


por Quatro médicos e dois enfermeiros. de
quantas maneiras ela poderá ser formada
se estão disponíveis 10 médicos e 6
enfermeiros?

e) 𝐶5,3 + 𝐶5,4 + 𝐶5,3

MATEMÁTICA III 31 ANÁLISE COMBINATÓRIA, BINÔMIO DE NEWTON E PROBABILIDADES


Para as questões 62, 63 e 64, considere um b) Duas cartas de copas.
baralho comum que possui 52 cartas sendo
13 de cada naipe (ouros, paus, espadas e
copas) e cada naipe contendo 13 cartas (ás,
2, 3, …, 10, Valete, Dama e Rei)

62) Sorteando se simultaneamente quatro


cartas, determine:
a) o número de maneiras distintas de
ocorrer o resultado do sorteio.
c) Uma carta de copas e outra de ouros.

b) De quantas formas distintas é possível 64) De quantas maneiras distintas poderão


escolher as quatro cartas de copas. ser sorteadas simultaneamente cinco cartas
de um baralho de modo que o resultado do
sorteio contenha:
a) três cartas de paus e duas de espadas?

b) o rei de ouros?
63) Duas cartas são sorteadas de uma só
vez de um baralho comum. Determine o
número de maneiras possíveis de ocorrer
um resultado formado por:
a) um rei e uma dama.

c) exatamente dois valetes?

CÁSSIO VIDIGAL 32 IFMG – CAMPUS OURO PRETO


65) Para montar uma cesta de café da 67) Marcam-se cinco pontos distintos sobre
manhã estão disponíveis os seguintes itens: uma reta r. Sobre outra reta s, paralela a r,
quatro tipos de pães, três tipos de queijo, marcam-se mais quatro pontos distintos.
três tipos de frutas, cinco sabores de geleia Quantos triângulos podem ser formados
e quatro sabores de tortas doces. De com vértices em três quaisquer desses
quantos modos distintos a cesta poderá ser pontos?
montada se um cliente pedir 2 tipos de
pães, 1 tipo de queijo, 2 frutas, 2 sabores
de geleia é 1 torta doce?

66) Resolva as seguintes equações:


a) 𝐶𝑛,2 = 136
68) O vencedor de um concurso de redação
de um colégio poderá, como prêmio,
escolher cinco livros, entre 10 de Machado
de Assis, 7 de Érico Veríssimo e 5 de
Clarice Lispector. De quantos modos
distintos o vencedor poderá fazer a escolha
de modo que:
a) sejam selecionados 2 de Machado de
Assis, 2 de Érico Veríssimo e 1 de Clarice
Lispector?

b) 𝐶𝑛,2 + 𝐶𝑛+1,𝑛−1 = 25

b) nenhum livro escolhido seja de Machado


de Assis?

MATEMÁTICA III 33 ANÁLISE COMBINATÓRIA, BINÔMIO DE NEWTON E PROBABILIDADES


c) pelo menos quatro livros de Clarice b) Se o casal pretende se conhecer
Lispector sejam escolhidos? obrigatoriamente Salvador, de quantos
modos distintos poderia ser feita a escolha?

c) Se, por motivos logísticos, Fortaleza só


pudesse ser visitada se São Luís também o
69) Em uma reunião havia 50 pessoas. fosse e vice-versa, determine de quantas
Cada uma cumprimentou as outras com um maneiras a escolha poderá ser feita?
aperto de mão. Quantas saudações foram
dadas nessa reunião?

70) Um casal decidiu que a viagem de lua


de mel seria feita pelo nordeste visitando
exatamente 3 das nove capitais.
a) De quantos modos distintos poderiam ser
escolhidas as três capitais sem levar em
consideração a ordem da visita?

CÁSSIO VIDIGAL 34 IFMG – CAMPUS OURO PRETO


72) Um Jornalista foi designado para cobrir 74) Ainda segundo site da CEF, “A aposta
uma reunião de ministros de estado. Ao mínima, de 6 números, custa R$ 3,50.
chegar ao local da reunião, descobriu que Quanto mais números marcar, maior o
havia terminado. Ao perguntar ao porteiro preço da aposta e maiores as chances de
o número de ministros presentes, ele disse: faturar o prêmio mais cobiçado do país”
“Ao saírem, todos os ministros se (http://www.loterias.caixa.gov.br/wps/portal/loterias/landin
cumprimentaram mutuamente, no total de g/megasena acesso em: 23/05/16). Quanto uma
15 apertos de mão”. Com base nessa pessoa gastaria para apostar em todas as
informação, qual foi o número de ministros combinações possíveis? Pesquise sobre o
presentes ao encontro? maior prêmio histórico pago pela Mega-
sena e compare a resposta desta questão.

75) Como visto no texto, “[...] você deve


marcar de 6 a 15 números do volante de
apostas.” Marcando-se 6 números, você
concorre com uma única combinação de 6
prognósticos (isso é óbvio, não?). Com
73) Segundo consta no site da CEF, “A quantas combinações de 6 prognósticos
Mega-Sena paga milhões para o acertador você concorrer se marcar:
dos 6 números sorteados. Ainda é possível a) 7 números? R: _______
ganhar prêmios ao acertar 4 ou 5 números b) 8 números? R: _______
dentre os 60 disponíveis no volante de c) 9 números? R: _______
apostas. Para realizar o sonho de ser o d) 10 números? R: _______
próximo milionário, você deve marcar de 6 e) 11 números? R: _______
a 15 números do volante, podendo deixar f) 12 números? R: _______
que o sistema escolha os números para g) 13 números? R: _______
você (Surpresinha) e/ou concorrer com a h) 14 números? R: _______
mesma aposta por 2, 4 ou 8 concursos i) 15 números? R: _______
consecutivos (Teimosinha)” (http://www.loterias.
caixa.gov.br/wps/portal/loterias/landing/megasena acesso
em: 23/05/16).
O volante da Mega-sena é composto de 60
números sendo que destes, 6 são
sorteados. Quantas são as combinações
possíveis de números?

MATEMÁTICA III 35 ANÁLISE COMBINATÓRIA, BINÔMIO DE NEWTON E PROBABILIDADES


BINÔMIO DE NEWTON
Veja os exemplos abaixo:
Você se lembra, lá do Ensino
fundamental, de como se desenvolve
(𝑎 + 𝑏)2 ?
6 6! 6∙5∙4!
Ex.1: ( ) = 𝐶6,4 = (6−4)!∙4! = 2∙1∙4! = 15
4
Em binômio de Newton, estudamos o
desenvolvimento desta potência e de todas 10 10! 10∙9∙8∙7!
as outras do tipo (𝑎 + 𝑏)𝑛 para valores de Ex.2: ( ) = 𝐶10,7 = 3!∙7! = 3∙2∙1∙7! = 120
7
𝑛 ∈ ℕ.
_______________________
Veja os exemplos:

Casos particulares
0
(𝑎 + 𝑏) = 1
(𝑎 + 𝑏)1 = 𝑎 + 𝑏 1º Caso: 𝑝 = 0
(𝑎 + 𝑏)2 = 𝑎2 + 2𝑎𝑏 + 𝑏 2
(𝑎 + 𝑏)3 = 𝑎3 + 3𝑎2 𝑏 + 3𝑎𝑏 2 + 𝑏 3 𝑛
( )=1 ∀𝑛 ∈ ℕ
(𝑎 + 𝑏)4 = 𝑎4 + 4𝑎3 𝑏 + 6𝑎2 𝑏 2 + 4𝑎𝑏 3 + 𝑏 4 0

2º Caso: 𝑝 = 1
É possível notar que à medida que o 𝑛
expoente aumenta, o desenvolvimento de ( )=𝑛 ∀𝑛 ∈ ℕ
0
(𝑎 + 𝑏)𝑛 torna-se mais complexo e as
contas ficam mais trabalhosas. No entanto, 3º Caso: 𝑝 = 𝑛
por meio de técnicas de contagem e
aplicações de propriedades dos binômios, é 𝑛
( )=1 ∀𝑛 ∈ ℕ
possível obter o desenvolvimento de 𝑛
(𝑎 + 𝑏)𝑛 de forma rápida e eficiente.
Os coeficientes binomiais tem
Coeficientes Binomiais aplicação no estudo do desenvolvimento de
(𝑎 + 𝑏)𝑛 como veremos mais a frente.
Dados dois números 𝑛 e 𝑝, com
𝑛 ≥ 𝑝, definimos como coeficiente
𝑛 Binomiais complementares
binomial de n sobre p, e indicamos por (𝑝)
o número: Dizemos que dois coeficientes binomiais de
mesmo numerador são complementares
𝒏 𝒏! quando a soma de seus denominadores é
(𝒑) = 𝑪𝒏,𝒑 = igual ao numerador, isto é:
(𝒏 − 𝒑)! 𝒑! 𝑛 𝑛
(𝑝) e (𝑞 ) são complementares
se 𝑝 + 𝑞 = 𝑛

O número 𝑛 é dito numerador e o


número 𝑝 é chamado de denominador de
𝑛
(𝑝).
CÁSSIO VIDIGAL 36 IFMG – CAMPUS OURO PRETO
8 8
( ) e ( ) são complementares 76) Calcule:
6 2 4
9 9 a) ( )
( ) 𝑒 ( ) são complementares 2
5 4
6 6
( ) 𝑒 ( ) são complementares
3 3

Propriedade dos binomiais


complementares 9
b) ( )
6
Dois binomiais complementares são
iguais

𝑛 𝑛
Para 𝑝 + 𝑞 = 𝑛 tem-se (𝑝) = (𝑞 )

2 3
c) ( ) + ( )
É possível demonstrar a assertiva 0 3
acima, observe:

Se 𝑝 + 𝑞 = 𝑛 então 𝑝 = 𝑛 − 𝑞

Assim:
6 8
𝑛 𝑛 𝑛! ( )+( )
3 4
(𝑝) = (𝑛 − 𝑞 ) = = d) 5
[𝑛 − (𝑛 − 𝑞)]! ∙ (𝑛 − 𝑞)! ( )
2

𝑛! 𝑛! 𝑛
= = = (𝑞 )
(𝑛 − 𝑛 + 𝑞)! ∙ (𝑛 − 𝑞)! 𝑞! ∙ (𝑛 − 𝑞)!

Daí, podemos concluir que:

𝑛 𝑛
(𝑝) = (𝑞 ) ⇔ (𝑝 = 𝑞 𝑜𝑢 𝑝 + 𝑞 = 𝑛)

Sendo 𝑝, 𝑛 e 𝑞 números naturais e 𝑛 ≥ 𝑝 e


𝑛 ≥ 𝑞.

𝑝 𝑝 𝑝
e) ( ) + ( ) + (𝑝) sendo 𝑝 ∈ ℕ e 𝑝 ≥ 1
7 7 0 1
Ex.1: Determine x de modo que ( ) = ( ).
𝑥 4
Resolução: Pela propriedade acima,
𝑥 = 4 ou 𝑥 + 4 = 7 → 𝑥 = 3

R: S = {3, 4 }
MATEMÁTICA III 37 ANÁLISE COMBINATÓRIA, BINÔMIO DE NEWTON E PROBABILIDADES
77) Resolva as seguintes equações: Triangulo de Pascal2
13 13
a) ( ) = ( )
𝑥 8 Os coeficientes binomiais podem ser
dispostos em uma tabela chamada de
Triângulo de Pascal. Nela, os coeficientes
de mesmo numerador agrupam-se em uma
mesma linha e os coeficientes de mesmo
denominador, agrupam-se em uma mesma
coluna.

0
L0 ( )
0
1 1
L1 ( ) ( )
21 21 0 1
b) ( )=( ) 2 2 2
𝑥+5 −2𝑥 + 17 L2 ( ) ( ) ( )
0 1 2
3 3 3 3
L3 ( ) ( ) ( ) ( )
0 1 2 3
4 4 4 4 4
L4 ( ) ( ) ( ) ( ) ( )
0 1 2 3 4
⋮ ⋮ ⋮ ⋮ ⋮ ⋱
𝑘 𝑘 𝑘 𝑘 𝑘 𝑘
Lk ( ) ( ) ( ) ( ) ( ) ⋯ ( )
0 1 2 3 4 𝑘

Notemos que L0 significa “linha do


numerador 0, L1 é a linha do numerador 1 e
assim até linha k que é a linha de
numerador k.

Calculando os valores dos


11
c) ( 2 ) = (
11
) coeficientes (pela fórmula da página 36
𝑥 2𝑥 + 3 desta apostila), obtemos outra
representação do triângulo:

1
1 1
1 2 1
1 3 3 1
1 4 6 4 1
1 5 10 10 5 1
⋮ ⋮ ⋮ ⋮ ⋮ ⋮ ⋱

Propriedades do Triângulo de Pascal


P1: Toda linha começa e termina com 1
𝑘 𝑘
De fato, ( ) = 1 e ( ) = 1
0 𝑘

2 Também conhecido como Triângulo de Tartaglia


CÁSSIO VIDIGAL 38 IFMG – CAMPUS OURO PRETO
P2: Em uma mesma linha, os coeficientes Esta propriedade é conhecida como
binomiais equidistantes dos extremos são Relação de Stifel e pode ser generalizada
iguais. como:

Por exemplo: 𝑛 𝑛−1 𝑛−1


Linha do 5 (𝑝 ) = ( )+( ), 𝑛 ≥ 𝑝
𝑝 𝑝−1
5 5 5 5 5 5
( ) ( ) ( ) ( ) ( ) ( )
0 1 2 3 4 5
∥ ∥ ∥ ∥ ∥ ∥ Observe que o primeiro membro da
1 5 10 10 5 1 igualdade apresenta um elemento genérico
(da linha 𝑛 e coluna 𝑝) do triângulo. Já no
segundo membro, representa a soma de
elementos da linha imediatamente acima
Linha do 6 (linha 𝑛 − 1) e uma mesma coluna (𝑝) e a
6 6 6 6 6 6 6 coluna da esquerda (𝑝 − 1).
( ) ( ) ( ) ( ) ( ) ( ) ( )
0 1 2 3 4 5 6
∥ ∥ ∥ ∥ ∥ ∥ ∥ P4: A soma dos elementos da linha de
1 6 15 20 15 6 1 denominador k é igual a 2𝑘 .

Soma
1 1
1 1 2
Note que os binomiais indicados (e 1 2 1 4
também os extremos) são binomiais 1 3 3 1 8
complementares e, como vimos na página 1 4 6 4 1 16
37, são iguais. 1 5 10 10 5 1 32
1 6 15 20 15 6 1 64
P3: Cada elemento (com exceção daqueles ⋮ ⋮ ⋮ ⋮ ⋮ ⋮ ⋮ ⋱ ⋮
que estão na primeira e na última coluna de
cada linha) é igual à soma de dois
elementos da linha anterior, mais
Símbolo de Somatório – 
especificamente, do termo imediatamente
acima e do termo a esquerda deste. Um símbolo muito importante na
álgebra é o símbolo do somatório indicado
Observe na tabela: pela forma maiúscula da letra grega Sigma
1
–  . Este símbolo representa a soma de um
1 1
certo número de parcelas com alguma
1 2 1
característica em comum.
1 3 3 1
1 4 6 4 1
1 5 10 10 5 1
1 6 15 20 15 6 1 1
Ex.1: Vamos calcular
⋮ ⋮ ⋮ ⋮ ⋮ ⋮ ⋮ ⋱ 5

∑(𝑘 − 1)2
Note que: 1 + 1 = 2, 3 + 3 = 6 e 4 + 1 = 5.
𝑘=0

Verifique a propriedade em outros (Lemos: Somatório de k menos 1 elevado


conjuntos de coeficientes binomiais. ao quadrado com k variando de 0 até 5)

MATEMÁTICA III 39 ANÁLISE COMBINATÓRIA, BINÔMIO DE NEWTON E PROBABILIDADES


Resolução: 3
2
5 b) ∑
𝑖+1
∑(𝑘 − 1)2 𝑖=0
𝑘=0
A expressão nos indica que devemos somar
sucessivas parcelas de termos do tipo
(𝑘 − 1)2 . Em cada parcela, o termo 𝑘 vai
variar de forma que na primeira parcela
teremos 𝑘 = 0, na segunda 𝑘 = 1 e assim
até chegarmos em 𝑘 = 5. Assim, podemos
escrever:
5

∑(𝑘 − 1)2 = (0 − 1)2 + (1 − 1)2 +


𝑘=0
+(2 − 1)2 + (3 − 1)2 + (4 − 1)2 + (5 − 1)2 =
5
2 2 2 2 2 2
= (−1) + 0 + 1 + 2 + 3 + 4 = c) ∑ 𝑥!
= 1 + 0 + 1 + 4 + 9 + 16 = 31 𝑥=0

Então,
5

∑(𝑘 − 1)2 = 31
𝑘=0

Ex.2: Calcular
2
5
∑( )
𝑘
𝑘=0
Resolução: 4
4
d) ∑( )
4 𝑘
5 5 5 5 𝑘=0
∑ ( ) = ( ) + ( ) + ( ) = 10 + 10 + 5 = 25
𝑘 2 3 4
𝑘=2

R: 25

78) Calcule as seguintes somas:


4

a) ∑(3𝑖 + 1)
𝑖=1

CÁSSIO VIDIGAL 40 IFMG – CAMPUS OURO PRETO


79) Qual o valor do somatório abaixo? Desenvolvimento do Binômio
10
1 1
∑( − ) Observemos o desenvolvimento de
𝑛 𝑛+1 (𝑎 + 𝑏)𝑛 para alguns valores de 𝑛
𝑛=1
apresentados na página 36.

(𝑎 + 𝑏)2 = 𝑎2 + 2𝑎𝑏 + 𝑏 2
 São 3 termos
 Os expoentes de 𝑎 decrescem de 2
até 0
 Os expoentes de 𝑏 crescem de 0 até 2
 Os coeficientes (1, 2, 1) são a linha do
triângulo de Pascal relativa ao
numerador 2

(𝑎 + 𝑏)3 = 𝑎3 + 3𝑎2 𝑏 + 3𝑎𝑏 2 + 𝑏 3


 São 4 termos
 Os expoentes de 𝑎 decrescem de 3
até 0
 Os expoentes de 𝑏 crescem de 0 até 3
 Os coeficientes (1, 3, 3, 1) são a linha
do Triângulo de Pascal relativa ao
numerador 3
80) Determine o valor de 𝑎 na expressão
4

∑(2𝑖 + 𝑎) = 60 (𝑎 + 𝑏)4 = 𝑎4 + 4𝑎3 𝑏 + 6𝑎2 𝑏 2 + 4𝑎𝑏 3 + 𝑏 4


𝑖=1  São 5 termos
 Os expoentes de 𝑎 decrescem de 4
até 0
 Os expoentes de 𝑏 crescem de 0 até 4
 Os coeficientes (1, 4, 6, 4, 1) são a
linha do Triângulo de Pascal relativa
ao numerador 4

Generalizando, podemos afirmar sobre o


desenvolvimento de (𝑎 + 𝑏)𝑛 :
 São n+1 termos
 Os expoentes de 𝑎 decrescem de 𝑛
até 0
 Os expoentes de 𝑏 crescem de 0 até 𝑛
 Os coeficientes são a linha do
Triângulo de Pascal relativa ao
numerador 𝑛.

MATEMÁTICA III 41 ANÁLISE COMBINATÓRIA, BINÔMIO DE NEWTON E PROBABILIDADES


Teorema de Newton
Dando linguagem matemática à generalização anterior, podemos escrever que:
𝑛 𝑛 𝑛 𝑛 𝑛
(𝑎 + 𝑏)𝑛 = ( ) 𝑎𝑛 𝑏 0 + ( ) 𝑎𝑛−1 𝑏 𝑎 + ( ) 𝑎𝑛−2 𝑏 2 + ⋯ + ( ) 𝑎1 𝑏 𝑛−1 + ( ) 𝑎0 𝑏 𝑛
0 1 2 𝑛−1 𝑛

Em que 𝑎 e 𝑏 são números reais e 𝑛 é número natural.

Utilizando o símbolo do somatório, podemos reduzir a expressão acima por:


𝑛
𝑛
(𝑎 + 𝑏)𝑛 = ∑ ( ) 𝑎𝑛−𝑘 𝑏 𝑘
𝑘
𝑘=0
(Lemos: a mais b elevado a n é igual ao somatório do binomial de n sobre k vezes a elevado
a n menos k vezes b elevado a k com k variando de zero até n)

Ex.1: Desenvolver (𝑎2 + 3)4 usando o teorema de Newton.


Resolução:
4

(𝑎2 + 3)4 = ∑ (4) (𝑎2 )4−𝑘 ∙ 3𝑘 =


𝑘
𝑘=0
4 4 4 4 4
= ( ) (𝑎2 )4−0 ∙ 30 + ( ) (𝑎2 )4−1 ∙ 31 + ( ) (𝑎2 )4−2 ∙ 32 + ( ) (𝑎2 )4−3 ∙ 33 + ( ) (𝑎2 )4−4 ∙ 34 =
0 1 2 3 4

= 1 ∙ (𝑎2 )4 ∙ 1 + 4 ∙ (𝑎2 )3 ∙ 3 + 6 ∙ (𝑎2 )2 ∙ 9 + 4 ∙ (𝑎2 )1 ∙ 27 + 1 ∙ (𝑎2 )0 ∙ 81 =

= 𝑎8 + 12𝑎6 + 54𝑎4 + 108𝑎2 ∙ 27 + 81

Ex.2: Desenvolver (2𝑥 − 3)5


Resolução:
5

(2𝑥 − 3)3 = ∑ (5) (2𝑥)5−𝑘 ∙ (−3)5 =


𝑘
𝑘=0

5 5 5 5 5 5
= ( ) (2𝑥)5 ∙ 1 − ( ) (2𝑥)4 ∙ 31 + ( ) (2𝑥)5 ∙ 32 − ( ) (2𝑥)2 ∙ 33 + ( ) 2𝑥 ∙ 34 − ( ) ∙ 1 ∙ 35 =
0 1 2 3 4 5

= 1 ∙ (2𝑥)5 ∙ 1 − 5 ∙ (2𝑥)4 ∙ 31 + 10 ∙ (2𝑥)5 ∙ 32 − 10 ∙ (2𝑥)2 ∙ 33 + 5 ∙ 2𝑥 ∙ 34 − 1 ∙ 1 ∙ 35 =

= 32𝑥 5 − 240𝑥 4 + 720𝑥 3 − 1080𝑥 2 + 810𝑥 − 243

CÁSSIO VIDIGAL 42 IFMG – CAMPUS OURO PRETO


81) Desenvolver:
a) (1 + x 2 )3

b) (𝑥 − 3𝑦)5

1 4
c) (𝑥 − 𝑥)

MATEMÁTICA III 43 ANÁLISE COMBINATÓRIA, BINÔMIO DE NEWTON E PROBABILIDADES


Termo Geral do Binômio Substituindo 𝑘 = 8 no termo geral
Muitas vezes, podemos estar encontrado:
interessados em encontrar apenas um 16 3𝑘
16
( ) ∙ 𝑥16− 2 = ( ) ∙ 𝑥 4 = 12870𝑥 4
termo específico do desenvolvimento do 𝑘 8
Binômio de Newton sem precisar escrever
todos os termos. Para isto, é relevante que R: 12 870𝑥 4
encontremos uma expressão que possa
representar qualquer o do desenvolvimento
de (𝑎 + 𝑏)𝑛 e, a partir dela, determinemos o Ex.2: Qual o termo independente de x no
10
termo procurado. 2
desenvolvimento de (𝑥 2 − 𝑥 3 ) ?
Já vimos, na página 42 desta Resolução:
apostila, que 10 2 10−𝑘
( ) ∙ ( 2) ∙ (−𝑥 3 )𝑘 =
𝑘 𝑥
𝑛 𝑛 10 (−1)𝑘 2
10−𝑘
(𝑎 + 𝑏)𝑛 = ( ) 𝑎𝑛 𝑏 0 + ⋯ + ( ) 𝑎0 𝑏 𝑛 =( )∙ ∙ 20−2𝑘 ∙ 𝑥 3𝑘 =
0 𝑛 𝑘 𝑥
10 (−1)𝑘 10−𝑘 5𝑘−20
𝑛 =( )∙ ∙2 ∙𝑥
O termo 𝑇𝑘+1 = ( ) 𝑎𝑛−𝑘 𝑏 𝑘 é 𝑘
𝑘
chamado de termo geral do binômio pois
Para que o termo seja independente de x,
para valores de 𝑘 (𝑘 = 1, 2, 3, 4 . . . , 𝑛)
então o expoente de x deve ser igual a
obtemos todos os termos do
zero, assim:
desenvolvimento. O índice 𝑘 + 1 determina
5𝑘 − 20 = 0 → 𝑘 = 4
a posição do termo quando ordenado em
termos de ordem decrescente dos
Substituindo k no termo encontrado:
expoentes de 𝑎. 10
( ) ∙ (−1)4 ∙ 210−4 ∙ 𝑥 5∙4−20 = ⋯ = 13440
4

R: 13 440
Ex.1: Qual termo do desenvolvimento de
1 16
(𝑥 + )
√𝑥

apresenta o x com expoente 4?

Resolução:
16 16−𝑘
1 𝑘 16 𝑥
16−𝑘
( )∙𝑥 ∙( ) =( )∙ 𝑘 =
𝑘 √𝑥 𝑘
𝑥2
3𝑘
16 16−
2 para 𝑘 = 0, 1, 2, . . . , 16.
=( )∙𝑥
𝑘

O expoente de x no desenvolvimento
3𝑘
(16 − 2 ) deve ser igual a 4 (pelo enunciado)
então:

3𝑘
16 − =4→𝑘=8
2
CÁSSIO VIDIGAL 44 IFMG – CAMPUS OURO PRETO
84) Determine, no desenvolvimento de
1 11
82) No desenvolvimento de (𝑥 2 − )
12
𝑥
𝑥2
( + 𝑥) a) O termo independente de x
3

determine o coeficiente de :
a) 𝑥12

b) 𝑥18
b) o termo que contém 𝑥13

83) no desenvolvimento de (𝑥 2 − 1)10,


determine:
a) o terceiro termo

c) o termo central

______________________
ATIVIDADES COMPLEMENTARES
Pág. 175 – Exercícios 27 e 28
MATEMÁTICA III 45 ANÁLISE COMBINATÓRIA, BINÔMIO DE NEWTON E PROBABILIDADES
PROBABILIDADES Evento

Na página 35 desta apostila Qualquer subconjunto de  é um


calculamos q quantidade total de evento de . Veja os exemplos a seguir:
combinações possíveis em um jogo de
Mega-sena e também a quantidade de
jogos de 6 prognósticos quando se marca 6,
7, 8, ..., 15 números num único cartão. Ex.: Os alunos de uma turma de formandos
resolveram rifar uma cesta de café da
Agora, em probabilidades, manhã. A Rifa constava de 90 bilhetes
utilizaremos estes resultados para, por numerados de 1 a 90.
exemplo, calcularmos as chances de O Resultado da rifa é um
ganhar na Mega-sena ou de outros eventos, experimento aleatório cujo espaço amostral
em outras situações, acontecerem. é  = {1, 2, 3, 4, ..., 89, 90}.
Se um professor resolver comprar
Experimento aleatório todos os bilhetes cujos números são
Quando lançamos um dado, quando múltiplos de 9, o conjunto de resultados
escolhemos ao acaso uma carta de baralho favoráveis ao professor é P = {9, 18, 27, 36,
ou quando retiramos uma bola numerada 45, 54, 63, 72, 81, 90}. Se a mãe de um
de uma urna, não podemos determinar de aluno resolve comprar os bilhetes formados
antemão qual objeto será sorteado. por dois algarismos iguais, o conjunto de
Experimentos como estes são chamados resultados favoráveis a ela é M = {11, 22,
de experimento aleatório pois, repetidos 33, 44, 55, 66, 77, 88}
em condições idênticas, podem apresentar Observe que os conjuntos P e M são
resultados diferentes. Tal variabilidade subconjuntos de . então P e M são
deve-se ao acaso. eventos de . A partir de agora,
denominaremos o conjunto evento por E.
Espaço amostral _________________________
Consideremos um evento aleatório.
O conjunto de todos os resultados possíveis Observações:
deste experimento é chamado de espaço 1) Se W = E então o evento é chamado de
amostral e é indicado pela letra grega evento certo.
ômega – . 2) Quando E = Ø, o evento é chamado de
evento impossível.

Ex.1: Ao lançar uma moeda e observar a


face voltada pra cima, podemos encontrar: “Tirar” um número menor ou igual a 6 no
lançamento de um dado é um evento certo.
 = {cara, coroa} Tirar um número maior que 6 no
Assim, n() = 2 lançamento de um dado, é um evento
impossível.
Ex.2: Ao lançar um dado e observar a face
voltada pra cima, podemos encontrar: Evento Complementar

 = {1, 2, 3, 4, 5, 6} Consideremos um evento E relativo


Assim, n() = 7 a um espaço amostral . Chamamos de
evento complementar de E (indicamos por
CÁSSIO VIDIGAL 46 IFMG – CAMPUS OURO PRETO
EC) ao evento que ocorre quando E não Dado um espaço amostral equiprovável Ω
ocorre. Observe o diagrama abaixo: finito e não vazio, e 𝐸 um evento, a
probabilidade de ocorrer algum elemento de
𝐸, indicado por 𝑝(𝐸) é

𝑛(𝐸)
𝑝(𝐸) = 𝑛(Ω)

ou seja:
Pelo digrama, note que:
𝑛ú𝑚𝑒𝑟𝑜 𝑑𝑒 𝑐𝑎𝑠𝑜𝑠 "𝑓𝑎𝑣𝑜𝑟á𝑣𝑒𝑖𝑠"
𝑐 𝑐 𝑝(𝐸) =
𝐸∩𝐸 =∅e𝐸∪𝐸 =Ω 𝑛ú𝑚𝑒𝑟𝑜 𝑑𝑒 𝑐𝑎𝑠𝑜𝑠 "𝑝𝑜𝑠𝑠í𝑣𝑒𝑖𝑠"

Ex.1:
Numa urna são colocadas 10 bolas No lançamento de uma moeda, qual a
numeradas de 1 a 10. Seja E o evento “ser probabilidade de se obter a face cara?
sorteada uma bola com número múltiplo de Resolução:
3”, Determinar Ec. Indicando por C e K as faces cara e
Resolução: coroa respectivamente, o espaço amostral
 = {1, 2, 3, 4, 5, 6, 7, 8, 9, 10} deste experimento é Ω = {C, K} e 𝑛(Ω) = 2.
E = {3, 6, 9} O evento que esperamos ocorrer é 𝐸 = {𝐶},
em que 𝑛(𝐸) = 1. Logo:
Então Ec ={1, 2, 4, 5, 7, 8, 10} e representa 1
o evento “é sorteado um número que não é 𝑝(𝐸) =
2
múltiplo de 3”.
1
R: A probabilidade procurada é de ou
Note que 𝐸 ∪ 𝐸 𝑐 = Ω. 2
50%.

Ex.2: No lançamento de um dado, qual a


Probabilidade em espaços amostrais
probabilidade de se obter, na face voltada
equiprováveis
para cima, um número primo?
Espaços amostrais equiprováveis
Resolução:
são aqueles espaços em que a chance de
𝛺 = {1, 2, 3, 4, 5, 6} e 𝑛(𝛺) = 6
qualquer evento ocorrer é igual a de todos
os demais. Assim, ao lançar um dado 𝐸 = {2, 3, 5} e 𝑛(𝐸) = 3
honesto, qualquer uma das faces tem
3 1
exatamente a mesma chance de ficar 𝑝(𝐸) = = = 50%
voltada para cima. O mesmo vale no 6 2
lançamento de uma moeda honesta e na
R: 50%
maioria das questões que vamos tratar de
agora em diante.
Ex.3: No lançamento de duas moedas, qual
A noção de probabilidade é intuitiva
a probabilidade de se obter, nas Faces
e definida por:
voltadas para cima, pelo menos uma cara?
MATEMÁTICA III 47 ANÁLISE COMBINATÓRIA, BINÔMIO DE NEWTON E PROBABILIDADES
Resolução: 86) Todo ano, uma igreja promove um
𝛺 = {(𝐶, 𝐶), (𝐶, 𝐾), (𝐾, 𝐶), (𝐾, 𝐾)} e 𝑛(𝛺) = 4 bazar beneficente para seus
𝐸 = {(𝐶, 𝐶), (𝐶, 𝐾), (𝐾, 𝐶)} e 𝑛(𝐸) = 3 frequentadores. Se a escolha do mês é
aleatória, qual a probabilidade de que esse
3 bazar seja realizado em:
𝑝(𝐸) = = 75% a) fevereiro?
4

R: 75%

Ex.4: No lançamento de dois dados, qual a


probabilidade de se obter a soma dos b) agosto?
pontos igual a 5?
Resolução:
Vamos construir o espaço amostral:
(1,1) (1, 2) (1, 3) (1, 4) (1, 5) (1, 6)
(2, 1) (2, 2) (2, 3) (2, 4) (2, 5) (2, 6)
(3, 1) (3, 2) (3, 3) (3, 4) (3, 5) (3, 6)
𝛺= c) no primeiro trimestre?
(4, 1) (4, 2) (4, 3) (4, 4) (4, 5) (4, 6)
(5, 1) (5, 2) (5, 3) (5, 4) (5, 5) (5, 6)
[(6, 1) (6, 2) (6, 3) (6, 4) (6, 5) (6, 6)]

em que 𝑛(𝛺) = 36
d) no segundo semestre?
E o evento que esperamos ocorrer é:
𝐸 = {(4, 1); (3, 2); (2, 3); (1, 4)} e 𝑛(𝐸) = 4

4
Assim: 𝑝(𝐸) = 36 = 0,111 …
87) Um professor quer sortear um CD entre
R: Aproximadamente 11,1% seus alunos. Na sua turma, há 40 alunos e
o número de rapazes excede o de moças
em 12. Qual a probabilidade de que o CD
seja sorteado para:
85) Um dado perfeito é lançado. Qual a a) uma moça?
probabilidade de que o número obtido ser
múltiplo de 3?

b) um rapaz?

CÁSSIO VIDIGAL 48 IFMG – CAMPUS OURO PRETO


88) Uma moeda é lançada três vezes a) o primeiro número obtido ser maior que o
sucessivamente. Qual a probabilidade de segundo?
observarmos:

a) duas caras e uma coroa? b) a soma dos pontos obtidos ser menor ou
igual a 4?

b) pelo menos duas caras?

c) o produto dos números obtidos ser par?

89) Um dado é lançado duas vezes


sucessivamente. Qual a probabilidade de:

d) não obtermos, em nenhum lançamento,


os números 1 e 6?

MATEMÁTICA III 49 ANÁLISE COMBINATÓRIA, BINÔMIO DE NEWTON E PROBABILIDADES


90) Para testar a eficiência de uma 91) Um paraquedista programou seu pouso
campanha de anúncio do lançamento do em uma fazenda retangular que possui um
novo sabão S, uma agência de propaganda lago no seu interior, conforme indicado
realizou uma pesquisa com 2 mil pessoas. abaixo. Se as condições climáticas não
Por falha da equipe, a agência omitiu os favorecem o paraquedista, o local do pouso
dados dos campos X, Y, Z e W no seu pode se tornar aleatório. Qual é, nesse
relatório sobre a pesquisa, conforme mostra caso, de o paraquedista pousar em terra?
a tabela a seguir: Adote 𝜋 = 3.
Nº de Não
Adquiriram
pessoas adquiriram Total
S
que: S
Viram o
X Y 1500
anuncio
Não viram
200 Z 500
o anuncio
Total 600 W 2000

a) Indique os valores dos campos X, Y, Z e


W.

b) Suponha que uma dessas duas mil


pessoas entrevistadas seja Escolhida ao
acaso e que todas as pessoas tenham a
mesma probabilidade de serem escolhidas.
determine a probabilidade de que esta
pessoa tenha visto o anúncio da campanha
e adquirido o sabão S.

CÁSSIO VIDIGAL 50 IFMG – CAMPUS OURO PRETO


92) Em certa região metropolitana , 52% da 95) Considere a equação linear na variável
população tem mais de 25 anos. Sabe-se x: (𝑎 − 2)𝑥 = 4
ainda que 30% da população com amis de
25 anos tem menos de 35 anos. Se o coeficiente 𝑎 for escolhido ao acaso
Escolhendo-se ao acaso um pessoa dessa entre os elementos {0, 1, 2, …, 9}, qual a
região, qual a probabilidade de essa pessoa probabilidade de que essa equação venha
ter 35 anos ou mais? a ter:
a) um única solução?

b) nenhuma solução

93) Para acessar o sistema de


computadores da empresa, cada
funcionário digita sua senha pessoal
formada por quatro letras distintas de nosso
alfabeto numa ordem preestabelecida. certa
vez um funcionário esqueceu a sua senha
lembrando apenas que ela começava com
K e terminava com W. Qual a probabilidade c) uma solução inteira
de ele ter acertado a senha, ao acaso, numa
única tentativa

96) Os 64 funcionários de uma empresa


responderam um questionário sobre os dois
cursos opcionais oferecidos por ela. Os
94) Três dados são lançados resultados foram os seguintes:
simultaneamente. Qual a probabilidade de 43 frequentam o curso de computação
não ocorrerem três números iguais? 31 frequentam o curso de espanhol
19 frequentam ambos os cursos
escolhendo ao acaso um funcionário da
empresa, qual probabilidade de que ele:

MATEMÁTICA III 51 ANÁLISE COMBINATÓRIA, BINÔMIO DE NEWTON E PROBABILIDADES


a) não frequentou nenhum curso? 99) Joga-se um dado três vezes
consecutivas qual a probabilidade de
surgirem os resultados abaixo em qualquer
ordem?

b) frequente exatamente um curso?

97) De um baralho de 52 cartas, 4 são 100) Oito pessoas sendo 5 homens e 3


extraídas simultaneamente. qual a mulheres serão organizados em uma fila.
probabilidade da ocorrência de: Qual a probabilidade de as pessoas do
a) Duas cartas de ouros e duas de paus? mesmo sexo ficarem juntas?

b) uma carta de cada naipe?


101) Com os algarismos 0, 1, 2, 3, 4 e 5,
formamos números de 3 algarismos. Um
desses números é escolhido ao acaso. Qual
a probabilidade de que ele:
a) seja formado por dois algarismos
distintos?

98) Dentre um grupo formado por dois


homens e quatro mulheres, três pessoas
são escolhidas ao acaso. Qual a
Probabilidade de que sejam escolhidos um
homem e duas mulheres? b) seja par?

CÁSSIO VIDIGAL 52 IFMG – CAMPUS OURO PRETO


102) A figura desta 104) Os cavalos X, Y e Z disputam uma
questão representa uma prova final da qual não poderá ocorrer
bandeira com 4 listras. empate. Sabe-se que a probabilidade de X
Dispondo-se de 4 cores vencer é igual ao dobro da probabilidade de
distintas, deseja-se Y vencer. Da mesma forma, a probabilidade
pintar todas as listras de de Y vencer é igual ao dobro da
modo que listras probabilidade de Z vencer. Calcule a
vizinhas tenham cores probabilidade de:
diferentes. a) X vencer;

a) de quantas maneiras distintas à bandeira


pode ser pintada?

b) Y vencer;

b) escolhendo-se aleatoriamente uma das c) Z vencer.


formas possíveis de pintar a bandeira, qual
probabilidade de seja uma que contenha as
4 cores?

105) Numa cidade com 30.000 domicílios,


10.000 domicílios recebem regularmente o
jornal da loja de eletrodomésticos X, 8.000
recebem regularmente o jornal do
supermercado Y e a metade do número de
domicílios não recebe nenhum dos dois
103) Unindo aleatoriamente dois vértices jornais. determine:
quaisquer de um pentágono, qual a a) o número de domicílios que recebem os
probabilidade de que o segmento dois jornais.
Determinado seja uma diagonal?

b) a probabilidade de um domicílio da
cidade, escolhido ao acaso, receber o jornal
da loja de eletrodomésticos e não receber o
jornal do supermercado.

MATEMÁTICA III 53 ANÁLISE COMBINATÓRIA, BINÔMIO DE NEWTON E PROBABILIDADES


106) Um dado é viciado de tal modo que, ao 108) Um jogo consiste em um prisma
ser lançado, é duas vezes mais provável triangular reto com uma lâmpada em cada
ocorrer face par do que ocorrer face ímpar. vértice e um quadro de interruptores para
Todas as faces pares tem a mesma chance acender essas lâmpadas. Sabendo-se que
de ocorrer e o mesmo acontece entre as quaisquer três lâmpadas podem ser acesas
faces ímpares. Lançando-se esse dado por um único interruptor e que cada
uma vez, qual a probabilidade de ocorrer: interruptor acende precisamente 3
a) face igual a 3? lâmpadas, calcule:
a) quantos interruptores existem nesse
quadro?

b) face par.

b) a probabilidade de, ao se escolher um


interruptor aleatoriamente, este acender
três lâmpadas de uma mesma Face.

107) Uma urna contem x bolas brancas, x2


bolas vermelhas e duas bolas pretas. Uma
bola é escolhida ao acaso e sabe-se que a
probabilidade de ela ser branca é maior que
20%. Quantas bolas brancas essa usa pode
conter?

CÁSSIO VIDIGAL 54 IFMG – CAMPUS OURO PRETO


PROBABILIDADE DA UNIÃO DE
DOIS EVENTOS
Ex.1: Frequentemente assumimos, com
alguma justificativa, que a paternidade leva
Dados dois eventos A e B de um a responsabilidade. Pessoas que passam
espaço amostral S a probabilidade de anos atuando de maneira descuidadosa e
ocorrer A ou B é dada por: irracional de alguma forma parecem se
tornar em pessoas diferentes uma vez que
𝑃(𝐴 𝑈 𝐵) = 𝑃(𝐴) + 𝑃(𝐵) – 𝑃(𝐴 ∩ 𝐵) elas se tornam pais, mudando muitos dos
seus antigos padrões habituais. Suponha
que uma estação de rádio tenha amostrado
100 pessoas, 20 das quais tinham crianças.
Eles observaram que 30 dessas pessoas
usavam cinto de segurança, e que 15
daquelas pessoas tinham crianças. Os
resultados são mostrados na Tabela
Paternidade Usam cinto Não usam cinto Total
Com crianças 15 5 20
Sem crianças 15 65 80
Total 30 70 100

Ex.1: Numa urna existem 10 bolas A partir da informação na tabela acima


numeradas de 1 a 10. Retirando uma bola podemos calcular probabilidades simples
ao acaso, qual a probabilidade de ocorrer (ou marginais ou incondicionais), conjuntas
múltiplos de 2 ou múltiplos de 3? e condicionais.
Resolução
A é o evento “múltiplo de 2”. A probabilidade de uma pessoa amostrada
{2, 4, 6, 8, 10} aleatoriamente usar cinto de segurança é
30
= 30%.
100
B é o evento “múltiplo de 3”.
{3, 6, 9} A probabilidade de uma pessoa ter criança
15
e usar cinto de segurança é 100 = 15%.
𝐴 ∩ 𝐵 (Múltiplo de A e B)
{6}
A probabilidade de uma pessoa usar cinto
5 3 1 7 de segurança dado que tem criança é
15
𝑃(𝐴 𝑈 𝐵) = + – = = 75%.
10 10 10 10 20
R: 70%
A probabilidade de uma pessoa ter criança
_________________________ dado que usa cinto de segurança é
15
= 50%.____________________
30
A probabilidade condicional também
PROBABILIDADE CONDICIONAL
pode ser obtida por:
A probabilidade de um evento A 𝑝(𝐴|𝐵) =
𝑝(𝐴∩𝐵)
ocorrer, dado que um outro evento B 𝑝(𝐵)

ocorreu, é chamada probabilidade Note que 𝑝(𝐴|𝐵) é calculado a partir de


condicional do evento A dado B. probabilidades sobre o espaço amostral
Veja o exemplo a seguir: original
MATEMÁTICA III 55 ANÁLISE COMBINATÓRIA, BINÔMIO DE NEWTON E PROBABILIDADES
PROBABILIDADES DE DOIS A probabilidade de ocorrer os dois eventos
EVENTOS SUCESSIVOS (ou sucessivamente será dada por:
simultâneos) 𝑝(𝐴 ∩ 𝐵) = 𝑝(𝐴) ∙ 𝑝(𝐵|𝐴)
Faremos os cálculos separadamente:
O cálculo da probabilidade de 3 1
𝑝(𝐴) = =
eventos simultâneos determina a chance de 30 10
dois eventos ocorrerem simultânea ou
sucessivamente. Para o cálculo de 𝑝(𝐵|𝐴) é preciso notar
que não teremos mais 30 bolinhas na urna,
A fórmula para o cálculo dessa pois uma foi retirada e não houve reposição,
probabilidade decorre da fórmula da restando 29 bolinhas na urna. Assim,
probabilidade condicional. Assim, teremos: 15
𝑝(𝐵|𝐴) =
𝑝(𝐴 ∩ 𝐵) 29
𝑝(𝐴|𝐵) =
𝑝(𝐵) Logo,

𝑝(𝐴 ∩ 𝐵) = 𝑝(𝐴|𝐵) ∙ 𝑝(𝐵) 1 15 3
𝑝(𝐴 ∩ 𝐵) = ∙ =
Isto significa que, para se avaliar a 10 29 58
probabilidade de ocorrerem dois eventos 3
simultâneos (ou sucessivos), que é Resp.: 58 ou 5,2%
𝑝(𝐴 ∩ 𝐵), basta multiplicar a probabilidade
de ocorrer um deles (𝑝(𝐵)) pela Ex.2: Numa caixa estão 20 livros sendo 12
probabilidade de ocorrer o outro, sabendo de Matemática e 8 de Administração. Dois
que o primeiro já ocorreu (𝑝(𝐴|𝐵)). deles são retirados sucessivamente e sem
reposição. Qual a probabilidade de serem
Vamos fazer alguns exemplos para escolhidos dois livros de Matemática?
explorar o uso da fórmula e a maneira
correta de interpretar os problemas Resolução: Mais uma vez, os eventos não
relacionados à probabilidade de eventos são independentes. Vamos determinar
simultâneos. cada um dos eventos.
A: Sair livro de Matemática: 𝑛(𝐸) = 12 e
𝑛(Ω) = 20.
Ex.1: Numa urna há 30 bolinhas numeradas B: Sair outro livro de Matemática: 𝑛(𝐸) = 11
de 1 a 30. Serão retiradas dessa urna duas e 𝑛(Ω) = 19.
bolinhas, ao acaso, uma após a outra, sem
reposição. Qual a probabilidade de sair um 12 3
𝑝(𝐴) = =
múltiplo de 10 na primeira e um número 20 5
ímpar na segunda?
11
𝑝(𝐵|𝐴) =
Resolução: o fato de a retirada das bolinhas 19
ocorrer sem reposição, implica que a
ocorrência do primeiro evento interfere na Logo,
probabilidade do segundo ocorrer. Portanto,
esses eventos não são independentes. 3 11 33
𝑝(𝐴 ∩ 𝐵) = ∙ =
Vamos determinar cada um dos eventos. 5 19 95
A: sair um múltiplo de 10 EA= {10, 20, 30}
33
B: sair um número ímpar EB={1, 3, 5, 7, 9, Resp.: 95 ou 34,7%
11, 13, 15, 17, 19, 21, 23, 25, 27, 29}
CÁSSIO VIDIGAL 56 IFMG – CAMPUS OURO PRETO
Ex.3: Em dois lançamentos sucessivos de 110) Um dado honesto é lançado. Qual a
um mesmo dado, qual a probabilidade de probabilidade de observarmos:
ocorrer um número maior que 3 e o número a) um múltiplo de 2 ou um múltiplo de 3?
2?

Resolução: note que, neste caso, a


ocorrência de um evento não influencia a
probabilidade de outro ocorrer, portanto são
dois eventos independentes. Vamos b) um par o um divisor de 5?
distinguir os dois eventos:

A: sair um número maior que 3 {4, 5, 6}.


B: sair o número 2
Vamos calcular a probabilidade de
ocorrência de cada um dos eventos. 111) No cadastro de um cursinho pré-enem
Observe que no lançamento de um dado, estão registrados 600 alunos sendo 380
temos 6 valores possíveis. Assim: meninos, 105 moças que já concluíram o
Ensino Médio e 200 meninos que ainda
3 1 estão cursando o Ensino Médio.
𝑝(𝐴) = =
6 2 Um nome do cadastro é selecionado ao
1 acaso. Qual a probabilidade de o nome
𝑝(𝐵) =
6 escolhido ser de:
a) uma menina?
Dessa forma, teremos:
1 1 1
𝑝= ∙ =
2 6 12
1
Resp.: 12 ou 8,33%

b) um menino que já concluiu o E.M.?


109) Considerando a situação apresentada
no Exemplo 2 (pág. 56 desta apostila), qual
a probabilidade de serem retirados,
aleatoriamente, dois livros de assuntos
diferentes?

c) um menino ou alguém que ainda está


cursando o E.M.?

MATEMÁTICA III 57 ANÁLISE COMBINATÓRIA, BINÔMIO DE NEWTON E PROBABILIDADES


112) Numa urna há 30 bolinhas numeradas 114) Um dado é lançado e se observa que
de 1 a 30. Uma delas é extraída ao acaso. o número obtido é par. Qual a probabilidade
Qual a probabilidade de o número indicado de ele ser maior que 3?
seja:
a) par ou múltiplo de 3?

115) Escolhendo-se ao acaso um número


do conjunto {𝑥 ∈ ℕ|1 ≤ 𝑥 ≤ 100}.
b) múltiplo de 5 ou de 7? a) Sabendo que o número escolhido é um
quadrado perfeito, qual a probabilidade de
ele ser par?

c) par ou múltiplo de 6?
b) Sabendo que o número escolhido é
múltiplo de 6, qual a probabilidade de ele
ser múltiplo de 10? E de 3?

113) A probabilidade de chover 5 ou mais


vezes por mês em uma praia da
Pernambuco é de 33%. A probabilidade de
chover 5 ou menos vezes no mês é de 81%.
Qual a probabilidade de chover exatamente
5 vezes?

CÁSSIO VIDIGAL 58 IFMG – CAMPUS OURO PRETO


116) Dois dados são lançados a) Qual a probabilidade de a ficha escolhida
simultaneamente. ser do Bloco A e estar quite com o
a) Qual a probabilidade de a soma dos condomínio?
pontos obtidos seja 10 sabendo-se que os
números obtidos são distintos?

b) Sabendo-se que a fica escolhida é do


bloco B, qual a probabilidade de ser de um
morador com condomínio atrasado?

b) Qual a probabilidade de se obter


números distintos sabendo-se que a soma 118) Num certo pais, 10% das declarações
dos pontos é 10? de Imposto de renda são suspeitas e
direcionadas para uma análise detalhada;
dentre estas, verificou-se que 20% são de
declarações fraudulentas. Entre as não
suspeitas, 2% são fraudulentas.
a) Se uma declaração é escolhida ao acaso,
qual a probabilidade de ela ser suspeita e
fraudulenta?

b) Se uma declaração é fraudulenta, qual a


probabilidade de ela ter sido suspeita?
117) Num prédio residencial há dois blocos:
Bloco A e Bloco B. No Bloco A, há 80
apartamentos dos quais 15% estão em
atraso com o pagamento do condomínio.
NO blobo B há 50 apartamentos, dos quais,
10% estão com as contas em atraso. As
fichas relativas a todos os apartamentos
estão reunidas e uma é escolhida ao acaso.

MATEMÁTICA III 59 ANÁLISE COMBINATÓRIA, BINÔMIO DE NEWTON E PROBABILIDADES


119) Uma moeda e um dado são lançados 121) Uma urna (I) contém 3 bolas
simultaneamente. Qual a probabilidade de: vermelhas, 4 brancas e 3 pretas. Outra urna
a) Ocorrer cara e o número 1? (II) contém 2 bolas vermelhas, 5 brancas e
2 pretas. Uma das urnas é escolhida ao
acaso e dela é extraída uma bola:
a) qual a probabilidade de ocorrer urna (I) e
bola branca?

b) Ocorrer coroa e um número primo?

b) qual a probabilidade de ocorrer bola


vermelha?

120) Uma urna contem 10 etiquetas


identificadas pelas letras A, B, C, D, E, F, G,
H, I e J. Duas delas são retiradas ao acaso,
sucessivamente. Qual a probabilidade de 122) Na prateleira de um supermercado há
saírem duas vogais, se a extração é feita: 20 latas de achocolatado das quais 4 estão
a) com reposição? além do prazo de validade. Uma mulher
passa e apanha uma delas ao acaso; logo
em seguida o rapaz apanha outra lata ao
acaso. Qual a probabilidade de:
a) ambos terem comprado achocolatado
com prazo dentro da validade?

b) sem reposição?
b) a mulher tenha comprado o produto com
prazo dentro da validade mas o rapaz não?

CÁSSIO VIDIGAL 60 IFMG – CAMPUS OURO PRETO


123) No canil há 10 cachorros sendo 7 da a) escolhendo-se, ao acaso, um aluno do
raça X e 3 da raça Y, cada um dentro de terceiro ano, a probabilidade de ser homem
uma ”jaula” fechada nas laterais. A é de 45%.
probabilidade de um cachorro da raça X latir
para um desconhecido é de 80% e, para a
raça Y, essa probabilidade é de 60%. Um
visitante chega ao canil, para ao acaso
diante de uma jaula e ver então o cachorro
que está nela. Qual a probabilidade de esse
cão não latir para o visitante? b) escolhendo-se, ao acaso, um aluno do
terceiro ano, a probabilidade de ser mulher
é igual a 20%.

c) escolhendo-se, ao acaso,
simultaneamente, dois alunos, sendo um de
cada turma, a probabilidade de ambos
16
serem do mesmo sexo é de .
33

d) escolhendo-se, ao acaso, um aluno do


terceiro ano, a probabilidade de ser mulher
ou ser da turma B é igual a 80%.

124) Em uma escola existem duas turmas


do terceiro ano: A e B. A tabela mostra a
distribuição por sexo dos alunos da turma.
e) reunindo-se as mulheres das duas
turmas escolhendo uma ao acaso a
Turma Homens Mulheres
probabilidade de ser da turma A é de 35%.
A 20 35
B 25 20

com base nesses dados, classifique cada


uma das afirmações seguintes como
verdadeira ou falsa.
MATEMÁTICA III 61 ANÁLISE COMBINATÓRIA, BINÔMIO DE NEWTON E PROBABILIDADES
125) Na loteria 126) Fernando e Cláudio foram pescar num
esportiva (Loteca) lago onde só existem trutas e carpas.
há 13 jogos. Em Fernando pescou. no total, o triplo da
cada jogo aposta- quantidade pescada por Cláudio. Fernando
se em um dos pescou duas vezes mais trutas do que
times (Coluna 1 ou carpas, enquanto Cláudio pescou
Coluna 2) ou em quantidades iguais de cartas de trutas. Os
empate (Coluna do peixes foram todos jogados num balaio e
meio). A aposta mínima é de um duplo, isto uma truta foi escolhida ao acaso nesse
é, num único jogo assinalam-se duas balaio. Qual a probabilidade de esta fruta ter
opções ao preço de R$0,50. Há também o sido escada por Fernando?
palpite triplo: as três colunas são
assinaladas para um determinado jogo.
Apostar em um triplo custa R$0,75.

a) qual a probabilidade de se acertar os 13


jogos com aposta mínima?

b) qual a probabilidade de se acertarem os 127) Um dado, cujas faces estão


13 jogos apostando um triplo? numeradas de 1 a 6, é dito perfeito se cada
1
uma das seis faces tem probabilidade 6 de
ocorrer em um lançamento. Considere o
experimento que consiste em três
lançamentos independentes de um dado
perfeito. Calcule a probabilidade de que o
produto desses três números seja:
c) na aposta máxima, são assinalados 5
a) par
duplos e triplos ao preço de R$216,00. qual
a probabilidade de se acertarem os 13 jogos
com a aposta máxima? a proporcionalidade
entre os valores apostados e as chances de
acerto considerando-se as apostas mínima
e máxima?

b) múltiplo de 10

CÁSSIO VIDIGAL 62 IFMG – CAMPUS OURO PRETO


128) Uma moeda honesta é lançada sete ATIVIDADES COMPLEMENTARES
vezes. Qual a probabilidade de ocorrerem Pág. 184 – Exercícios 01 a 07
duas caras e cinco coroas? Pág. 187 – Exercícios 08 a 15
Pág. 192 e 193 – Exercícios 22 a 27
Pág. 196 – Exercícios 28 a 37

129) A incidência de uma doença numa


população é de 30%. Se oito pessoas
submetem-se a um teste para detecção da
doença, qual é a probabilidade de cinco
delas apresentarem teste positivo?

130) Um aluno, afobado com o tempo que


lhe resta de prova, decide chutar as últimas
10 questões do ENEM. Cada questão
apresenta 5 alternativas. Qual é a
probabilidade de um aluno acertar 4 destas
10?

MATEMÁTICA III 63 ANÁLISE COMBINATÓRIA, BINÔMIO DE NEWTON E PROBABILIDADES


RESPOSTAS 28) a) 120 b) 60
1) a) Pantalona Preta, Flare Preta, Legging
29) a) 120 b) 30
Preta, Sarouel Preta e Skinny Preta;
Pantalona Marrom, Flare Marrom, Legging 30) a) 6 b) 24
Marrom, Sarouel Marrom e Skinny Marrom; c) 720 d) 362 880
e Pantalona Azul, Flare Azul, Legging Azul, e) 120 f) 3 628 800
Sarouel Azul e Skinny Azul.
b) 15 opções 31) 24

2) 125 números 32) a) 120 b) 5040


c) 8 d) 1⁄90
3) 60 números

4) 4680 33) a) 362 880 b) 161 280


c) 100 800 d) 5040
2 resultados. Aline-Bárbara ou Bárbara-
5) Aline
e) Metade do total

6 resultados. ABC, ACB, BAC, BCA, CAB e 34) a) 120 b) 24


6) CBA
35) 8640
7) 24 resultados.
36) a) 12! b) 2 ∙ (6!)2
8) 6 modelos. c) 36 ∙ 10! d) (6!)2

37) 576 38) 103 680

39) a) S = { 4 } b)
9) 56 maneiras.
40) 293ª
10) VV, VF, FV e FF. 4 maneiras.
41) a) 120 b) 24
VVV, VVF, VFV, VFF, FVV, FVF. FFV, FFF. c) 2156 d) 0
11) 8 maneiras.
42) 95º 43) 151 200
12) 16 maneiras. 13) 1024 maneiras.
44) 125 970 45) 60
14) 520 maneiras. 15) 276 anagramas
46) 40 47) 136
16) 27 ∙ 26 ∙ 25 ∙ 24 ∙ 23 ∙ 22 anagramas. (ou 213
127 200 anagramas) 48) 220 49) 286

17) a) 10 b) 1⁄90 50) 10


c) 3 d) 𝑛2 − 𝑛
1 51) a) 15 120 b) 5040 c) 8400
e)
𝑛2 −3𝑛+2
52) 5 53) 84
18) S={4} 19) 90
54) 2002 55) 420
20) a) 32 292 000 b) 1 723 680
56) 867 57) 126
21) a) 210 b) 110
c) 5 d) 120 58) a) 10 b) 6

22) a) 720 b) 72 c) 504 59) a) 165 b) 84 c) 20


d) 0 e) 16
23) a) 729 b) 504 c) 225
60) a) 45 b) 120
24) 336 25) 420
61) 3150
26) S={ 11 }
62) a) 270 725 b) 715
27) a) 336 b) 60
c) 126 d) 180 63) a) 16 b) 78 c) 169

CÁSSIO VIDIGAL 64 IFMG – CAMPUS OURO PRETO


64) a) 22 308 b) 249 900 c) 103 776

65) 2160 96) a)


9
64

66) a) S = { 17 } b) S = { 5 } 97) a) 2,25% b) 10% c) 50%

67) 70 98) a) 60% b)


9
16

68) a) 4725 b) 792 c) 86 99) a)


1
b) 10,5%
36
69) 1225 100) 1
70) a) 84 b) 28 c) 42 28
101) a)
5
71) 28 72) 6 9

102) a) 108
73) 50 063 860 74) R$ 175.223.510,00
103) a) 50% b)
1
75) a) 7 b) 28 c) 84 2
d) 210 e) 462 f) 924 104) a)
4
b)
2
7 9
g) 1716 h) 3003 i) 5005
105) a) 3000
76) a) 6 b) 84 c) 2
106) a)
1
b)
2
c)
1
c) 9 d) p+2 9 7 7

77) a) S = {8, 5 } b) S = {1, 4} 107) 1, 2 ou 3 b)


7
30

78) a) 34 b)
6
25
108) a) 20 b)
2
3
c) 154 d) 24 = 16
109) Resolução:
Nesta situação, são dois casos que nos
79) 10 80) a = 10
interessam:
11 No primeiro caso, retirarmos, nesta ordem, um
81) a) 1 + 3𝑥 2 + 3𝑥 4 + 𝑥 6 livro de Matemática e outro de administração ou,
num segundo caso, retirarmos, primeiro, um
b) 𝑥 5 − 15𝑥 4 𝑦 + 90𝑥 3 𝑦 2 − 270𝑥 2 𝑦 3 +
livro de Administração e depois um de
+405𝑥𝑦 4 − 243𝑦 5 Matemática.
4 2 4 1
c) 𝑥 − 4𝑥 − 2 + 4 𝑝 = 𝑝(𝑀 ∩ 𝐴) 𝑜𝑢 𝑝(𝐴 ∩ 𝑀)
𝑥 𝑥
Em que M é o evento “Retirar um livro de
82) a) 1 b) 308/243 Matemática” e A, “Retirar um livro de
Administração”.
83) a) 45𝑥 16 b) −252𝑥 10

84) a) não existe b) −165𝑥 13 𝑝(𝑀 ∩ 𝐴)


1
85) M: 𝑛(𝑀) = 12 n e (Ω) = 20.
3 A: 𝑛(𝐴) = 8 e 𝑛(Ω) = 19.
86) a)
1
b)
1
c)
1
d)
1 12 8 24
12 12 4 2 𝑝(𝑀 ∩ 𝐴) = ∙ =⋯=
20 19 95
87) a) 35% b) 65%
𝑝(𝐴 ∩ 𝑀)
88) a)
3
8
b)
1
2 A: 𝑛(𝐴) = 8 e 𝑛(Ω) = 20.
M: 𝑛(𝑀) = 12 n e (Ω) = 19.
89) a)
5
12
b)
1
6
c)
3
4
d)
4
9
8 12 24
𝑝(𝑀 ∩ 𝐴) = ∙ =⋯=
90) a) X = 400, Y = 1100, Z = 300 e W = 1400 20 19 95
Assim:
b) 20% 𝑝 = 𝑝(𝑀 ∩ 𝐴) 𝑜𝑢 𝑝(𝐴 ∩ 𝑀)
24 24 48
91) 88,75% 92) 36,4% 𝑝= + =
95 95 95
93) 1 94) 35
552 36 48
Resp.:
95
= 50,5%
95) a) 90%
MATEMÁTICA III 65 ANÁLISE COMBINATÓRIA, BINÔMIO DE NEWTON E PROBABILIDADES
110) a)
2
b)
5
PAIVA, Manoel; Matemática – Ensino
5 6

111) a)
11
b)
3
c)
33 Médio, Volume 2. 2.ed. São Paulo.
30 10 40

112) a)
2
b)
1 Moderna, 2013.
3 3

14% 114) 2 RIGONATTO, Marcelo. "Probabilidade de


113)
3 eventos simultâneos"; Brasil Escola.
115) a) 50% b)
3
, 100%
16 Disponível em <http://brasilescola.
1 2
116) a) b) uol.com.br/matematica/probabilidade-
15 3
34 5
117) a) b) eventos-simultaneos.htm>. Acesso em 30
65 17

118) a) 2% b) 52,6% de maio de 2016.


1 1
119) a) b)
12 4
1
120) a) 9% b) Links dos vídeos sugeridos:
15
1 47
121) a)
5
b)
180
Pág. 04:
122) a)
12
b)
16
www.vidigal.ouropreto.ifmg.edu.br/princmul
19 95

123) 26% tadit/


124) a) Verdadeira b) Falsa, é 44,4%.
c) Verdadeira d) Verdadeira
7
e) Falsa, é . Pág. 09:
11

125) a) 0,000125% b) 0,000188% www.vidigal.ouropreto.ifmg.edu.br/fatorial/


c) 0,054%; sim pois tanto a chance de
ganhar quanto o custo da aposta foram
multiplicados por 432.
Pág. 44
126) 80%
www.vidigal.ouropreto.ifmg.edu.br/termo-
127) a)
7
b)
5
8 36
geral-do-binomial/
128) 16,4%
129) 4,67%
130) 8,8%

REFERÊNCIA BIBLIOGRÁFICA

DANTE, Luiz Roberto; Matemática. São


Paulo, Ática, 2004
MACHADO, Antônio dos Santos;
Matemática, Temas e Metas. São Paulo,
Atual, 1988
IEZZI, Gelson e outros; Matemática,
Volume único. São Paulo, Atual, 2002
CÁSSIO VIDIGAL 66 IFMG – CAMPUS OURO PRETO
Equações de primeiro grau

Para resolver um problema matemático, quase sempre devemos transformar uma sentença apresentada
com palavras em uma sentença que esteja escrita em linguagem matemática. Esta é a parte mais
importante e talvez seja a mais difícil da Matemática.

Sentença com palavras Sentença matemática

2 melancias + 2Kg = 14Kg 2 x + 2 = 14


Normalmente aparecem letras conhecidas como variáveis ou incógnitas. A partir daqui, a Matemática se
posiciona perante diferentes situações e será necessário conhecer o valor de algo desconhecido, que é o
objetivo do estudo de equações.

Equações do primeiro grau em 1 variável


Trabalharemos com uma situação real e dela tiraremos algumas informações importantes. Observe a
balança:

A balança está equilibrada. No prato esquerdo há um "peso" de 2Kg e duas melancias com "pesos" iguais.
No prato direito há um "peso" de 14Kg. Quanto pesa cada melancia?
2 melancias + 2Kg = 14Kg
Usaremos uma letra qualquer, por exemplo x, para simbolizar o peso de cada melancia. Assim, a equação
poderá ser escrita, do ponto de vista matemático, como:
2x + 2 = 14
Este é um exemplo simples de uma equação contendo uma variável, mas que é extremamente útil e
aparece na maioria das situações reais. Valorize este exemplo simples.
Podemos ver que toda equação tem:
 Uma ou mais letras indicando valores desconhecidos, que são denominadas variáveis ou
incógnitas;
 Um sinal de igualdade, denotado por =.
 Uma expressão à esquerda da igualdade, denominada primeiro membro ou membro da esquerda;
 Uma expressão à direita da igualdade, denominada segundo membro ou membro da direita.
A letra x é a incógnita da equação. A palavra incógnita significa desconhecida e equação tem o prefixo equa
que provém do Latim e significa igual.

2x+2 = 14
1o. membro sinal de igualdade 2o. membro

As expressões do primeiro e segundo membro da equação são os termos da equação.


Para resolver essa equação, utilizamos o seguinte processo para obter o valor de x.

2x + 2 = 14 Equação original
2x + 2 - 2 = 14 - 2 Subtraímos 2 dos dois membros
2x = 12 Dividimos por 2 os dois membros
x=6 Solução
Observação: Quando adicionamos (ou subtraímos) valores iguais em ambos os membros da equação, ela
permanece em equilíbrio. Da mesma forma, se multiplicamos ou dividimos ambos os membros da equação
por um valor não nulo, a equação permanece em equilíbrio. Este processo nos permite resolver uma
equação, ou seja, permite obter as raízes da equação.
Exemplos:
1. A soma das idades de André e Carlos é 22 anos. Descubra as idades de cada um deles, sabendo-
se que André é 4 anos mais novo do que Carlos.
Solução: Primeiro passamos o problema para a linguagem matemática. Vamos tomar a letra c para
a idade de Carlos e a letra a para a idade de André, logo a=c-4. Assim:
c + a = 22
c + (c - 4) = 22
2c - 4 = 22
2c - 4 + 4 = 22 + 4
2c = 26
c = 13

Resposta: Carlos tem 13 anos e André tem 13-4=9 anos.


2. A população de uma cidade A é o triplo da população da cidade B. Se as duas cidades juntas têm
uma população de 100.000 habitantes, quantos habitantes tem a cidade B?
Solução: Identificaremos a população da cidade A com a letra a e a população da cidade com a letra
b. Assumiremos que a=3b. Dessa forma, poderemos escrever:
a + b = 100.000
3b + b = 100.000
4b = 100.000
b = 25.000

Resposta: Como a=3b, então a população de A corresponde a: a=3×25.000=75.000 habitantes.


3. Uma casa com 260m2 de área construída possui 3 quartos de mesmo tamanho. Qual é a área de
cada quarto, se as outras dependências da casa ocupam 140m 2?
Solução: Tomaremos a área de cada dormitório com letra x.
3x + 140 = 260
3x = 260 -140
3x = 120
x = 40

Resposta: Cada quarto tem 40m2.


Exercícios: Resolver as equações
1. 2x + 4 = 10
2. 5k - 12 = 20
3. 2y + 15 - y = 22
4. 9h - 2 = 16 + 2h

Desigualdades do primeiro grau em 1 variável


Relacionadas com as equações de primeiro grau, existem as desigualdades de primeiro grau, (também
denominadas inequações) que são expressões matemáticas em que os termos estão ligados por um dos
quatro sinais:

< menor
> maior
< menor ou igual
> maior ou igual
Nas desigualdades, o objetivo é obter um conjunto de todas os possíveis valores que pode(m) assumir uma
ou mais incógnitas na equação proposta.
Exemplo: Determinar todos os números inteiros positivos para os quais vale a desigualdade:
2x + 2 < 14
Para resolver esta desigualdade, seguiremos os seguintes passos:

Passo 1 2x + 2 < 14 Escrever a equação original


Passo 2 2x + 2 - 2 < 14 - 2 Subtrair o número 2 dos dois membros
Passo 3 2x < 12 Dividir pelo número 2 ambos os membros
Passo 4 x<6 Solução
Concluímos que o conjunto solução é formado por todos os números inteiros positivos menores do que 6:
S = {1, 2, 3, 4, 5}
Exemplo: Para obter todos os números pares positivos que satisfazem à desigualdade
2x + 2 < 14
obteremos o conjunto solução:
S = {2, 4}
Observação: Se há mais do que um sinal de desigualdade na expressão, temos várias desigualdades
"disfarçadas" em uma.
Exemplo: Para determinar todos os números inteiros positivos para os quais valem as (duas)
desigualdades:
12 < 2x + 2 < 20
poderemos seguir o seguinte processo:

12 < 2x + 2 < 20 Equação original


12 - 2 < 2x + 2 - 2 < 20 - 2 Subtraímos 2 de todos os membros
10 < 2x < 18 Dividimos por 2 todos os membros
5 < x < 9 Solução
O conjunto solução é:
S = {6, 7, 8, 9}
Exemplo: Para obter todos os números inteiros negativos que satisfazem às (duas) desigualdades
12 < 2x + 2 < 20
obteremos apenas o conjunto vazio, como solução, isto é:
S=Ø={}

Desigualdades do primeiro grau em 2 variáveis


Uma situação comum em aplicações é aquela em que temos uma desigualdade envolvendo uma equação
com 2 ou mais incógnitas. Estudaremos aqui apenas o caso em aparecem 2 incógnitas x e y. Uma forma
geral típica, pode ser:
ax+by<c
onde a, b e c são valores dados.
Exemplo: Para obter todos os pares ordenados de números reais para os quais:
2x + 3y > 0
observamos que o conjunto solução contém os pares:
(0,0), (1,0), (0,1), (-1,1), (1,-1), ...
Há infinitos pares ordenados de números reais satisfazendo a esta desigualdade, o que torna impossível
exibir todas as soluções. Para remediar isto, utilizaremos um processo geométrico que permitirá obter uma
solução geométrica satisfatória.
Processo geométrico:
(1) Traçamos a reta 2x+3y=0;
(2) Escolhemos um par ordenado, como (1,1), fora da reta;
(3) Se (1,1) satisfaz à desigualdade 2x+3y>0, colorimos a região que contém este ponto, caso contrário,
colorimos a região que está do outro lado da reta.
(4) A região colorida é o conjunto solução para a desigualdade.

Sistemas linear de equações do primeiro grau


Uma equação do primeiro grau, é aquela em que todas as incógnitas estão elevadas à potência 1. Este tipo
de equação poderá ter mais do que uma incógnita.
Um sistema de equações do primeiro grau em duas incógnitas x e y, é um conjunto formado por duas
equações do primeiro nessas duas incógnitas.
Exemplo: Seja o sistema de duas equações:
2 x + 3 y = 38
3 x - 2 y = 18
Resolver este sistema de equações é o mesmo que obter os valores de x e de y que satisfazem
simultaneamente a ambas as equações.
x=10 e y=6 são as soluções deste sistema e denotamos esta resposta como um par ordenado de números
reais:
S = { (10,6) }

Método de substituição para resolver este sistema


Entre muitos outros, o método da substituição, consiste na idéia básica de isolar o valor algébrico de uma
das variáveis, por exemplo x, e, aplicar o resultado à outra equação.
Para entender o método, consideremos o sistema:
2 x + 3 y = 38
3 x - 2 y = 18
Para extrair o valor de x na primeira equação, usaremos o seguinte processo:

2x + 3y = 38 Primeira equação
2x + 3y - 3y = 38 - 3y Subtraímos 3y de ambos os membros
2x = 38 - 3y Dividimos ambos os membros por 2
x = 19 - (3y/2) Este é o valor de x em função de y
Substituímos aqora o valor de x na segunda equação 3x-2y=18:

3x - 2y = 18 Segunda equação
3(19 - (3y/2)) - 2y = 18 Após substituir x, eliminamos os parênteses
57 - 9y/2 - 2y = 18 multiplicamos os termos por 2
114 - 9y - 4y = 36 reduzimos os termos semelhantes
114 - 13y = 36 separamos variáveis e números
114 - 36 = 13y simplificamos a equação
78 = 13y mudamos a posição dos dois membros
13 y = 78 dividimos ambos os membros por 6
y=6 Valor obtido para y
Substituindo y=6 na equação x=19-(3y/2), obtemos:
x = 19 - (3×6/2) = 19 - 18/2 = 19-9 = 10
Exercício: Determinar a solução do sistema:
x+y=2
x-y=0
Cada equação do sistema acima pode ser visto como reta no plano cartesiano. Construa as duas retas no
plano e verifique que, neste caso, a solução é um par ordenado que pertence à interseção das duas retas.

Relação entre sistemas lineares e retas no plano


No contexto que estamos trabalhando aqui, cada equação da forma ax+by=c, representa uma reta no plano
cartesiano. Um sistema com duas equações de primeiro grau em 2 incógnitas sempre pode ser interpretado
como um conjunto de duas retas localizadas no plano cartesiano.
Reta 1: ax + by = c
Reta 2: dx + ey = f
Há três modos de construir retas no plano: retas concorrentes, retas paralelas e retas coincidentes.

Se o sistema é formado por duas equações que são retas no plano cartesiano, temos a ocorrência de:
Retas concorrentes: quando o sistema admite uma única solução que é um par ordenado localizado na
interseção das duas retas;
Retas paralelas: quando o não admite solução, pois um ponto não pode estar localizado em duas retas
paralelas;
Retas coincidentes: quando o admite uma infinidade de soluções pois as retas estão sobrepostas.
Exemplos das três situações
Tipos de retas Sistema
x+y=2
Concorrentes
x-y=0
x+y=2
Paralelas
x+y=4
x+y=2
Coincidentes
2x + 2y = 4
Problemas com sistemas de equações:
1. A soma das idades de André e Carlos é 22 anos. Descubra as idades de cada um deles, sabendo-
se que André é 4 anos mais novo do que Carlos.
Solução: A idade de André será tomada com a letra A e a idade de Carlos com a letra C. O sistema
de equações será:
C + A = 22
C-A=4

Resposta: C = 13 e A = 9
2. A população de uma cidade A é o triplo da população da cidade B. Se as duas cidades juntas têm
uma população de 100.000 habitantes, quantos habitantes tem a cidade B?
Solucão: Identificando a população da cidade A com a letra A e a população da cidade B com B, o
sistema de equações será:
A + B = 100000
A = 3B

Resposta: A = 75000, B= 25000.


3. Uma casa com 260m2 de área construída tem 3 dormitórios de mesmo tamanho. Qual é a área de
cada dormitório se as outras dependências da casa ocupam 140m 2?
Solução: Identificaremos a área de cada dormitório com a letra D e a área das outras dependências
com a letra O. Assim, o sistema será:
3D + O = 260
O = 140

Resposta: D = 40
Desigualdades com 2 Equações em 2 variáveis
Outra situação bastante comum é aquela em que existe uma desigualdade com 2 equações em 2 ou mais
incógnitas. Estudaremos aqui apenas o caso em aparecem 2 equações e 2 incógnitas x e y. Uma forma
geral pode ter a seguinte forma típica:
ax+by<c
dx+ey>f
onde as constantes: a, b, c, d, e, f; são conhecidas.
Exemplo: Determinar todos os pares ordenados de números reais para os quais:
2x + 3y > 6
5x + 2y < 20
Há infinitos pares ordenados de números reais satisfazendo a esta desigualdade, o que torna impossível
exibir todas as soluções. Para remediar isto, utilizaremos um processo geométrico que permitirá obter uma
solução geométrica satisfatória.
Processo geométrico:
(1) Traçar a reta 2x+3y=6 (em vermelho);
(2) Escolher um ponto fora da reta, como o par (2,2) e observar que ele satisfaz à primeira desigualdade;
(3) Devemos colorir o semi-plano contendo o ponto (2,2) (em verde);
(4) Traçar a reta 5x+2y=20 (em azul);
(5) Escolher um ponto fora da reta, por exemplo, o próprio par já usado antes (2,2) (não é necessário que
seja o mesmo) e observamos que ele satisfaz à segunda desigualdade;
(6) Colorir o semi-plano contendo o ponto (2,2), inclusive a própria reta. (cor azul)
(7) Construir a interseção (em vermelho) das duas regiões coloridas.
(8) Esta interseção é o conjunto solução para o sistema com as duas desigualdades.

Esta situação gráfica é bastante utilizada em aplicações da Matemática a estudos de Economia e Processos
de otimização. Um dos ramos da Matemática que estuda este assunto é a Pesquisa Operacional.
Equações do segundo grau

Equações algébricas são equações nas quais a incógnita x está sujeita a operações algébricas como:
adição, subtração, multiplicação, divisão e radiciação.
Exemplos:
1. a x + b = 0
2. a x² + bx + c = 0
3. a x4 + b x² + c = 0
Uma equação algébrica está em sua forma canônica, quando ela pode ser escrita como:
ao xn + a1 xn-1 + ... + an-1 x1 + an = 0

onde n é um número inteiro positivo (número natural). O maior expoente da incógnita em uma equação
algébrica é denominado o grau da equação e o coeficiente do termo de mais alto grau é denominado
coeficiente do termo dominante.
Exemplo: A equação 4x²+3x+2=0 tem o grau 2 e o coeficiente do termo dominante é 4. Neste caso,
dizemos que esta é uma equação do segundo grau.

A fórmula quadrática de Sridhara (Bhaskara)


Mostraremos na sequência como o matemático Sridhara, obteve a Fórmula (conhecida como sendo) de
Bhaskara, que é a fórmula geral para a resolução de equações do segundo grau. Um fato curioso é que a
Fórmula de Bhaskara não foi descoberta por ele mas pelo matemático hindu Sridhara, pelo menos um
século antes da publicação de Bhaskara, fato reconhecido pelo próprio Bhaskara, embora o material
construído pelo pioneiro não tenha chegado até nós.
O fundamento usado para obter esta fórmula foi buscar uma forma de reduzir a equação do segundo grau a
uma do primeiro grau, através da extração de raízes quadradas de ambos os membros da mesma.
Seja a equação:
a x² + b x + c = 0
com a não nulo e dividindo todos os coeficientes por a, temos:
x² + (b/a) x + c/a = 0
Passando o termo constante para o segundo membro, teremos:
x² + (b/a) x = -c/a
Prosseguindo, faremos com que o lado esquerdo da equação seja um quadrado perfeito e para isto
somaremos o quadrado de b/2a a ambos os membros da equação para obter:
x² + (b/a) x + (b/2a)² = -c/a + (b/2a)²
Simplificando ambos os lados da equação, obteremos:
[x+(b/2a)]2 = (b² - 4ac) / 4a²
Notação: Usaremos a notação R[x] para representar a raiz quadrada de x>0. R[5] representará a raiz
quadrada de 5. Esta notação está sendo introduzida aqui para fazer com que a página seja carregada mais
rapidamente, pois a linguagem HTML ainda não permite apresentar notações matemáticas na Internet de
uma forma fácil.
Extraindo a raiz quadrada de cada membro da equação e lembrando que a raiz quadrada de todo número
real não negativo é também não negativa, obteremos duas respostas para a nossa equação:
x + (b/2a) = + R[(b²-4ac) / 4a²]
ou
x + (b/2a) = - R[(b²-4ac) / 4a²]
que alguns, por preguiça ou descuido, escrevem:
contendo um sinal ± que é lido como mais ou menos. Lembramos que este sinal ± não tem qualquer
significado em Matemática.
Como estamos procurando duas raízes para a equação do segundo grau, deveremos sempre escrever:
x' = -b/2a + R[b²-4ac] /2a
ou
x" = -b/2a - R[b²-4ac] /2a
A fórmula de Bhaskara ainda pode ser escrita como:

onde D (às vezes usamos a letra maiúscula "delta" do alfabeto grego) é o discriminante da equação do
segundo grau, definido por:
D = b² - 4ac

Equação do segundo grau


Uma equação do segundo grau na incógnita x é da forma:
a x² + b x + c = 0
onde os números reais a, b e c são os coeficientes da equação, sendo que a deve ser diferente de zero.
Essa equação é também chamada de equação quadrática, pois o termo de maior grau está elevado ao
quadrado.

Equação Completa do segundo grau


Uma equação do segundo grau é completa, se todos os coeficientes a, b e c são diferentes de zero.
Exemplos:
1. 2 x² + 7x + 5 = 0
2. 3 x² + x + 2 = 0

Equação incompleta do segundo grau


Uma equação do segundo grau é incompleta se b=0 ou c=0 ou b=c=0. Na equação incompleta o coeficiente
a é diferente de zero.
Exemplos:
1. 4 x² + 6x = 0
2. 3 x² + 9 = 0
3. 2 x² = 0

Resolução de equações incompletas do 2o. grau


Equações do tipo ax²=0: Basta dividir toda a equação por a para obter:
x² = 0
significando que a equação possui duas raízes iguais a zero.
Equações do tipo ax²+c=0: Novamente dividimos toda a equação por a e passamos o termo constante
para o segundo membro para obter:
x² = -c/a
Se -c/a for negativo, não existe solução no conjunto dos números reais.
Se -c/a for positivo, a equação terá duas raízes com o mesmo valor absoluto (módulo) mas de sinais
contrários.
Equações do tipo ax²+bx=0: Neste caso, fatoramos a equação para obter:
x (ax + b) = 0
e a equação terá duas raízes:
x' = 0 ou x" = -b/a

Exemplos gerais
1. 4x²=0 tem duas raízes nulas.
2. 4x²-8=0 tem duas raízes: x'=R[2], x"= -R[2]
3. 4x²+5=0 não tem raízes reais.
4. 4x²-12x=0 tem duas raízes reais: x'=3, x"=0
Exercícios: Resolver as equações incompletas do segundo grau.
1. x² + 6x = 0
2. 2 x² = 0
3. 3 x² + 7 = 0
4. 2 x² + 5 = 0
5. 10 x² = 0
6. 9 x² - 18 = 0

Resolução de equações completas do 2o. grau


Como vimos, uma equação do tipo: ax²+bx+c=0, é uma equação completa do segundo grau e para resolvê-
la basta usar a fórmula quadrática (atribuída a Bhaskara), que pode ser escrita na forma:

onde D=b²-4ac é o discriminante da equação.


Para esse discriminante D há três possíveis situações:
1. Se D<0, não há solução real, pois não existe raiz quadrada real de número negativo.
2. Se D=0, há duas soluções iguais:
x' = x" = -b / 2a
3. Se D>0, há duas soluções reais e diferentes:
x' = (-b + R[D])/2a
x" = (-b - R[D])/2a
Exemplos: Preencher a tabela com os coeficientes e o discriminante de cada equação do segundo grau,
analisando os tipos de raízes da equação.
Equação a b c Delta Tipos de raízes
x²-6x+8=0 1 -6 8 4 reais e diferentes
x²-10x+25=0
x²+2x+7=0
x²+2x+1=0
x²+2x=0

O uso da fórmula de Bhaskara

Mostraremos agora como usar a fórmula de Bhaskara para resolver a equação:


x² - 5 x + 6 = 0
1. Identificar os coeficientes: a=1, b= -5, c=6
2. Escrever o discriminante D = b²-4ac.
3. Calcular D=(-5)²-4×1×6=25-24=1
4. Escrever a fórmula de Bhaskara:

5. Substituir os valores dos coeficientes a, b e c na fórmula:


x' = (1/2)(5+R[1]) = (5+1)/2 = 3
x" = (1/2)(5-R[1]) = (5-1)/2 = 2

Exercícios
1. Calcular o discriminante de cada equação e analisar as raízes em cada caso:
a. x² + 9 x + 8 = 0
b. 9 x² - 24 x + 16 = 0
c. x² - 2 x + 4 = 0
d. 3 x² - 15 x + 12 = 0
e. 10 x² + 72 x - 64 = 0
2. Resolver as equações:
a. x² + 6 x + 9 = 0
b. 3 x² - x + 3 = 0
c. 2 x² - 2 x - 12 = 0
d. 3 x² - 10 x + 3 = 0

Equações fracionárias do segundo grau


São equações do segundo grau com a incógnita aparecendo no denominador.
Exemplos:
1. 3/(x² - 4) + 1/(x - 3) = 0
2. 3/(x²-4)+1/(x-2)=0
Para resolver este tipo de equação, primeiramente devemos eliminar os valores de x que anulam os
denominadores, uma vez que tais valores não servirão para as raízes da equação, pois não existe fração
com denominador igual a 0. Na sequência extraímos o mínimo múltiplo comum de todos os termos dos
denominadores das frações, se houver necessidade.
1. Consideremos o primeiro exemplo:
3/(x² - 4) + 1/(x - 3) = 0
x deve ser diferente de 3, diferente de 2 e diferente de -2, assim podemos obter o mínimo múltiplo
comum entre os termos como:
MMC(x) = (x² - 4)(x - 3)
Reduzindo as frações ao mesmo denominador que deverá ser MMC(x), teremos:
[3(x-3) + 1(x²-4)] / (x²-4)(x-3) = 0
o que significa que o numerador deverá ser:
3(x - 3) + 1(x² - 4) = 0
que desenvolvido nos dá:
x2 + 3x - 13 = 0
que é uma equação do segundo grau que pode ser resolvida pela fórmula de Bhaskara. Não
existirão números reais satisfazendo esta equação.
2. Consideremos agora o segundo exemplo:
(x+3)/(2x-1)=2x/(x+4)
O mínimo múltiplo comum entre 2x-1 e x+4 é MMC=(2x-1)(x-4) (o produto entre estes fatores) e
MMC somente se anulará se x=1/2 ou x= -4. Multiplicando os termos da equação pelo MMC,
teremos uma sequência de expressões como:
(x+3)(x+4)=2x(2x-1)
x² + 7x + 12 = 4x² - 2x
-3x² + 9x + 12 = 0
3x² - 9x - 12 = 0
x² - 3x - 4 = 0
(x-4)(x+1) = 0

Solução: x'=4 ou x"= -1


3. Estudemos outro exemplo:
3/(x²-4)+1/(x-2)=0
O mínimo múltiplo comum é MMC=x²-4=(x-2)(x+2) e este MMC somente se anulará se x=2 ou x= -2.
Multiplicando os termos da equação pelo MMC, obteremos:
3 + (x+2)=0
cuja solução é x= -5
Exercícios: Resolver as equações do segundo grau fracionárias:
1. x + 6/x = -7
2. (x+2)/(x+1) = 2x/(x-4)
3. (2-x)/x + 1/x² = 3/x
4. (x+2)/(x-2) + (x-2)/(x+2) = 1

Equações bi-quadradas
São equações do 4o. grau na incógnita x, da forma geral:
a x4 + b x² + c = 0
Na verdade, esta é uma equação que pode ser escrita como uma equação do segundo grau através da
substituição:
y = x²
para gerar
a y² + b y + c = 0
Aplicamos a fórmula quadrática para resolver esta última equação e obter as soluções y' e y" e o
procedimento final deve ser mais cuidadoso, uma vez que
x² = y' ou x² = y"
e se y' ou y" for negativo, as soluções não existirão para x.
Exemplos:
1. Para resolver x4-13x²+36=0, tomamos y=x², para obter y²-13y+36=0, cujas raízes são y'=4 ou y"=9,
assim:
x² = 4 ou x² = 9
o que garante que o conjunto solução é:
S = { 2, -2, 3, -3}
2. Para resolver x4-5x²-36=0, tomamos y=x², para obter y²-5y-36=0, cujas raízes são y'= -4 ou y"=9 e
desse modo:
x² = -4 ou x² = 9
o que garante que o conjunto solução é:
S = {3, -3}
3. Se tomarmos y=x² na equação x4+13x²+36=0, obteremos y²+13y+36=0, cujas raízes são y'= -4 ou
y"= -9 e dessa forma:
x² = -4 ou x² = -9
o que garante que o conjunto solução é vazio.
Inequação do 1º Grau

Uma inequação do 1° grau na incógnita x é qualquer expressão do 1° grau que pode ser escrita numa das
seguintes formas:
ax + b > 0;
ax + b < 0;
ax + b ≥ 0;
ax + b ≤ 0.
Onde a, b são números reais com a ≠ 0.
Exemplos:
-2x + 7 > 0
x – 10 ≤ 0
2x + 5 ≤ 0
12 – x < 0

Resolvendo uma inequação de 1° grau


Uma maneira simples de resolver uma equação do 1° grau é isolarmos a incógnita x em um dos membros
da igualdade. Observe dois exemplos:
Exemplo1: Resolva a inequação -2x + 7 > 0.
Solução:
-2x > -7
Multiplicando por (-1)
2x < 7
x < 7/2
Portanto a solução da inequação é x < 7/2.
Exemplo 2: Resolva a inequação 2x – 6 < 0.
Solução:
2x < 6
x < 6/2
x<3
Portanto a solução da inequação e x < 3
Pode-se resolver qualquer inequação do 1° grau por meio do estudo do sinal de uma função do 1° grau,
com o seguinte procedimento:
1. Iguala-se a expressão ax + b a zero;
2. Localiza-se a raiz no eixo x;
3. Estuda-se o sinal conforme o caso.
Exemplo 1:
-2x + 7 > 0
-2x + 7 = 0
x = 7/2

Exemplo 2:
2x – 6 < 0
2x – 6 = 0
x=3
Inequação de 2º Grau

As inequações são expressões matemáticas que utilizam, na sua formatação, os seguintes sinais de
desigualdades:

>: maior que


<: menor que
≥: maior ou igual
≤: menor ou igual
≠: diferente

As inequações do 2º grau são resolvidas utilizando o teorema de Bháskara. O resultado deve ser
comparado ao sinal da inequação, com o objetivo de formular o conjunto solução.

Exemplo 1

Vamos resolver a inequação 3x² + 10x + 7 < 0.

S = {x ? R / –7/3 < x < –1}

Exemplo 2

Determine a solução da inequação –2x² – x + 1 ≤ 0.


S = {x ? R / x ≤ –1 ou x ≥ 1/2}

Exemplo 3

Determine a solução da inequação x² – 4x ≥ 0.

S = {x ? R / x ≤ 0 ou x ≥ 4}

Exemplo 4

Calcule a solução da inequação x² – 6x + 9 > 0.

S = {x ? R / x < 3 e x > 3}
Porcentagem
* Definição

PORCENTAGEM pode ser definida como a centésima parte de uma grandeza, ou o cálculo baseado em
100 unidades.
É visto com freqüência as pessoas ou o próprio mercado usar expressões de acréscimo ou redução nos
preços de produtos ou serviços.

Alguns exemplos:

- O Leite teve um aumento de 25%


Quer dizer que de cada R$ 100,00 teve um acréscimo de R$ 25,00

- O cliente teve um desconto de 15% na compra de uma calça jeans


Quer dizer que em cada R$ 100,00 a loja deu um desconto de R$ 15,00

- Dos funcionários que trabalham na empresa, 75% são dedicados.


Significa que de cada 100 funcionários, 75 são dedicados ao trabalho ou a empresa.

* Noção da porcentagem em números

Exemplos:

a)

60 de 150 dias de trabalho = 90 dias


100

O número 90 dias de trabalho representa : PORCENTAGEM

b)

70 de R$ 120,00 de compra = R$ 84,00


100

O valor de R$ 84,00 representa : PORCENTAGEM

* O que é taxa de porcentagem

É definido como taxa de porcentagem o valor obtido aplicando uma determinada taxa a um certo valor.
Também pode-se fixar a taxa de porcentagem como o numerador de uma fração que tem como
denominador o número 100.

* Como calcular porcentagem

Todo o cálculo de porcentagem, como informado, é baseado no número 100.

O cálculo de tantos por cento de uma expressão matemática ou de um problema a ser resolvido é indicado
pelo símbolo (%), e pode ser feito, na soma, por meio de uma proporção simples.

Para que se possam fazer cálculos com porcentagem (%), temos que fixar o seguinte:

1) A taxa está para porcentagem (acréscimo, desconto, etc), assim como o valor 100 está para a quantia a
ser encontrada.
Exemplificando:

Um título tem desconto 10%, sobre o valor total de R$ 100,00. Qual o valor do título?

30% : R$ 100,00

100% : X

X = R$ 30,00

2) O número que se efetua o cálculo de porcentagem é representado por 100.

Exemplificando:

Efetue o cálculo 10% de 50

100% : 50

10% :X

X=5

Obs. Nos dois exemplos dados foram usados o sistema de cálculo de regra de três, já ensinados em
tutoriais anteriores.

3) O capital informado tem sempre por igualdade ao 100.

Exemplificando:

Efetua-se o resgate de um cheque pré-datado no valor de R$ 150,00 e obtem-se um desconto de 20%

100% : R$ 150,00

20% : X

X = R$ 30,00

* Exemplos para fixação de definição

1) Um jogador de basquete, ao longo do campeonato, fez 250 pontos, deste total 10% foram de cestas de
02 pontos. Quantas cestas de 02 pontos o jogador fez do total de 250 pontos.

10% de 250 = 10 X 250 = 2500 = 25


100 100

Portanto, do total de 250 pontos o jogador fez 25 pontos de 02 pontos.

2) Um celular foi comprado por R$ 300,00 e revendido posteriormente por R$ 340,00, qual a taxa percentual
de lucro ?

Neste caso é procurado um valor de porcentagem no qual são somados os R$ 300,00 iniciais com a
porcentagem aumentada e que tenha como resultado o valor de R$ 340,00

300 + 300.X/100 = 340

3X = 340 – 300
X = 40/3

X = 13,333 (dízima periódica)

Assim, a taxa de lucro obtida com esta operação de revenda foi de 13,33%

* Fator Multiplicante

Há uma dica importante a ser seguida, no caso de cálculo com porcentagem. No caso se houver acréscimo
no valor, é possível fazer isto diretamente através de uma operação simples, multiplicando o valor do
produto/serviço pelo fator de multiplicação.

Veja:

Tenho um produto X, e este terá um acréscimo de 30% sobre o preço normal, devido ao prazo de
pagamento. Então basta multiplicar o valor do mesmo pelo número 1,30. Caso o mesmo produto ao invés
de 30% tenha 20% de acréscimo então o fator multiplicante é 1,20.

Observe esta pequena tabela:

Exemplo: Aumente 17% sobre o valor de um produto de R$ 20,00, temos R$ 20,00 * 1,17 = R$ 23,40

E assim sucessivamente, é possível montar uma tabela conforme o caso.

Da mesma forma como é possível, ter um fator multiplicante quando se tem acréscimo a um certo valor,
também no decréscimo ou desconto, pode-se ter este fator de multiplicação.

Neste caso, faz-se a seguinte operação: 1 – taxa de desconto (isto na forma decimal)

Veja:

Tenho um produto Y, e este terá um desconto de 30% sobre o preço normal. Então basta multiplicar o valor
do mesmo pelo número 0,70. Caso o mesmo produto ao invés de 30% tenha 20% de acréscimo então o
fator multiplicante é 0,80.

Observe esta pequena tabela:


Exemplo: Desconto de 7% sobre o valor de um produto de R$ 58,00, temos R$ 58,00 * 0,93 = R$ 53,94

E assim sucessivamente, é possível montar uma tabela conforme o caso.

* Exercícios resolvidos de porcentagem

Os exercícios propostos estão resolvidos, em um passo-a-passo prático para que se possa acompanhar a
solução de problemas envolvendo porcentagem e também para que se tenha uma melhor fixação sobre o
conteúdo.

1) Qual valor de uma mercadoria que custou R$ 555,00 e que pretende ter com esta um lucro de 17%?

Solução:

100% : 555
17 X

X = 555x17 /100 = 9435/100

X = 94,35

Temos o valor da mercadoria: R$ 555,00 + R$ 94,35

Preço Final: R$ 649,35

Obs. Este cálculo poderia ser resolvido também pelo fator multiplicador: R$ 555,00 * 1,17 = R$ 649,35

2) Um aluno teve 30 aulas de uma determinada matéria. Qual o número máximo de faltas que este aluno
pode ter sabendo que ele será reprovado, caso tenha faltado a 30% (por cento) das aulas ?

Solução:

100% : 30
30% :X

X = 30.30 / 100 = 900 / 100 = 9

X=9

Assim, o total de faltas que o aluno poderá ter são 9 faltas.

3) Um imposto foi criado com alíquota de 2% sobre cada transação financeira efetuada pelos consumidores.
Se uma pessoa for descontar um cheque no valor de R$ 15.250,00, receberá líquido quanto?

100% : 15.250
0,7% : X

Neste caso, use diretamente o sistema de tabela com fator multiplicador. O capital principal que é o valor do
cheque é : R$ 15.250,00 * 0,98 = R$ 14.945,00

Assim, o valor líquido do cheque após descontado a alíquota será de R$ 14.945,00. Sendo que os 2% do
valor total representam a quantia de R$ 305,00.

Somando os valores: R$ 14.945,00 + R$ 305,00 = R$ 15.250,00


Potenciação

Assim como podemos expressar e resolver de forma mais simples, uma soma de
várias parcelas iguais recorrendo à multiplicação, da mesma forma podemos
recorrer à exponenciação para obtermos o produto de vários fatores iguais.
A potenciação ou exponenciação é a operação de elevar um número ou
expressão a uma dada potência. Para entendermos o significado disto, observe a
figura em vermelho abaixo:

Note que temos o número dois ( 2 ) com o número três ( 3 ) sobrescrito à sua
direita ( 23 ). Dizemos que o número 2 está elevado à terceira potência, ou
ainda que 23 é a terceira potência de 2.
Nesta potência o número 2 é a sua base e ao número 3 damos o nome
de expoente.
Esta potência representa a multiplicação de três fatores iguais a dois, então 23 é
igual a 2 . 2 . 2 que é igual a 8.
Potências com expoente 2 ou 3 possuem uma outra forma particular de leitura. A
potência 23 também pode ser lida como dois ao cubo, assim como a
potência 32 pode ser lida como três ao quadrado.

Potências de Base Real com Expoente Inteiro


Nestas condições há quatro situações em particular que iremos tratar. A saber,
quando o expoente é maior que um, quando é igual a um, quando é igual a zero e
quando é negativo.

Expoente Maior que 1


De forma geral:
, isto é, a multiplicação de n fatores iguais a a.
Este é o caso de mais fácil compreensão, pois o conceito da exponenciação está
bem claro. Observe a expressão abaixo:

54, que se lê 5 elevado a 4, ou 5 elevado à quarta potência é igual ao produto


de quatro fatores todos eles iguais a cinco. Ao
multiplicarmos 5 vezes 5 vezes 5 vezes 5 iremos obter 625 que é o resultado da
exponenciação. O número de fatores iguais a 5 é justamente o numeral
do expoente.
Apesar de estarmos trabalhando com expoentes inteiros, as bases podem ser
decimais:

Assim como também podem ser fracionárias:

Expoente Igual a 1
Todo número elevado a 1 é igual ao próprio número:

Expoente Igual a 0
Todo número, diferente de zero, elevado a 0 é igual a 1:

00 é indeterminado, embora em algumas situações convenciona-se que seja igual


a 1. Para qualquer outro expoente real n positivo, temos que 0n = 0.
Mais à frente teremos outras informações que nos levarão a concluir que 00 = 0/0 e
como não existe divisão por zero no conjunto dos números reais, trata-se então de
uma indeterminação.
Ao estudarmos os expoentes negativos, a seguir, poderemos concluir que 0n é
indefinido para qualquer n real negativo, por exemplo, 0-2 pode ser expresso
como 1/02, o que nos leva à 1/0 e como sabemos, a divisão real de1 por 0 é
indefinida, pois não existe nenhum número real que multiplicado por 0 resulte
em 1.

Expoente Negativo
Qualquer número diferente de zero elevado a um expoente negativo é igual ao
inverso deste número elevado ao oposto do expoente:

Vejamos agora a explicação onde se baseiam estes três últimos conceitos


explicados acima.

Propriedades das Potências de Base Real com Expoente Inteiro

Multiplicação de Potências de Mesma Base


A multiplicação de potências de mesma base é igual a esta base elevada à soma
dos expoentes.
Vamos analisar o desenvolvimento da expressão a elevado à quinta potência
vezes a elevado ao quadrado para confirmarmos esta afirmação:

Primeiramente vamos substituir as potências por suas respectivas multiplicações:

Repare que a expressão foi substituída pela multiplicação de 7 fatores iguais a a.


Pelo conceito da exponenciação podemos então escrever a seguinte potência:

De onde concluímos que:

Generalizando:

Divisão de Potências de Mesma Base


A divisão de potências de mesma base, diferente de zero, é igual a esta base
elevada à diferença dos expoentes.
Vamos utilizar as mesmas potências analisadas na propriedade anterior, mas agora
fazendo a análise em relação à divisão:

Substituindo as potências por suas respectivas multiplicações:

Utilizamos uma fração ao invés do operador , apenas para visualisarmos mais


facilmente o próximo passo, que será a simplificação de dois fatores do numerador
com dois fatores do denominador:

Do estudado até agora sabemos que:

Então chegamos a conclusão de que:


Novamente generalizando temos:

Note que a base a deve ser diferente de 0, pois como sabemos não existe
quociente real para a divisão por zero neste conjunto numérico.

Entendendo porque a0 = 1
Para a ≠ 0 sabemos que:

Então se tivermos m = n temos que:

Sabemos que:

Já todo número, diferente de zero, dividido por ele mesmo é igual a 1 e que todo
número menos ele mesmo é igual a zero.
Logo concluímos que:

É por isto que todo número, diferente de zero, elevado a 0 é igual a 1:

Entendendo porque a1 = a
Para a ≠ 0 sabemos que:

Logo se tivermos m = n + 1 temos que:

Como:

Então:

Logo:

Agora vamos transformar as potências do primeiro membro em multiplicações do


fator a:

Repare que o numerador da fração no primeiro membro possui um fator a a mais


que o denominador, pois o expoente da potência do numerador tem uma unidade a
mais que o expoente da potência do denominador. Simplificando a fração temos:

Ou ainda:

Uma outra forma de entendermos porque a1 = a é que pela própria definição de


potência, o expoente indica o número de fatores e como o expoente é igual a 1,
obviamente este fator será o próprio número.

Entendendo porque a-n = 1/an


Como já vimos para a ≠ 0 temos que:

Se tivermos m = 0:

Como a0 = 1, temos:

Ou:
Potência de um Produto
A potência do produto de dois ou mais fatores é igual ao produto de cada um
destes fatores elevados ao expoente em questão:

Vamos tomar como exemplo o produto de três fatores distintos elevados ao cubo:

Não custa nada fazermos uma verificação só para conferir:

Potência de um Quociente
Podemos proceder de forma análoga ao que fizemos no caso da multiplicação, mas
neste caso os divisores não podem ser iguais a zero:

Exemplo:

Vamos verificar:

Potência de um Expoente Fracionário


Podemos transformar uma potência com expoente fracionário em um radical:

Exemplo:

Potência de uma Raiz


Ao elevarmos um radical a uma dada potência, estaremos obtendo o mesmo
resultado que obteríamos se elevássemos apenas o seu radicando a esta mesma
potência:

Exemplo:

Potência de uma Potência


Novamente para uma base diferente de zero podemos expressar a seguinte
igualdade:

Vamos como de costume recorrer a um exemplo:

E agora vamos verificar a veracidade desta propriedade:

Você sabe por que 2 + 3 . 5 é igual a 17 e não igual a 30?


Simplesmente porque o operador da multiplicação tem precedência sobre o
operador da adição. Você deve primeiro realizar a multiplicação e depois a adição.
Agora veja a expressão abaixo:
Qual é a razão desta desigualdade?
No caso de devemos calcular primeiro por causa da precedência dos
parênteses, o que está entre parênteses deve ser calculado primeiro. Já no caso
de devemos calcular primeiro, pois neste caso a precedência é
calcularmos do expoente mais externo para o mais interno.
Usemos como exemplo a expressão para verificarmos a desigualdade:
No primeiro membro iremos resolver primeiro 43 que é igual a 64, já no segundo
membro vamos resolver primeiro32 que é igual a 9:

Finalmente vamos elevar 64 ao quadrado e 4 à nona potência:

Alternativamente também podemos realizar as seguintes operações, multiplicando


os expoentes da potência do primeiro membro:
Radiciação

O tópico em questão agora é a radiciação que é a operação inversa da


exponenciação.

Observe a figura em vermelho abaixo:

Esta imagem representa a raiz cúbica de oito. A expressão matemática é


um radical, ela é composta pelo número 3 que é o índice da raiz, pelo símbolo
da radiciação e pelo número 8 que é o seu radicando.
Mas o que significa a raiz cúbica de oito?
Quando estudamos a potenciação, vimos que 23 é igual a 2 . 2 . 2 que é igual a 8.
Partimos do número 2 e através de uma multiplicação de 3 fatores iguais a 2,
chegamos ao número 8. Agora temos o caminho inverso, araiz cúbica de oito é a
operação que nos aponta qual é número que elevado a 3 é igual a 8, ou seja, é a
operação inversa da potenciação.

Raízes de Radicando Real com Índice Não Nulo


A raiz enésima de a é igual a b, se e somente se b elevado a enésima potência
for igual a a:

Não Existe a Raiz de um Radicando Negativo e Índice Par


Por quê?
Vamos tomar como exemplo a raiz quadrada de menos 16 expressa por .
Segundo a definição temos:

Qual é o valor numérico que b deve assumir para que multiplicado por ele mesmo
seja igual a -16?

Como sabemos na multiplicação de números reais ao multiplicarmos dois números,


diferentes de zero, com o mesmo sinal, o resultado sempre será positivo, então não
existe um número no conjunto dos números reais que multiplicado por ele
mesmo dará um valor negativo, pois o sinal é o mesmo em ambos os fatores
da multiplicação.

A Raiz de um Radicando Negativo e Índice Ímpar é Negativa


Em uma multiplicação se todos os sinais forem positivos, obviamente o produto
final também será positivo, já se tivermos fatores negativos, se estes forem em
quantidade par o resultado será positivo, se forem em quantidade ímpar o
resultado será negativo. É evidente que nenhum dos fatores pode ser igual a zero.
Então a raiz enésima de a, um número real negativo será negativa se o índice
for ímpar. Se for par como vimos acima, não existirá.
Vamos analisar a raiz quinta de menos 32 que se expressa como :

Como o expoente de b é ímpar, ou seja, o número de fatores que representa a


potência é impar, para que o resultado seja -32, é preciso que b seja negativo.
Então a raiz de um número negativo e índice ímpar sempre será um número
negativo.
Neste exemplo -2 é o número negativo que elevado a 5 resulta em -32, logo:

Note que na potência colocamos o -2 entre parênteses, pois se não o fizéssemos,


apenas o 2 estaria elevado à quinta potência. Como o expoente é ímpar, não faria
diferença no resultado se não os tivéssemos utilizado, mas isto seria imprescindível
se o expoente fosse um número par, para que não houvesse erro de sinal no
resultado da potenciação.

A Raiz de um Radicando Positivo também é Positiva


Não importa se o índice é par ou impar, em não sendo nulo, a raiz de um radicando
positivo também será positiva.
Vamos analisar a , que se lê raiz quadrada de nove:

Logo 3 é o número que elevado ao quadrado dá 9.


Mas você pode também se perguntar:
E se for -3? Se elevarmos -3 ao quadrado também iremos obter nove!
Correto, mas lembra-se da definição da raiz para um radicando positivo?

Tanto o radicando quanto a raiz devem ser positivos, é por isto que não podemos
considerar o -3.

A Raiz de um Radicando Nulo também é Nula


Isto é verdade desde que o índice não seja nulo também.
Exemplo:
, pois .

Propriedades da Radiciação
As propriedades que vamos estudar agora são consideradas no conjunto dos
números reais positivos ou nulos, podendo não se verificar caso o radicando seja
negativo, pois como sabemos, não existe raiz real de um número negativo.

A Raiz de uma Potência é uma Potência com Expoente Fracionário


Assim como de uma potenciação podemos chegar a uma radiciação, desta
podemos chegar a uma potenciação:

Exemplo:

Já que n não pode ser zero, a partir desta propriedade concluímos que não existe
raiz de índice zero. Se n fosse zero, o denominador da fração do expoente seria
zero, que sabemos não ser permitido.

Mudança de Índice pela sua Multiplicação/Divisão e do Expoente do


Radicando por um Mesmo número Não Nulo
Se multiplicarmos ou dividirmos tanto o índice do radical, quanto o expoente do
radicando por um mesmo número diferente de zero, o valor do radical continuará o
mesmo:

Exemplos:
Raiz de uma Potência
A raiz n de uma potência de a elevado a m, é a potência m da raiz n de a:

Exemplo:

Produto de Radicais de Mesmo Índice


O produto de dois radicais de mesmo índice é igual à raiz deste índice do produto
dos dois radicandos:

Exemplo:

Vamos verificar:

Divisão de Radicais de Mesmo Índice


O quociente de dois radicais de mesmo índice é igual a raiz deste índice do
quociente dos dois radicandos:

Exemplo:

Verificando:

Simplificação de Radicais Através da Fatoração


Podemos simplificar e em alguns casos até mesmo eliminar radicais, através
da decomposição do radicando em fatores primos. O raciocínio é simples,
decompomos o radicando em fatores primos por fatoração e depois simplificamos
os expoentes que são divisíveis pelo índice do radicando.
Vamos simplificar decompondo 91125 em fatores primos:

Como 91125 = 36 . 53 podemos dizer que:

Repare que tanto o expoente do fator 36, quanto o expoente do fator 53 são
múltiplos do índice do radicando que é igual a 3. Vamos então simplificá-los:

Perceba que através da fatoração de 91125 e da simplificação dos expoentes dos


fatores pelo índice do radicando, extraímos a sua raiz cúbica eliminando assim o
radical.
Vejamos agora o caso do radical :

Logo 2205 = 32 . 5 . 72, então:

Como os expoentes dos fatores 32 e 72 são divisíveis pelo índice 2, vamos


simplificá-los retirando-os assim do radical:

Neste caso o expoente do fator 5 não é divisível pelo índice 2 do radicando, por isto
após a simplificação não conseguimos eliminar o radical.
Agora vamos analisar o número :

Note que 729 = 36, então:

Neste caso o expoente de 36 não é divisível pelo índice 5, mas é maior, então
podemos escrever:

Repare que agora o expoente do fator 35 é divisível pelo índice 5, podemos então
retirá-lo do radical:

Agora vamos pensar um pouco. Após a fatoração tínhamos o radical . O


expoente 6 não é divisível por 5, pois ao realizarmos a divisão, obtemos um
quociente de 1 e um resto também de 1. Pois bem, o 1 do quociente será o
expoente da base 3 ao sair o radical. A parte que ainda ficou no radical terá como
expoente o 1 do resto. Vamos a alguns exemplos para melhor entendermos a
questão:
Simplifique .
Dividindo 18 por 7 obtemos um quociente de 2 é um resto de 4, logo fora do
radical a base 5 terá o expoente 2do quociente e a base dentro do radical terá o
expoente 4 que é o resto da divisão:

Logo:

Outro exemplo, simplifique .


A divisão de 15 por 5 resulta em quociente 3 e resto 0, pois a divisão é exata, mas
não há problema. Seguindo as explicações temos:

Veja que quando o é resto for zero podemos eliminar o radical, já que o radicando
sempre será igual a 1, pois todo número natural não nulo elevado a zero é igual a
um:

Nos casos em que os expoentes de todos os fatores forem menores que o índice do
radical como, por exemplo, em , a simplificação não poderá ser realizada.
Razão e Proporção

RAZÃO
Conceitualmente a razão do número a para o número b, sendo b ≠ 0, é igual ao quociente de a por b que podemos
representar das seguintes formas:


As razões acima podem ser lidas como:
• razão de a para b
• a está para b
• a para b
Em qualquer razão, ao termo a chamamos de antecedente e ao termo b chamamos de consequente.

Razão inversa ou recíproca


Vejamos as seguintes razões:

e
Elas são tidas como razões inversas ou recíprocas.
Note que o antecedente de uma é o consequente da outra e vice-versa.
Uma propriedade das razões inversas é que o produto delas é sempre igual a 1, isto se deve ao fato de uma ser o
inverso multiplicativo da outra.
Agora vejamos as seguintes razões:

e
A primeira razão possui os números 1 e 2 como seu respectivo antecedente e consequente, já a segunda razão possui
o número 2 como o seu antecedente e o número 1, omitido, como o seu consequente. Em função disto, pelo
antecedente de uma ser o consequente da outra e vice-versa, estas duas razões também são inversas uma em relação
a outra.
Apesar de uma razão ser apresentada na forma de uma fração ou de uma divisão, você pode calcular o seu valor final a
fim de se obter o seu valor na forma decimal. Por exemplo:
A razão de 15 para 5 é 3, pois 15 : 5 = 3 na forma decimal, ou seja, 15 é o triplo de 5.
Neste outro caso, a razão de 3 para 4 é 0,75, pois 3 : 4 = 0,75 na forma decimal.

Razão centesimal
Como visto acima, a razão de 3 para 4 é 0,75, pois 3 : 4 = 0,75 na forma decimal, ou seja, 3 equivale a 75% de 4. 75%
nada mais é que uma razão de antecedente igual 75 e consequente igual a 100. É por isto é chamada de razão
centesimal.

Exemplos
O salário de Paulo é de R$ 2.000,00 e João tem um salário de R$ 1.000,00. Qual a razão de um salário para outro?
Temos: Salário de Paulo : Salário de João.
Então:

A razão acima pode ser lida como a razão de 2000 para 1000, ou 2000 está para 1000. Esta razão é igual a 2, o que
equivale a dizer que o salário de Paulo é o dobro do salário de João, ou seja, através da razão estamos fazendo uma
comparação de grandezas, que neste caso são os salários de Paulo e João.
Portanto a razão de um salário para outro é igual a 2.
Eu tenho uma estatura de 1,80m e meu filho tem apenas 80cm de altura. Qual é a razão de nossas alturas?
Como uma das medidas está em metros e a outra em centímetros, devemos colocá-las na mesma unidade. Sabemos
que 1,80m é equivalente a 180cm. Temos então a razão de 180cm para 80cm:

2,25 é a razão de nossas alturas.

Proporção

A igualdade entre razões denomina-se proporção.


Os números a, b, c e d, todos diferentes de zero, formam nesta ordem, uma proporção se, e somente se, a razão a : b
for igual à razão c : d.
Indicamos esta proporção por:

Chamamos aos termos a e d de extremos e aos termos b e c chamamos de meios.


Veja que a razão de 10 para 5 é igual a 2 (10 : 5 = 2).
A razão de 14 para 7 também é igual a 2 (14 : 7 = 2).
Podemos então afirma que estas razões são iguais e que a igualdade abaixo representa uma proporção:

Lê-se a proporção acima da seguinte forma:


"10 está para 5, assim como 14 está para 7".

Propriedade fundamental das proporções


Qualquer que seja a proporção, o produto dos extremos é igual ao produto dos meios. Assim sendo, dados os números
a, b, c e d, todos diferentes de zero e formando nesta ordem uma proporção, então o produto de a por d será igual ao
produto de b por c:

Segunda propriedade das proporções


Qualquer que seja a proporção, a soma ou a diferença dos dois primeiros termos está para o primeiro, ou para o
segundo termo, assim como a soma ou a diferença dos dois últimos termos está para o terceiro, ou para o quarto termo.
Então temos:

ou
Ou

ou

Terceira propriedade das proporções


Qualquer que seja a proporção, a soma ou a diferença dos antecedentes está para a soma ou a diferença dos
consequentes, assim como cada antecedente está para o seu respectivo consequente. Temos então:

ou
Ou

ou
Quarta proporcional
Dados três números a, b, e c, chamamos de quarta proporcional o quarto número x que junto a eles formam a
proporção:

Tendo o valor dos números a, b, e c, podemos obter o valor da quarta proporcional, o número x, recorrendo à
propriedade fundamental das proporções. O mesmo procedimento utilizado na resolução de problemas de regra de três
simples.

Terceira proporcional
Em uma proporção onde os meios são iguais, um dos extremos é a terceira proporcional do outro extremo:

Na proporção acima a é a terceira proporcional de c e vice-versa.

Exemplos
Paguei R$15,00 por 1kg de carne. Se eu tivesse pago R$25,00 teria comprado 2kg. A igualdade da razão do preço de
compra pela quantidade, dos dois casos, resulta em uma proporção?
Os termos da nossa suposta proporção são: 15, 1, 25 e 2.
Podemos utilizar a propriedade fundamental das proporções para verificamos se tais termos nesta ordem formam ou
não uma proporção.
Temos então:

Como 30 difere de 25, não temos uma igualdade, consequentemente não temos uma proporção.
Poderíamos também ter analisado as duas razões:

Como as duas razões possuem valores diferentes, obviamente não se trata de uma proporção.

Como uma das razões resulta em 15 e a outra resulta em 12,5, concluímos que não se trata de uma proporção, já que
15 difere de 12,5.
A proporção não ocorreu porque ao comprar 2kg de carne, eu obteria um desconto de R$ 2,50 no preço do quilograma,
o que deixaria as razões desproporcionais.

A soma de dois números é igual a 240. Sabe-se que um deles está para 5, assim como o outro está para 7. Quais são
estes números?
Para a resolução deste exemplo utilizaremos a terceira propriedade das proporções. Chamando um dos números de a e
o outro de b, podemos montar a seguinte proporção:

Sabemos que a soma de a com b resulta em 240, assim como a adição de 5 a 7 resulta em 12. Substituindo estes
valores na proporção teremos:

Portanto:
Concluímos então que os dois números são 100 e 140.

Quatro números, todos diferentes de zero, 10, 8, 25 e x formam nesta ordem uma proporção. Qual o valor de x?
Seguindo o explicado sobre a quarta proporcional temos:

O valor do número x é 20.

Exercícios 1

1) Qual a razão que é igual a 2/7 e cujo antecedente seja igual a 8.

Resolução:

Vamos igualar as razões.

8=2
X 7

2x = 8 x 7

2x = 56

X = 56/2

X = 28

Desta forma a razão igual a 2/7, com antecedente igual a 8 é : 8/28 = 2/7

2) Almejando desenhar uma representação de um objeto plano de 5m de comprimento, usando uma escala de
1:20, qual será o comprimento no desenho:

Resolução:

Escala: 1
20

Sabendo que 1m = 100 cm.


Então 5m = 5 x 100 = 500 cm.

O comprimento no desenho será:

500 x 1 = 500 / 20 =
20

25 cm

Desta forma em uma escala 1:20 em plano de 5m, o comprimento do desenho será 25 cm.

3) Em uma sala de aula, a razão de moças para o número de rapazes é de 5/4. Se o número total de alunos desta
turma é de 45 pessoas, caso exista uma festa quantas moças ficariam sem par ?

Resolução:

Primeiro vamos denominar o número de moças por X, e o número de rapazes por Y.

x/y = 5/4 (Igualam-se as razões)

x + y = 45 (Soma total de alunos)

x + y = 5 + 4 (Aplicação das propriedades das proporções)


x 5

45/x = 9/5

45 x 5 = 9x

225 = 9x ---> x = 225/9 ---> x = 25 moças

Substituindo X = 25 na expressão x + y = 45, temos :

25 + y = 45 ---> y = 45 – 25 ----> y = 20 rapazes

Tendo por base que cada rapaz fique apenas com uma moça, o número de moças que ficariam sem par será : 25 – 20 =
5 moças

Então, o número de moças que ficará sem par é igual a 5.

EXERCÍCIOS 2

01. Se (3, x, 14, ...) e (6, 8, y, ...) forem grandezas diretamente proporcionais, então o valor de x + y é:
a) 20
b) 22
c) 24
d) 28
e) 32

02. Calcular x e y sabendo-se que (1, 2, x, ...) e (12, y, 4, ...) são grandezas inversamente proporcionais.

03. Dividir o número 160 em três partes diretamente proporcionais aos números 2, 3 e 5.

04. Repartir uma herança de R$ 495.000,00 entre três pessoas na razão direta do número de filhos e na razão inversa
das idades de cada uma delas. Sabe-se que a 1ª pessoa tem 30 anos e 2 filhos, a 2ª pessoa tem 36 anos e 3 filhos e a
3ª pessoa 48 anos e 6 filhos.

05. Dois números estão na razão de 2 para 3. Acrescentando-se 2 a cada um, as somas estão na razão de 3 para 5.
Então, o produto dos dois números é:

a) 90
b) 96
c) 180
d) 72
e) -124

06. (PUC) Se (2; 3; x; ...) e (8; y; 4; ...) forem duas sucessões de números diretamente proporcionais, então:

a) x = 1 e y = 6
b) x = 2 e y = 12
c) x = 1 e y = 12
d) x = 4 e y = 2
e) x = 8 e y = 12

07. Sabe-se que y é diretamente proporcional a x e que y = 10 quando x = 5. De acordo com estes dados, qual:

a) a sentença que relaciona y com x?


b) o gráfico da função f: [-2; 3] ® ℝ definida pela sentença anterior?
c) o valor de y quando x = 2?

08. São dados três números reais, a < b < c. Sabe-se que o maior deles é a soma dos outros dois e o menor é um
quarto do maior. Então a, b e c são, respectivamente, proporcionais a:

a) 1, 2 e 3
b) 1, 2 e 5
c) 1, 3 e 4
d) 1, 3 e 6
e) 1, 5 e 12

09. Dividindo-se 70 em partes proporcionais a 2, 3 e 5, a soma entre a menor e a maior parte é:

a) 35
b) 49
c) 56
d) 42
e) 28

10. Três pessoas montam uma sociedade, na qual cada uma delas aplica, respectivamente, R$ 20.000,00, R$
30.000,00 e R$ 50.000,00. O balanço anual da firma acusou um lucro de R$ 40.000,00. Supondo-se que o lucro seja
dividido em partes diretamente proporcionais ao capital aplicado, cada sócio receberá, respectivamente:

a) R$ 5.000,00; R$ 10.000,00 e R$ 25.000,00


b) R$ 7.000,00; R$ 11.000,00 e R$ 22.000,00
c) R$ 8.000,00; R$ 12.000,00 e R$ 20.000,00
d) R$ 10.000,00; R$ 10.000,00 e R$ 20.000,00
e) R$ 12.000,00; R$ 13.000,00 e R$ 15.000,00

Resolução:

01. E

02. x = 3 e y = 6

03. As partes são: 32, 48 e 80.

04. A 1ª pessoa deve receber R$ 120.000,00, a 2ª pessoa R$ 150.000,00 e a terceira pessoa R$ 225.000,00.

05. B

06. C

07. a) y = 2x

c) y = 4

08. C
09. B
10. Cvg
REGRA DE TRES SIMPLES E COMPOSTA
Regra de três simples

Regra de três simples é um processo prático para resolver problemas que envolvam quatro valores dos
quais conhecemos três deles. Devemos, portanto, determinar um valor a partir dos três já conhecidos.

Passos utilizados numa regra de três simples:

1º) Construir uma tabela, agrupando as grandezas da mesma espécie em colunas e mantendo na mesma
linha as grandezas de espécies diferentes em correspondência.

2º) Identificar se as grandezas são diretamente ou inversamente proporcionais.

3º) Montar a proporção e resolver a equação.

Exemplos:

1) Com uma área de absorção de raios solares de 1,2m², uma lancha com motor movido a energia solar
consegue produzir 400 watts por hora de energia. Aumentando-se essa área para 1,5m², qual será a
energia produzida?

Solução: montando a tabela:

Área (m²) Energia (Wh)


1,2--------400
1,5-------- x

Identificação do tipo de relação:

Área--------Energia
1,2---------400↓
1,5---------- X↓

Inicialmente colocamos uma seta para baixo na coluna que contém o x (2ª coluna).
Observe que: Aumentando a área de absorção, a energia solar aumenta.
Como as palavras correspondem (aumentando - aumenta), podemos afirmar que as grandezas são
diretamente proporcionais. Assim sendo, colocamos uma outra seta no mesmo sentido (para baixo) na 1ª
coluna. Montando a proporção e resolvendo a equação temos:

Área--------Energia
1,2---------400↓
1,5-----------x↓

1,2X = 400.1,5

x= 400.1,5 / 1,2

x= 500

Logo, a energia produzida será de 500 watts por hora.

2) Um trem, deslocando-se a uma velocidade média de 400Km/h, faz um determinado percurso em 3 horas.
Em quanto tempo faria esse mesmo percurso, se a velocidade utilizada fosse de 480km/h?

Solução: montando a tabela:


1) Velocidade (Km/h) Tempo (h)
400-----------------3
480---------------- x

2) Identificação do tipo de relação:

velocidade----------tempo
400↓-----------------3↑
480↓---------------- x↑

Obs: como as setas estão invertidas temos que inverter os numeros mantendo a primeira coluna e
invertendo a segunda coluna ou seja o que esta em cima vai para baixo e o que esta em baixo na segunda
coluna vai para cima

velocidade----------tempo
400↓-----------------X↓
480↓---------------- 3↓

480X = 400 . 3

x = 400 . 3 / 480

X = 2,5

Inicialmente colocamos uma seta para baixo na coluna que contém o x (2ª coluna).
Observe que: Aumentando a velocidade, o tempo do percurso diminui.

Como as palavras são contrárias (aumentando - diminui), podemos afirmar que as grandezas são
inversamente proporcionais. Assim sendo, colocamos uma outra seta no sentido contrário (para cima) na 1ª
coluna. Montando a proporção e resolvendo a equação temos:

Logo, o tempo desse percurso seria de 2,5 horas ou 2 horas e 30 minutos.

3) Bianca comprou 3 camisetas e pagou R$120,00. Quanto ela pagaria se comprasse 5 camisetas do
mesmo tipo e preço?

Solução: montando a tabela:

Camisetas----preço (R$)
3------------- 120
5---------------x

3x=5.120

o três vai para o outro lado do igual dividindo

x = 5.120/3

x= 200

Observe que: Aumentando o número de camisetas, o preço aumenta.


Como as palavras correspondem (aumentando - aumenta), podemos afirmar que as grandezas são
diretamente proporcionais. Montando a proporção e resolvendo a equação temos:

Logo, a Bianca pagaria R$200,00 pelas 5 camisetas.


4) Uma equipe de operários, trabalhando 8 horas por dia, realizou determinada obra em 20 dias. Se o
número de horas de serviço for reduzido para 5 horas, em que prazo essa equipe fará o mesmo trabalho?

Solução: montando a tabela:

Horas por dia-----Prazo para término (dias)

8↑------------------------20↓
5↑------------------------x ↓

invertemos os termos

Horas por dia-----Prazo para término (dias)

8↑-------------------------x↑
5↑------------------------20↑

5x = 8. 20

passando-e o 5 para o outro lado do igual dividindo temos:

5x = 8. 2 / 5

x = 32

Observe que: Diminuindo o número de horas trabalhadas por dia, o prazo para término aumenta.
Como as palavras são contrárias (diminuindo - aumenta), podemos afirmar que as grandezas são
inversamente proporcionais. Montando a proporção e resolvendo a equação temos:

EXERCICIOS

1) Uma roda dá 80 voltas em 20 minutos. Quantas voltas dará em 28 minutos? (R:112)

2) Com 8 eletricistas podemos fazer a instalação de uma casa em 3 dias. Quantos dias levarão 6 eletricistas
para fazer o mesmo trabalho? (R: 4)

3) Com 6 pedreiros podemos construir um a parede em 8 dias. Quantos dias gastarão 3 pedreiros para
fazer a mesma parede? (R:16)

4) Uma fabrica engarrafa 3000 refrigerantes em 6 horas. Quantas horas levará para engarrafar 4000
refrigerantes? (R: 8)

5) Quatro marceneiros fazem um armário em 18 dias. Em quantos dias 9 marceneiros fariam o mesmo
armário? (R:8)

6) Trinta operários constroem uma casa em 120 dias. Em quantos dias 40 operários construiriam essa
casa? (R: 90)

7) Uma torneira despeja em um tanque 50 litros de água em 20 minutos. Quantas horas levará para
despejar 600 litros? (R: 4)

8) Na construção de uma escola foram gastos 15 caminhões de 4 m³ de areia. Quantos caminhões de 6 m³


seriam necessários para fazer o mesmo trabalho? (R: 10)

9) Com 14 litros de tinta podemos pintar uma parede de 35 m². Quantos litros são necessários para pintar
uma parede de 15 m²? (R: 6)

10) Um ônibus, a uma velocidade média de 60 km/h, fez um percurso em 4 horas. Quanto levará,
aumentando a velocidade média para 80 km/h? (R:3)
11) Para se obterem 28 kg de farinha, são necessários 40 kg de trigo. Quantos quilogramas do mesmo trigo
são necessários para se obterem 7 kg de farinha? (R:10)

12) Cinco pedreiros fazem uma casa em 30 dias. Quantos dias levarão 15 pedreiros para fazer a mesma
casa? (R:10)

13) Uma máquina produz 100 peças em 25 minutos. Quantoas peças produzirá em 1 hora? (R:240)

14) Um automóvel faz um percurso de 5 horas à velocidade média de 60 km/h. Se a velocidade fosse de 75
km /h quantas horas gastaria para fazer o mesmo percurso? (R:4)

15)Uma maquina fabrica 5000 alfinetes em 2 horas. Qauntos alfinetes ela fabricará em 7 horas? (R:17.500)

16) Quatro quilogramas de um produto químico custam R$ 24.000,00 quanto custarão 7,2 Kg desse mesmo
produto? (R:43.200,00)

17) Oito operarios fazem um casa em 30 dias. quantos dias gastarão 12 operários para fazer a mesma
casa? (R:20)

18) Uma torneira despeja 2700 litros de água em 1 hora e meia. Quantos litros despeja em 14 minutos? (R:
420)

19) Quinze homens fazem um trabalho em 10 dias, desejando-se fazer o mesmo trabalho em 6 dias,
quantos homens serão necessários? (R:25)

20) Um ônibus, à velocidade de 90 Km/h, fez um percurso em 4 horas. Quanto tempo levaria se
aumentasse a velocidade para 120 Km/h? (R: 3)

21) Num livro de 270 páginas, há 40 linhas em cada página. Se houvesse 30 linhas, qual seria o número de
páginas desse livro? (R:360)

Regra de três composta

regra de três composta é utilizada em problemas com mais de duas grandezas, direta ou inversamente
proporcionais.

Exemplos:

1) Em 8 horas, 20 caminhões descarregam 160m3 de areia. Em 5 horas, quantos caminhões serão


necessários para descarregar 125m3?

Solução: montando a tabela, colocando em cada coluna as grandezas de mesma espécie e, em cada linha,
as grandezas de espécies diferentes que se correspondem:
Horas --------caminhões-----------volume
8↑----------------20↓----------------------160↑
5↑------------------x↓----------------------125↑

A seguir, devemos comparar cada grandeza com aquela onde está o x.


Observe que:
Aumentando o número de horas de trabalho, podemos diminuir o número de caminhões. Portanto a relação
é inversamente proporcional (seta para cima na 1ª coluna).
Aumentando o volume de areia, devemos aumentar o número de caminhões. Portanto a relação é
diretamente proporcional (seta para baixo na 3ª coluna). Devemos igualar a razão que contém o termo x
com o produto das outras razões de acordo com o sentido das setas.
Montando a proporção e resolvendo a equação temos:

Horas --------caminhões-----------volume
8↑----------------20↓----------------------160↓
5↑------------------x↓----------------------125↓
20/ x = 160/125 . 5/8 onde os temos da ultima fração foram invertidos

simplificando fica

20/x = 4/5

4x = 20 . 5

4x = 100

x = 100 / 4

x = 25

Logo, serão necessários 25 caminhões

2) Numa fábrica de brinquedos, 8 homens montam 20 carrinhos em 5 dias. Quantos carrinhos serão
montados por 4 homens em 16 dias?
Solução: montando a tabela:

Homens----- carrinhos------ dias


8-----------------20--------------5
4-------------------x-------------16

Observe que:
Aumentando o número de homens, a produção de carrinhos aumenta. Portanto a relação é diretamente
proporcional (não precisamos inverter a razão).
Aumentando o número de dias, a produção de carrinhos aumenta. Portanto a relação também é diretamente
proporcional (não precisamos inverter a razão). Devemos igualar a razão que contém o termo x com o
produto das outras razões.
Montando a proporção e resolvendo a equação temos:

20/x= 8/4 . 5/16

20 / x = 40 / 64

40x = 20 . 64

40 x = 1280

x = 1280 / 40

x = 32

Logo, serão montados 32 carrinhos

Método mais prático de solução da regra de três composta

Faça a comparação da grandeza que irá determinar com as demais grandezas. Se esta grandeza for
inversa, invertemos os dados dessa grandeza das demais grandezas.

A grandeza a se determinar não se altera, então, igualamos a razão das grandezas e determinamos o valor
que se procura.

Veja:

1) Na alimentação de 02 bois, durante 08 dias, são consumidos 2420 kgs de ração. Se mais 02 bois são
comprados, quantos quilos de ração serão necessários para alimentá-los durante 12 dias.
Assim: serão necessários 7260 Kgs de ração

2) Se 10 metros de um tecido custam R$ 50,00, quanto custará 22 metros ?

Solução: O problema envolve duas grandezas (quantidade de tecidos e preço da compra)

Assim: 22 metros custarão R$ 110,00

3) Em 06 dias de trabalho, 12 confeiteiros fazem 960 tortas. Em quantos dias 04 confeiteiros poderão fazer
320 tortas
Solução: O problema envolve três grandezas (tempo, número de confeiteiros, quantidade de tortas)

EXERCÍCIOS

01 – Com 10 kg de trigo podemos fabricar 7kg de farinha. Quantos quilogramas de trigo são necessários
para fabricar 28 kg de farinha?

02 – Com 50 kg de milho, obtemos 35 kg de fubá. Quantas sacas de 60 kg de fubá podemos obter com 1
200 kg de milho ?

03 – Sete litros de leite dão 1,5 quilos de manteiga. Quantos litros de leite serão necessários para se
obterem 9 quilos de manteiga ?

04 – Em um banco, contatou-se que um caixa leva, em média, 5 minutos para atender 3 clientes. Qual é o
tempo que esse caixa vai levar para atender 36 clientes ?

05 – Paguei R$ 6,00 por 1.250 kg de uma substância. Quanto pagaria por 0,750 kg dessa mesma
substância ?

06 – Seis máquinas escavam um túnel em 2 dias. Quantas máquinas idênticas serão necessárias para
escavar esse túnel em um dia e meio ?

07 – Uma fonte fornece 39 litros de água em 5 minutos. Quantos litros fornecerá em uma hora e meia ?

08 – Abrimos 32 caixas e encontramos 160 bombons. Quantas caixas iguais necessitamos para obter 385
bombons ?
09 – Um automóvel percorre 380 km em 5 horas. Quantos quilômetros percorrerá em 7 horas, mantendo a
mesma velocidade média ?

10 – Um automóvel gasta 24 litros de gasolina para percorrer 192 km. Quantos litros de gasolina gastará
para percorrer 120 km ?

11 – Uma torneira despeja 30 litros de água a cada 15 minutos. Quanto tempo levará para encher um
reservatório de 4m3 de volume?

12 – Um relógio adianta 40 segundos em 6 dias. Quantos minutos adiantará em 54 dias ?

13 – Um relógio atrasa 3 minutos a cada 24 horas.

a) Quantos minutos atrasará em 72 horas ?

b) Quantos minutos atrasará em 18 dias ?

c) Quantos dias levará para o relógio ficar atrasado 45 minutos ?

14 – Quero ampliar uma foto 3 x 4 (3 cm de largura e 4 cm de comprimento) de forma que a nova foto tenha
10,5 m de largura. Qual será o comprimento da foto ampliada?

15 – Uma foto mede 2,5 cm por 3,5 cm e se quer ampliá-la de tal maneira que o lado maior meça 14 cm.
Quanto deve medir o lado menor da foto ampliada ?

16 – Duas piscinas têm o mesmo comprimento, a mesma largura e profundidades diferentes. A piscina A
tem 1,75 m de profundidade e um volume de água de 35 m3. Qual é o volume de água da piscina B, que
tem 2 m de profundidade?

17 – Uma roda de automóvel dá 2750 voltas em 165 segundos. Se a velocidade permanecer constante,
quantas voltas essa roda dará em 315 segundos?

18 – A combustão de 48 g de carbono fornece 176 gás carbônico. A combustão de 30 g de carbono fornece


quantos gramas de gás carbônico?

19 – Num mapa, a distância Rio-Bahia, que é de 1.600 km, está representada por 24 cm. A quantos
centímetros corresponde, nesse mapa, a distância Brasília-Salvador, que é de 1200 km ?

20 – Sabendo-se que, para cada 5 fitas de música brasileira, tenho 2 fitas de música estrangeira, quantas
fitas de música brasileira eu tenho se possuo 22 fitas estrangeiras ?

21 – Duas piscinas têm a mesma largura e a mesma profundidade e comprimentos diferentes. Na piscina
que tem 8 m de comprimento, a quantidade de água que cabe na piscina é de 45.000 litros. Quantos litros
de água cabem na piscina que tem 10 m de comprimento ?

22 – Em uma prova de valor 6, Cristina obteve a nota 4,8. Se o valor da prova fosse 10, qual seria a nota
obtida por Cristina?

23 – Uma vara de 3 m em posição vertical projeta uma sombra de 0,80 m. Nesse mesmo instante, um
prédio projeta uma sombra de 2,40 m. Qual a altura do prédio ?

24 – Uma tábua de 2 m, quando colocada verticalmente, produz uma sombra de 80 cm. Qual é a altura de
um edifício que, no mesmo instante, projeta uma sombra de 12 m ?

25 – Uma tábua com 1,5 m de comprimento foi colocada verticalmente em relação ao chão e projetou urna
sombra de 53 cm. Qual seria a sombra projetada no mesmo instante por um poste que tem 10,5 m de
altura?
26 – Se 3/7 da capacidade de um reservatório correspondem a 8.400 litros, a quantos litros correspondem
2/5 da capacidade do mesmo tanque?

27 – Uma circunferência, com 8 cm de diâmetro, tem 25,1 cm de comprimento. Qual é o comprimento de


outra circunferência que tem 14 cm de diâmetro ?

28 – Uma folha de alumínio tem 400 cm2 de área e tem uma massa de 900 g. Qual será, em g, a massa de
uma peça quadrada, da mesma folha de alumínio, que tem 40 cm de lado? ( Determine a área da peça
quadrada ).

29 – Para azulejar uma parede retangular, que tem 6,5 m de comprimento por 3 m de altura, foram usados
390 azulejos. Quantos azulejos iguais a esses seriam usados para azulejar uma parede que tem 15 m2 de
área?

30 – Sabe-se que 100 graus aferidos na escala Celsius (100°C) correspondem a 212 graus aferidos na
escala Fahrenheit (212°F). Em Miami, nos Estados Unidos, uma temperatura, lida no termômetro
Fahrenheit, registrou 84,8 graus. Qual é a temperatura correspondente se lida no termômetro Celsius?

31 – Com 4 latas de tinta pintei 280 m2 de parede. Quantos metros quadrados poderiam ser pintados com
11 latas dessa tinta?

32 – Um corredor de Fórmula 1 manteve, em um treino, a velocidade média de 153 km/h. Sabendo-se que 1
h = 3 600 s, qual foi a velocidade desse corredor em m/s ?

33 – A velocidade de um móvel é de 30m/s, Qual será sua velocidade em km/h ?

34 – Para fazer um recenseamento, chegou-se à seguinte conclusão: para visitar 102 residências, é
necessário contratar 9 recenseadores. Numa região em que existem 3 060 residências, quantos
recenseadores precisam ser contratados ?

35 – O ponteiro de um relógio de medição funciona acoplado a uma engrenagem, de modo que 4 voltas
completas da engrenagem acarretam uma volta completa no mostrador do relógio. Quantas voltas
completas, no mostrador do relógio, o ponteiro dá quando a engrenagem dá 4.136 voltas ?

36 – O ponteiro menor de um relógio percorre um ângulo de 30 graus em 60 minutos. Nessas condições,


responda :

a) Quanto tempo ele levará para percorrer um ângulo de 42 graus ?

b) Se O relógio foi acertado às 12 horas ( meio-dia ), que horas ele estará marcando?

37 – Uma rua tem 600 m de comprimento e está sendo asfaltada. Em seis dias foram asfaltados 180 m da
rua Supondo-se que o ritmo de trabalho continue o mesmo, em quantos dias o trabalho estará terminado?

38 – Um muro deverá ter 49 m de comprimento. Em quatro dias, foram construídos 14 m do muro.


Supondo-se que o trabalho continue a ser feito no mesmo ritmo, em quantos dias será construído o restante
do muro?

39 – Um automóvel percorreu uma distância em 2 horas, à velocidade média de 90 km por hora. Se a


velocidade média fosse de 45 km por hora, em quanto tempo o automóvel faria a mesma distância?

40 – Com a velocidade de 75 km/h, um ônibus faz percurso em 40 minutos. Devido a um pequeno


congestionamento, esse ônibus fez o percurso de volta em 50 minutos. Qual a velocidade média desse
ônibus no percurso de volta?

41 – Para transportar material bruto para uma construção, foram usados 16 caminhões com capacidade de
5 cm3 cada um. Se a capacidade de cada caminhão fosse de 4 cm 3, quantos caminhões seriam
necessários para fazer o mesmo serviço ?
42 – Com o auxílio de uma corda, que julgava ter 2 m de comprimento, medi o comprimento de um fio
elétrico e encontrei 40 m. Descobri, mais tarde, que a corda media na realidade, 2,05 m. Qual é o
comprimento verdadeiro do fio?

43 – Com uma certa quantidade de arame pode.se fazer uma tela de 50 m de comprimento por 1,20 m de
largura. Aumentando-se a largura em 1,80 m, qual será o comprimento de uma outra tela feita com a
mesma quantidade de arame da tela anterior ?

44 – Para construir a cobertura de uma quadra de basquete, 25 operários levaram 48 dias. Se fosse
construída uma cobertura idêntica em outra quadra e fossem contratados 30 operários de mesma
capacidade que os primeiros, em quantos dias a cobertura estaria pronta ?

45 – Para forrar as paredes de uma sala, foram usadas 21 peças de papel de parede com 80 cm de largura.
Se houvesse peças desse mesmo papel que tivessem 1,20 m de largura, quantas dessas peças seriam
usadas para forrar a mesma parede ?

46 – Para pintar um barco, 12 pessoas levaram 8 dias, Quantas pessoas, de mesma capacidade de
trabalho que as primeiras, são necessárias para pintar o mesmo barco em 6 dias ?

47 – Uma torneira, despejando 4,25 litros de água por minuto, enche uma caixa em 3 horas e meia. Em
quanto tempo uma torneira que despeja 3,5 I de água por minuto encherá uma caixa de mesma capacidade
que a primeira ?

48 – Oito pedreiros fazem um muro em 72 horas. Quanto tempo levarão 6 pedreiros para fazer o mesmo
muro ?

49 – Dez operários constroem uma parede em 5 horas. Quantos operários serão necessários para construir
a mesma parede em 2 horas ?

50 – Uma certa quantidade de azeite foi colocada em latas de 2 litros cada uma, obtendo-se assim 60 latas.
Se fossem usadas latas de 3 litros, quantas latas seriam necessárias para colocar a mesma quantidade de
azeite ?

51 – Um corredor gastou 2 minutos para dar uma volta num circuito à velocidade média de 210 km/h.
Quanto tempo o corredor gastaria para percorrer o circuito à velocidade média de 140km/h ?

52 – Para se transportar cimento para a construção de um edifício, foram necessários 15 caminhões de 2m3
cada um. Quantos caminhões de 3m3 seriam necessários para se fazer o mesmo serviço?

53 – Uma torneira despeja 16 litros por minuto e enche uma caixa em 5 horas. Quanto tempo levará para
encher a mesma caixa uma torneira que despeja 20 litros por minuto?

54 – Com certa quantidade de fio, um tear produz 35 m de tecido com 50 cm de largura. Quantos m de
tecido com 70 cm de largura esse tear pode produzir com a mesma quantidade de fio ?

55 – A área de um terreno é dada pelo produto do comprimento pela largura. Um terreno retangular tem 50
m de comprimento por 32 m de largura. Se você diminuir 7 m da largura, de quantos m deverá aumentar o
comprimento para que a área do terreno seja mantida ?

56 – Na construção de uma quadra de basquete, 20 pedreiros levam 15 dias. Quanto tempo levariam 18
pedreiros para construir a mesma quadra ?

57 – Um livro possui 240 páginas e cada página 40 linhas. Qual seria o número de páginas desse livro se
fossem colocadas apenas 30 linhas em cada página ?

58 – Para paginar um livro que tem 45 linhas em cada páginas são necessárias 280 páginas. Quantas
páginas com 30 linhas cada uma seriam necessárias para paginar o mesmo livro?

59 – Com velocidade média de 60 km/h, fui de carro de uma cidade A para uma cidade B em 16 min. Se a
volta foi feita em 12 minutos, qual a velocidade média da volta ?

60 – ( MACK – SP ) Uma engrenagem de 36 dentes movimenta outra de 48 dentes. Quantas voltas dá a


maior enquanto a menor dá 100 voltas ?

61 – Um caminhão percorre 1.116 km em 6 dias, correndo 12 horas por dia. Quantos quilômetros percorrerá
10 dias, correndo 14 horas por dia?

62 – Uma certa máquina, funcionando 4 horas por dia, fabrica 12.000 pregos durante 6 dias. Quantas horas
por essa máquina deveria funcionar para fabricar 20.000 pregos em 20 dias?

63 – Um ciclista percorre 75km em 2 dias, pedalando 3 horas por dia. Em quantos dias faria uma viagem
200 km, pedalando 4 horas por dia?

64 – Foram empregados 4 kg de fio para tecer 14 m de fazenda de 0,8 m de largura. Quantos quilogramas
serão precisos para produzir 350 m de fazenda com 1,2 m de largura ?

65 – Em 30 dias, uma frota de 25 táxis consome 100.000 l de combustível. Em quantos dias uma frota de 36
táxis consumiria 240.000 de combustível?

66 – Um folheto enviado pela Sabesp informa que uma torneira, pingando 20 gotas por minuto, em 30 dias,
ocasiona um desperdício de 100 l de água. Na casa de Helena, uma torneira esteve pingando 30 gotas por
minuto durante 50 dias. Calcule quantos litros de água foram desperdiçados.

67 – Numa fábrica de calçados, trabalham 16 operários que produzem, em 8 horas de serviço diário, 240
pares de calçados. Quantos operários São necessários para produzir 600 pares de calçados por dia, com
10 horas de trabalho diário?

68 – Meia dúzia de datilógrafos preparam 720 páginas em 18 dias. Em quantos dias 8 datilógrafos, com a
mesma capacidade dos primeiros, prepararão 800 páginas ?

69 – Para erguer um muro com 2,5 m de altura e 30 m de comprimento, certo número de operários levou 24
dias. Em quantos dias esse mesmo número de operários ergueria um muro de 2 m de altura e 25 m de
comprimento ?

70 – Um automóvel, com velocidade média de 60 km/h, roda 8 h por dia e leva 6 dias para fazer certo
percurso. Se a sua velocidade fosse de 80 km/h e se rodasse 9 horas por dia, em quanto tempo ele faria o
mesmo percurso?

71 – Dois carregadores levam caixas do depósito para um caminhão. Um deles leva 4 caixas por vez e
demora 3 minutos para ir e voltar. O outro leva 6 caixas por vez e demora 5 minutos para ir e voltar.
Enquanto o mais rápido leva 240 caixas, quantas caixas leva o outro ?

72 – O consumo de 8 lâmpadas, acesas durante 5 horas por dia, em 18 dias, é de 14 quilowatts. Qual será
o consumo em 15 dias, deixando apenas 6 dessas lâmpadas acesas durante 4 horas por dia?

73 – Em 6 dias, 6 galinhas botam 6 ovos. Quantos ovos botam 12 galinhas em 12 dias?

74 – Se 5 gatos pegam 5 ratos em 5 minutos, 100 gatos pegam 100 ratos em quantos minutos ?

75 – ( UNIV. BRASíLIA ) Com 16 máquinas de costura aprontaram 720 uniformes em 6 dias de trabalho.
Quantas máquinas serão necessárias para confeccionar 2.160 uniformes em 24 dias?

76 – ( USP – SP ) Uma família composta de 6 pessoas consome em 2 dias 3 kg de pão. Quantos quilos de
pão serão necessários para alimentá-la durante 5 dias, estando ausentes 2 pessoas?

77 – ( CEFETQ – 1991 ) Quinze operários trabalhando oito horas por dia, em 16 dias, constroem um muro
de 80 metros de comprimento. Em quantas horas por dia, 10 operários construirão um muro de 90 metros
de comprimento, da mesma altura e espessura do anterior, em 24 dias ?
78 – ( CEFET – 1993 ) Os desabamentos, em sua maioria, são causados por grande acúmulo de lixo nas
encostas dos morros. Se 10 pessoas retiram 135 toneladas de lixo em 9 dias, quantas toneladas serão
retiradas por 40 pessoas em 30 dias ?

79 – ( CEFETQ – 1996 ) Uma frota de caminhões percorreu 3 000 km para transportar uma mercadoria,
com velocidade média de 60 km/h, gastando 10 dias. Quantos dias serão necessários para que, nas
mesmas condições, uma frota idêntica percorra 4 500 km com uma velocidade média de 50 km/h ?

80 – ( CEFETQ – 1997 ) Há 40 dias, um torneira na casa de Neilson está apresentando um vazamento de


45 gotas por minuto. Se um vazamento de 20 gotas por minuto, apresentado pela mesma torneira,
desperdiça 100 litros de água em 30 dias, calcular o número de litros de água já desperdiçados na casa de
Neilson.

81 – ( EsPECEx – 1981 ) Se 12 recenseadores visitam 1440 famílias em 5 dias de trabalho de 8 horas por
dia, quantas famílias serão visitadas por 5 recenseadores, em 6 dias, trabalhando 4 horas por dia ?

82 – ( EsPECEx – 1982 ) Um grupo de jovens, em 16 dias, fabricam 320 colares de 1,20 m de cada.
Quantos colares de 1,25 m serão fabricados em 5 dias ?

83 – ( EsPECEx – 1983 ) Um trem percorreu 200 km em certo tempo. Se tivesse aumentado sua velocidade
em 10 km/h, teria percorrido essa distância em 1 hora menos. Determinar a velocidade do trem, em km/h.

Regra de Três – Questões Objetivas

84 – Se 4 máquinas fazem um serviço em 6 dias, então 3 dessas máquinas farão o mesmo serviço em:

a) 7 dias b) 8 dias c) 9 dias d) 4,5 dias

85 – Um quilo de algodão custa R$ 50,00. Um pacote de 40 gramas do mesmo algodão custa :

a) R$ 1,80 b) R$ 2,00 c) R$ 2,20 d) R$ 2,50

86 – Um litro de água do mar contém 25 gramas de sal. Então, para se obterem 50 kg de sal, o número
necessário de litros de água do mar será:

a) 200 b) 500 c) 2 000 d) 5 000

87 – Um avião percorre 2 700 km em quatro horas. Em uma hora e 20 minutos de vôo percorrerá:

a) 675 km b) 695 km c) 810 km d) 900 km

88 – Na fabricação de 20 camisetas, 8 máquinas gastam 4 horas. Para produzir 15 dessas camisetas, 4


máquinas gastariam quantas horas ?

a) 3 horas b) 6 horas c) 5 horas d) 4 horas

89 – Em 7 dias, 40 cachorros consomem 100 kg de ração. Em quantos dias 3/8 deles comeriam 75 kg de
ração ?

a) 10 dias. b) 12 dias. c) 14 dias. d) 18 dias

90 – Três máquinas imprimem 9.000 cartazes em uma dúzia de dias. Em quantos dias 8/3 dessas máquinas
imprimem 4/3 dos cartazes, trabalhando o mesmo número de horas por dia?

a) 4 dias. b) 6 dias. c) 9 dias. d) 12 dias


91 – ( VESTIBULINHO – SP ) Numa corrida de FórmuIa 1, um corredor dá uma volta na pista em 1 minuto e
30 segundos com velocidade média de 200 km por hora. Se sua velocidade média cair para 180km por
hora, o tempo gasto para a mesma volta na pista será de:

a) 2 min b) 2 min e 19 segundos

c) 1 min e 40 segundos d) 1 min e 50 segundos

92 – ( UMC – SP ) Um carro consumiu 50 litros de álcool para percorrer 600 km. Supondo condições
equivalentes, esse mesmo carro, para percorrer 840 km, consumirá :

a) 68 litros b) 80 litros c) 75 litros d) 70 litros

93 – ( UF – MG ) Uma empresa tem 750 empregados e comprou marmitas individuais congeladas


suficientes para o almoço deles durante 25 dias. Se essa empresa tivesse mais 500 empregados, a
quantidade de marmitas já adquiridas seria suficiente para um numero de dias igual a:

a) 10 b) 12 c) 15 d) 18

94 – ( UDF ) Uma máquina varredeira limpa uma área de 5.100 m2 em 3 horas de trabalho. Nas mesmas
condições, em quanto tempo limpará uma área de 11.900 m2 ?

a) 4 horas b) 5 horas c) 7 horas d) 9 horas

95 – ( PUC – SP ) Um motorista de táxi, trabalhando 6 horas por dia durante 10 dias, gasta R$ 1.026,00 de
gás. Qual será o seu gasto mensal, se trabalhar 4 horas por dia ?

a) R$ 1.026,00 b) R$ 2.052,00

c) R$ 3.078,00 d) R$ 4.104,00

96 – ( VUNESP – SP ) Um secretário gastou 15 dias para desenvolver um certo projeto, trabalhando 7 horas
por dia. Se o prazo concedido fosse de 21 dias para realizar o mesmo projeto, poderia ter trabalhado :

a) 2 horas a menos por dia. b) 2 horas a mais por dia.

c) 3 horas a menos por dia. d) 3 horas a mais por dia.

97 – ( MACK – SP ) Se 15 operários em 9 dias de 8 horas ganham R$ 10.800,00; 23 operários em 12 dias


de 6 horas ganhariam :

a) R$ 16.560,00 b) R$ 17.560,00.

c) R$ 26.560,00. d) R$ 29.440,00

98 – ( SANTA CASA – SP ) Sabe-se que 4 máquinas, operando 4 horas por dia, durante 4 dias, produzem 4
toneladas de certo produto Quantas toneladas do mesmo produto seriam produzidas por 6 máquinas
daquele tipo, operando 6 horas por dia, durante 6 dias ?

a) 8 b) 15 c) 10,5 d) 13,5

99 – ( FEP – PA ) Para asfaltar 1 km de estrada, 30 homens gastaram 12 dias trabalhando 8 horas por
horas por dia. Vinte homens, para asfaltar 2 km da mesma estrada, trabalhando 12 horas por dia, gastarão :

a) 6 dias. b) 12 dias. c) 24 dias. d) 28 dias.

100 – ( PUCCAMP-SP ) Operando 12 horas por dia horas, 20 máquinas produzem 6000 peças em 6 dias.
Com 4 horas a menos de trabalho diário, 15 daquelas máquinas produzirão 4.000 peças em:
a) 8 dias b) 9 dias

c) 9 dias e 6 horas. d) 8 dias e 12 horas.

101 – ( USP – SP ) Uma família de 6 pessoas consome em 2 dias 3 kg de pão. Quantos quilos serão
necessários para alimentá-lo durante 5 dias estando ausentes 2 pessoas ?

a) 3 quilos b) 4 quilos c) 5 quilos d) 6 quilos

102 – ( Unimep – SP ) Se dois gatos comem dois ratos em dois minutos, para comer 60 ratos em 30
minutos são necessários:

a) 4 gatos b) 3 gatos c) 2 gatos

d) 5 gatos e) 6 gatos

102 – ( FAAP – SP ) Numa campanha de divulgação do vestibular, o diretor mandou confeccionar cinqüenta
mil folhetos. A gráfica realizou o serviço em cinco dias, utilizando duas máquinas de mesmo rendimento, oito
horas por dia. O diretor precisou fazer nova encomenda. Desta vez, sessenta mil folhetos. Nessa ocasião,
uma das máquinas estava quebrada. Para atender o pedido, a gráfica prontificou-se a trabalhar 12 horas
por dia, executando o serviço em :

a) 5 dias b) 8 dias c) 10 dias d) 12 dias

103 – ( PUC Campinas 2001 ) Em uma fábrica, constatou-se que eram necessários 8 dias para produzir
certo nº de aparelhos, utilizando-se os serviços de 7 operários, trabalhando 3 horas a cada dia. Para reduzir
a dois dias o tempo de produção, é necessário :

a) triplicar o nº de operários

b) triplicar o nº de horas trabalhadas por dia

c) triplicar o nº de horas trabalhadas por dia e o nº de

operários

d) duplicar o nº de operários

e) duplicar o nº de operários e o número de horas

trabalhadas por dia

104 – ( UNICAMP 2001. ) Uma obra será executada por 13 operários (de mesma capacidade de trabalho)
trabalhando durante 11 dias com jornada de trabalho de 6 horas por dia. Decorridos 8 dias do início da obra
3 operários adoeceram e a obra deverá ser concluída pelos operários restantes no prazo estabelecido
anteriormente. Qual deverá ser a jornada diária de trabalho dos operários restantes nos dias que faltam
para a conclusão da obra no prazo previsto ?

a) 7h 42 min

b) 7h 44 min

c) 7h 46 min

d) 7h 48 min

e) 7h 50 min
105 – ( CEFET – 1990 ) Uma fazenda tem 30 cavalos e ração estocada para alimentá-los durante 2 meses.
Se forem vendidos 10 cavalos e a ração for reduzida à metade. Os cavalos restantes poderão ser
alimentados durante:

a) 10 dias b) 15 dias c) 30 dias

d) 45 dias e) 180 dias

106 – ( CEFETQ – 1980 ) Em um laboratório de Química, trabalham 16 químicos e produzem em 8 horas de


trabalho diário, 240 frascos de uma certa substância. Quantos químicos são necessários para produzir 600
frascos da mesma substância, com 10 horas de trabalho por dia ?

a) 30 b) 40 c) 45 d) 50

107 – ( Colégio Naval – 1995 ) Se K abelhas, trabalhando K meses do ano, durante K dias do mês, durante
K horas por dia, produzem K litros de mel; então, o número de litros de mel produzidos por W abelhas,
trabalhando W horas por dia, em W dias e em W meses do ano será :

a) b) c) d) e)

Respostas dos Exercícios de Regra de Três Simples e Composta

01) 40 kg
02) 14 sacas
03) 42 litros
04) 60 min
05) 60 minutos = 1 hora
06) 8 máquinas
07) 702 litros
08) 77 caixas
09) 532 km
10) 15 litros
11) 33 h 20 min
12) 6 minutos
13) 9 min / 54 min / 15 dias
14) 14 cm
15) 10 cm
16) 40 m3
17) 5.250 voltas
18) 110 g
19) 18 cm
20) 55 fitas
21) 56.250 litros
22) Nota 8
23) 9 metros
24) 30 m
25) 371 cm ou 3,71 m
26) 7.840 litros
27) 43.925 cm
28) 3.600 g
29) 300 azulejos
30) 40 graus
31) 770 m2
32) 42 m/s
33) 108 km/h
34) 270 recenseadores
35) 1.034 voltas
36) a)84 min b) 1 h 24 min
37) 14 dias
38) 10 dias
39) 4 horas
40) 60 km/h
41) 20 caminhões
42) 41 m
43) 20 metros
44) 40 dias
45) 14 peças
46) 16 pessoas
47) 4 h 15 min
48) 96 horas
49) 25 operários
50) 40 latas
51) 3 minutos
52) 10 caminhões
53) 4 horas
54) 25 m
55) 20 cm
56) 16 dias e 16 horas
57) 320 páginas
58) 420 páginas
59) 80 km/h
60) 75 voltas
61) 2.170 km
62) 2 horas
63) 4 dias
64) 150 kg
65) 50 dias
66) 250 litros
67) 32 operários
68) 15 dias
69) 16 dias
70) 4 dias
71) 216 caixas
72) 7 kw
73) 24 ovos
74) 5 min
75) 12 máquinas
76) 5 kg
77) 9 horas
78) 1.800 toneladas
79) 18 dias
80) 300 litros
81) 360 famílias
82) 480 colares
83) 5 horas
84) letra d
85) letra b
86) letra c
87) letra d
88) letra b
89) letra c
90) letra b
91) letra c
92) letra d
93) letra c
94) letra c
95) letra b
96) letra a
97) letra a
98) letra d
99) letra c
100) letra a
101) letra c
102) letra a
103) letra e
104) letra d
105) letra d
106) letra d
107) letra e
Sistema métrico decimal
1 - Medidas de comprimento
No sistema métrico decimal, a unidade fundamental para medir comprimentos é o metro, cuja abreviação é
m. Existem os múltiplos e os submúltiplos do metro, veja na tabela:

Múltiplos u.f. Submúltiplos

quilômetro hectômetro decâmetr metro Decímetro centímetro Milímetro


o

km hm dam m Dm cm mm

1 000 m 100 m 10 m 1m 0,1 m 0,01 m 0,001 m

Existem outras unidades de medida mas que não pertencem ao sistema métrico decimal. Vejamos as
relações entre algumas dessas unidades e as do sistema métrico decimal:
1 polegada = 25 milímetros
1 milha = 1 609 metros
1 légua = 5 555 metros
1 pé = 30 centímetros
Obs: valores aprximados

1.1 - Transformação de unidades de comprimento


Observando o quadro das unidades de comprimento, podemos dizer que cada unidade de comprimento é
10 vezes maior que a unidade imediatamente inferior, isto é, as sucessivas unidades variam de 10 em 10.
Concluí-se então que para transformar uma unidade para um submúltiplo, basta multiplicar por 10 n onde n
é o número de colunas à direita do número na tabela. Já para passar para um múltiplo, basta dividir por 10 n
onde n é o número de colunas à esquerda do número na tabela.
Por exemplo: 7 m = 7 x 102 cm = 700 cm
500 m = 500 x 10-3 km = 0,5 km

2- Medidas de superfície
No sistema métrico decimal, a unidade fundamental para medir superfícies é o metro quadrado, cuja
representação é m2 . O metro quadrado é a medida da superfície de um quadrado de um metro de lado.
Como na medida de comprimento, na área também temos os múltiplos e os submúltiplos:

Múltiplos u.f. Submúltiplos

km2 hm2 dam2 m2 dm2 cm2 mm2

1 000 000 m2 10 000 m2 100 m2 1 m2 0,01 m2 0,0001 m2 0,000001 m2

2.1 - Transformação de unidades de superfície


Analogamente à transformação de unidades da medida de comprimento, faremos para a medida de área,
porém para cada devemos multiplicar ou dividir por 10 2 e não 10. Veja os exemplos:
a) 5 m2 = 5 x 102 dm2 = 500 dm2
b) 3 km2 = 3 x 106 m2 = 3 000 000 m2
c) 20 000 m2 = 20 000 x 10-6 km2 = 0,02 km2
obs. Quando queremos medir grandes porções de terra (como sítios, fazendas etc.) usamos uma unidade
agrária chamada hectare (ha).
O hectare é a medida de superfície de um quadrado de 100 m de lado.
1 hectare (há) = 1 hm2 = 10 000 m2
Em alguns estados do Brasil, utiliza-se também uma unidade não legal chamada alqueire.
- 1 alqueire mineiro é equivalente a 48 400 m2.
- 1 alqueire paulista é equivalente a 24 200 m 2.

3 - Áreas das figuras geométricas planas


Constantemente no estudo de gráficos, precisamos determinar a área compreendida entre a curva e o eixo-
x. Daremos aqui as fórmulas, para o cálculo da área, das figuras mais utilizadas na Física.

4 - Medidas de volume
No sistema métrico decimal, a unidade fundamental para medir volume é o metro cúbico, cuja abreviatura
é m3 . O metro cúbico (m3) é o volume ocupado por um cubo de 1 m de aresta. Como nas medidas de
comprimento e de área, no volume também temos os múltiplos e os submúltiplos:

Múltiplos u.f. Submúltiplos

km3 hm3 dam3 m3 dm3 cm3 mm3

1 000 000 000 1000 000 1000 m3 1 m3 0,001 m3 0,00001 m3 0,000000001 m3


m3 m3

As mais utilizadas, além do metro cúbico, são o decímetro cúbico e o centímetro cúbico.

4.1 - Transformação de unidades de volume


Analogamente à transformação de unidades da medida de comprimento, faremos para a medida de área,
porém para cada devemos multiplicar ou dividir por 10 3 e não 10. Veja os exemplos:
a) 8,2 m3 = 8,2 x 103 dm3 = 8 200 dm3
b) 500 000 cm3 = 500 000 x 10-6 m3 = 0,5 m3

5 - Medidas de capacidade
A unidade fundamental para medir capacidade de um sólido é o litro.
De acordo com o Comitê Internacional de Pesos e Medidas, o litro é, aproximadamente, o volume
equivalente a um decímetro cúbico, ou seja:
1 litro = 1,000027 dm3
Porém, para todas as aplicações práticas, simples, podemos definir:
1 litro = 1 dm3
Veja os exemplos:
1) Na leitura do hidrômetro de uma casa, verificou-se que o consumo do último mês foi de 36 m 3. Quantos
litros de água foram consumidos?
Solução: 36 m3 = 36 000 dm3 = 36 000 litros
2) Uma indústria farmacêutica fabrica 1 400 litros de uma vacina que devem ser colocados em ampolas de
35 cm3 cada uma. Quantas ampolas serão obtidas com essa quantidade de vacina?
Solução: 1 400 litros = 1 400 dm3 = 1 400 000 cm3
(1 400 000 cm3) : (35 cm3) = 40 000 ampolas.
5.1 - Outras unidades para medir a capacidade
São também utilizadas outras unidades para medir capacidade, que são múltiplos e submúltiplos do litro:

Múltiplos u.f. Submúltiplos

hectolitro decalitro litro decilitro centilitro mililitro

hl dal l dl cl ml

100 l 10 l 1l 0,1 l 0,01 l 0,001 l

Obs. 1) Não é usado nem consta da lei o quilolitro.


Obs. 2) Além do litro, a unidade mais usado é o mililitro (ml), principalmente para medir pequenos volumes,
como a quantidade de líquido de uma garrafa, de uma lata ou de uma ampola de injeção.

5.1.1 - Transformação de unidades de capacidade


Observando o quadro das unidades de capacidade, podemos verificar que cada unidade de capacidade é
10 vezes maior que a unidade imediatamente inferior, isto é, as sucessivas unidades variam de 10 em 10.
Veja os exemplos:
1) Expressar 15 l em ml.
Solução: 15 l = (15 x 103) ml = 15 000 ml
2) Expressar 250 ml em cm3.
Solução: 250 ml = 0,25 l = 0,25 dm3 = 250 cm3
1 - NOÇÕES DE LÓGICA

1.1 - DEFINIÇÕES INICIAIS

PROPOSIÇÃO é toda sentença, expressa em palavras ou símbolos, que pode ser valorada como
VERDADEIRA (V) ou FALSA (F).

Estas sentenças devem ser declarativas, pois as interrogativas, as exclamativas ou outras não
podem ser classificadas em verdadeiras ou falsas.

Exemplos:

- O Brasil é um país da América do Sul.

- 2 é um número par.

PROPOSIÇÃO SIMPLES ou ATÔMICA é quando a proposição não contém qualquer outra


proposição.

PROPOSIÇÃO COMPOSTA ou MOLECULAR é quando se pode extrair dela uma outra


proposição.

Exemplos:

- Proposição simples: A terra é redonda.

- Proposição Composta: Eduarda é filha de Luís e Cláudia. Dessa proposição pode se extrair as
proposições: Eduarda é filha de Luís e Eduarda é filha de Cláudia.

1.2 - CONECTIVOS LÓGICOS

Conectivos lógicos são palavras ou expressões que frequentemente estão presentes nas
proposições. São eles: “não”, ”e”,” “ou, “se …então”,” se e somente se”.

Exemplo: Se Luís Felipe não é adulto então ele é criança ou adolescente.

Essa é uma proposição composta com os conectivos lógicos “não”, “se … então”, e “ou”.
Os conectivos agem sobre as proposições compostas a que estão ligados de modo que seu
valor lógico (verdadeiro ou falso) depende somente

a) do valor lógico de cada uma das proposições componentes;


b) e da forma como essas preposições estão ligadas pelos conectivos lógicos utilizados.
Exemplo

Proposições Valor Lógico

3 é um número primo V

3 é um número fracionário F

3 é um número primo e fracionário F

3 é um número primo ou fracionário V

1.3 - PRINCIPAIS ESTRUTURAS LÓGICAS E SUAS DENOMINAÇÕES

Estruturas Denominações Representações Exemplos


Fundamentais

Não A Negação ∼A 10 não é um


número par

A ou B Disjunção A∨B 10 é um número par


ou é um número
primo

Ou A ou B Disjunção Exclusiva A∨B Ou 10 é um número


par ou 10 é um
número primo

AeB Conjunção A∧B 10 é um número par


e 10 é um não primo

Se A, então B Condicional A→B Se 10 é um número


par então 10 é um
número primo

A se e somente se B Bicondicional A↔B 10 é um número par


se e somente se 10
é um número primo.
1.4 - TABELAS-VERDADE DAS ESTRUTURAS FUNDAMENTAIS

Negação (∼A, A , ¬A)

Dada uma proposição A chama-se negação de A à preposição A acrescida do conectivo “não”


ou de outro equivalente.

Exemplo: A: 10 é um número par

∼ A: 10 não é um número par.

Outras formas de se expressar a negação:

Não é verdade que A

É falso que A

Tabela-verdade da negação

A ∼A

V F

F V

Disjunção (A ∨ B)

Disjunção é a proposição composta formada por duas preposições quaisquer que estão ligadas
pelo conectivo “ou”

Exemplo:

A: 5 é um número primo

B: 10 é um número ímpar

A ∨ B 5 é um número primo ou 10 é um número ímpar.


Tabela-verdade da disjunção (A ∨ B)

A B A∨B

V V V

V F V

F V V

F F F

Exemplos

A B A∨B

5 é um número primo (V) 10 é um número par (V) 5 é um número primo ou 10 é


um número par (V)

5 é um número primo (V) 10 é um número ímpar (F) 5 é um número primo ou 10 é


um número ímpar (V)

5 é um número par (F) 10 é um número par (V) 5 é um número par ou 10 é


um número par (V)

5 é um número par (F) 10 é um número ímpar (F) 5 é número par ou 10 é um


número ímpar (F)

CONCLUSÃO: Para uma disjunção ser verdadeira basta uma das proposições ser verdadeira.

Disjunção Exclusiva (A ∨ B)

Disjunção exclusiva é uma preposição composta formadas por duas preposições quaisquer em
cada uma delas tem está precedida pelo conectivo “ou”

Exemplo

A: 5 é um número primo

B: 10 é um número par

5
A ∨ B: Ou 5 é um número primo ou 10 é um número par.

Tabela-verdade da disjunção (A ∨ B)

A B A∨B

V V F

V F V

F V V

F F F

Exemplo:

A B A∨B

5 é um número ímpar (V) 10 é um número par (V) Ou 5 é um número ímpar ou


10 é um número par (F)

5 é um número ímpar (V) 10 é um número ímpar (F) Ou 5 é um número ímpar ou


10 é um número é ímpar (V)

5 é um número par (F) 10 é um número par (V) Ou 5 é um número par ou 10


é um número par (V)

5 é um número par (F) 10 é um numero ímpar (F) Ou 5 é um número par ou 10


é um número ímpar (F)

CONCLUSÃO: Uma disjunção exclusiva é verdadeira somente quando as preposições têm


valores lógicos contrários

Conjunção (A ∧ B)

Conjunção é a preposição composta por duas preposições quaisquer ligadas pelo conectivo “e”

Exemplo:

A: 5 é um número primo
B: 10 é um número par

A ∧ B: 5 é um número primo e 10 é um número par.

Tabela-verdade da conjunção (A ∧ B)

A B A∧B

V V V

V F F

F V F

F F V

Exemplo

A B A∧B

5 é um número ímpar (V) 10 é um número par (V) 5 é um número ímpar e 10 é


um número par (V)

5 é um número ímpar (V) 10 é um número ímpar (F) 5 é um número ímpar e 10 é


um número ímpar (F)

5 é um número par (F) 10 é um número par (V) 5 é um número par e 10 é um


número par (F)

5 é um número par (F) 10 é um número ímpar (F) 5 é um número par e 10 é um


número ímpar (F)

CONCLUSÃO: Uma conjunção só é verdadeira se as duas preposições são verdadeiras.

Condicional (A → B)

Em uma preposição condicional “Se A, então B” a preposição precedida da conjunção “se” é


chamada “condição” ou “antecedente”, enquanto a preposição B, precedida da proposição
“então” é denominada de “conclusão” ou “conseqüente”
Exemplo

A: 5 é um número ímpar

B: O dobro de 5 é um número par

A → B: Se 5 é um número ímpar, então o dobro de 5 é um número par.

Outras formas de expressar a condicional

Se A, B

B, se A

A implica B

A somente se B

A é suficiente para B

B é necessário para A

Tabela-verdade da condicional (A → B)

A B A→B

V V V

V F F

F V V

F F V

Exemplo

Considere a afirmativa: “Se um número é ímpar seu dobro é par” e as seguintes possibilidades:

A B A→B
Um número é ímpar (V) O dobro do número é par (V) Se um número é ímpar, então
seu dobro é par(V)

Um número é ímpar (V) O dobro do número é ímpar Se um número é ímpar seu


(F) dobro é par (F)

Um número é par (F) O dobro do número é par (V) Se um número é ímpar, então
seu dobro é par (V) (porque
nada se disse sobre o dobro de
um número par. Como uma
preposição deve ser verdadeira
ou falsa e essa não é falsa,
então ela é verdadeira)

Um número é par (F) O dobro do número é ímpar Se um número é par, então


seu dobro é ímpar (V) (como o
dobro do número ser par
estava condicionado ao fato
do número ser ímpar e sendo o
número par não
necessariamente ele deveria
ser par, logo a preposição não
é falsa. Portanto ela é
verdadeira)

IMPORTANTE: Usualmente quando se tem uma condicional é necessário que as preposições A


e B se relacionem de alguma forma ou guardem uma relação de causa ou efeito. Mas, segundo
as regras da Lógica, mesmo quando não existem essas relações entre A e B, a proposição A
→ B só é falsa se A é verdadeira e B é falsa.

Bicondicional (A ↔ B)

Bicondicional é uma preposição composta de duas preposições quaisquer ligadas pelo


conectivo “se e somente se”.

Exemplo:

A: 14 é múltiplo de 7

B: 14 é divisível por 7

A ↔ B: 14 é múltiplo de 7 se e somente se 14 é divisível por 7

Outras formas de se expressar a bicondicional


A se e só se B

Todo A é b e todo B é A.

Todo A é B e reciprocamente.

Se A então B e reciprocamente.

A é necessário e suficiente para B.

A é suficiente para B e B é suficiente para A.

A é necessário para B e B é necessário para A.

Tabela-verdade da condicional (A ↔ B)

A B A↔B

V V V

V F F

F V F

F F V

Exemplo

A B A↔B

14 é múltiplo de 7 (V) 14 é divisível por 7 (V) 14 é múltiplo de 7 se e somente se


14 é divisível por 7 (V)

14 é múltiplo de 7 (V) 14 não é divisível por 7F 14 é múltiplo de 7 se e somente se


14 não é divisível por 7 (F)

14 não é múltiplo de 7 14 é divisível por 7 (V) 14 não é múltiplo de 7 se e


(F) somente se 14 é múltiplo de 7 (F)

14 não é múltiplo de 7 14 não é divisível por 7 (F) 14 não é múltiplo de 7 se e


(F) somente se 14 não é divisível por 7
(V)
CONCLUSÃO: Uma preposição bicondicional só é verdadeira se as preposições que a compõem
têm o mesmo valor lógico.

1.5 - OUTRAS DEFINIÇÕES

Sentenças abertas: A expressão P(x) é uma sentença aberta na variável x se, e somente se,
P(x) se tornar uma preposição sempre que substituirmos a variável x por qualquer elemento
de um certo conjunto denominado universo do discurso.

Universo do discurso: conjunto de todos os valores que a variável x pode assumir.

Exemplo:

Universo do discurso: Conjunto de todos os números inteiros

Sentença aberta: O dobro de um número inteiro é igual a 6.

Sentença matemática aberta: 2x = 6

Observe que a sentença aberta é uma preposição verdadeira para x = 3 e falsa para todos os
demais números inteiros. Entretanto, a preposição conseguida quando se substitui x por todos
os valores do universo ela não tem necessariamente verdadeira.

Tautologia Uma preposição composta é uma tautologia se ela for sempre verdadeira,
independente dos valores lógicos das preposições que a compõem.

Exemplo: Se 2 é um número par e primo, então 2 é um número par ou 2 é um número primo.

Tabela-verdade da tautoplogia

A B A∧B A∨B A ∧ B→
→A∨B

2 é número par 2 é número 2 é um número 2 é um número Se 2 é um


par ou um número par e
(V) primo (V) par e primo (V) número primo primo, então 2 é
(V) um número par
ou um número
primo (V)

2 é um número 2 não é um 2 é um número 2 é um número Se 2 é um


par (V) número primo par e não é par ou não é um número par e
(F) primo (F) número primo primo então 2 é
(V) um número par
ou é um número
primo (V)

2 não é um 2 é um número 2 não é um 2 não é um Se 2 é um


número par (F) primo (V) número par e é número par ou 2 número par e
um número é um número primo, então 2 é
primo (F) primo (V) um número par
ou um número
primo (V)

2 não é um 2 não é um 2 não é um 2 não é um Se 2 é um


número par (F) número primo número par e número par ou 2 número par e
(F) um número não é um primo então 2 é
primo (F) número primo um número par
(F) ou um número
primo (V)

Contradição

Uma proposição composta formada por uma ou mais proposições é uma contradição se, e
somente se, independente dos valores lógicos de suas preposições componentes, ela é sempre
falsa.

Exemplo

Um número é par se e somente se ele não é par.

Tabela-verdade da Contradição

A ∼A A↔B
V F F

F V F

OBSERVAÇÃO: A negação de uma tautologia é sempre uma contradição e a negação se uma


contradição é sempre uma tautologia.

Contingência

Uma preposição composta é uma contingência se seu valor lógico depende dos valores lógicos
das preposições que a compõem.

Proposições equivalentes:Duas proposições são equivalentes se são compostas pelas mesmas


proposições simples e têm tabelas-verdade idênticas. (A ⇔ B)

1.6 - LEIS FUNDAMENTAIS DO PENSAMENTO LÓGICO

1ª Lei: Princípio da Identidade: Se uma preposição qualquer é verdadeira então ela é


verdadeira. ( P → P)

2ª Lei: Princípio da não contradição: Nenhuma preposição pode ser verdadeira e também
falsa. ∼(P ∧ ∼P)

3ª Lei: Princípio do terceiro excluído:Uma proposição ou verdadeira ou é falsa. (ou P ou ∼P)

1.7 - REGRAS DE EQUIVALÊNCIAS

Leis da Comutatividade

 A∧B⇔B∧A
 A∨B⇔B∨A
 A∨B⇔B∨A
 A↔B⇔B↔B

Leis de Associatividade

 (A ∧ B) ∧ C ⇔ A ∧ (B ∧ C)
 (A ∨ B) ∨ C ⇔ A ∨ (B ∨ C)

Leis da Distributividade

 A ∧ (B ∨ C) ⇔ (A ∧ B) ∨ (A ∧ C)
 A ∨ (B ∧ C) ⇔ (A ∨ B) ∧ (A ∨ C)

Lei da dupla negação

 ∼ (∼ A) ⇔ A

Lei das Equivalências da Condicional

 A → B ⇔ ∼A ∨ B
 A → B ⇔ ∼B → ∼A

Leis das Equivalências da Bicondicional

 A ↔ B ⇔ (A → B) ∧ (B → A)
 A ↔ B ⇔ (A ∧ B) ∨ (∼B ∧ ∼A)
 A ↔ B ⇔ ∼ (A ∨ B)

1.8 - TABELA DAS NEGAÇÕES DE PROPOSIÇÕES COMPOSTAS

Proposição Negação direta Equivalente da negação

AeB Não (A e B) Não A e não B

A ou B Não (A ou B) Não A ou não B

Se A então B Não (se A então B) A e não B

A se e somente se B Não (A se e somente se B) Ou A ou B

Todo A é B Não (todo A é B) Algum A não é B

Algum A é B Não (algum A é B) Nenhum A é B


1.9 - PROPOSIÇÕES CATEGÓRICAS

Na Lógica clássica (aristotélica) usa-se apenas quatro tipos de proposições, denominadas


proposições categóricas. Elas podem ser universais ou particulares e são

Afirmativas Negativas

Universais Todo A é B Nenhum A é B

Particulares Algum A é B Algum A não é B

1.10 - DIAGRAMAS LÓGICOS

Diagrama lógico é um esquema de representação das relações entre as diversas partes que
compõem uma proposição. O modelo mais usado são os diagramas de Venn-Euler.

Nesses modelos, o universo do discurso (conjunto de tudo que se admite como possível em
um dado contexto) é representado por um retângulo e cada proposição é indicada por uma
região delimitada dentro do universo do discurso.

U
Uma proposição é verdadeira em qualquer
A ponto dentro de sua região e falsa em todos os
demais pontos do universo. Assim, na região 1
1 do diagrama ao lado A é verdadeira e na região
2 B ela é falsa.

Ao representar uma estrutura lógica por um diagrama, somente as regiões para as quais o
resultado da tabela-verdade da estrutura representada for verdadeiro serão sombreadas.

Diagrama da Negação
Se a proposição for representada pelo conjunto A,
então a negação “não A” corresponderá ao
A conjunto complementar de A.

∼A

Diagrama da Conjunção

A ∧ B corresponde à interseção A ∩ B

A B

Diagrama da Disjunção

A ∨ B corresponde à união A ∪ B

AA B

Diagrama da disjunção exclusiva


A ∨ B corresponde ao conjunto (A – B) ∪ (B-A)

AA B

Diagrama da Condicional

a) Sombreando somente as regiões correspondentes aos resultados V da tabela-verdade da


proposição condicional.

AA BB

b) Como a inclusão do conjunto A no conjunto B

A A
B
Diagrama da Bicondicional

A ↔ B corresponde à igualdade dos conjuntos A e B


A=B
A (V) e B (V)
A

A (V) e B (V) 
 → A ↔ B (V)
~ A (F) e ~ B(F) 
A BB

1.11 - EXERCÍCIOS PROPOSTOS I

1) Considere a seguinte afirmativa : “Todos os bons alunos tiram notas boas” Em relação a essa
proposição é correto afirmar que

(a) Alguns bons estudantes não tiram notas boas.

(b) O conjunto dos bons estudantes contém o conjunto dos alunos que tiram notas boas.

(c) Todo bom estudante tira notas boas.

(d) Nenhum bom estudante tira notas boas.

(e) O conjunto dos bons estudantes contém o conjunto dos estudantes que tiram notas boas.

2) ) Considere a seguinte afirmativa : “Todo brasileiro gosta de samba” Em relação a essa


proposição é correto afirmar que

(a) toda pessoa que gosta de samba é brasileira.


(b) toda pessoa que não é brasileira não gosta de samba.

(c) toda pessoa que não gosta de samba não é brasileira.

(d) algum brasileiro não gosta de samba.

(e) alguma pessoa que não gosta de samba é brasileira.

3) Se Duda é bonita então Marina é graciosa. Se Marina é graciosa então Cláudia é autoritária.
Sabe-se que Cláudia não é autoritária. Nessas condições é correto afirmar que

(a) Duda não é graciosa.

(b) Marina não é bonita.

(c) Duda não é autoritária.

(d) Cláudia não é bonita.

(d) Duda não é bonita

4) Todo atleta é musculoso. Nenhum mineiro é musculoso. Nessas condições é correto afirmar
que

(a) algum atleta é mineiro.

(b) nenhum atleta é mineiro.

(c) nenhum atleta é musculoso.

(d) alguém que é musculoso é mineiro.

(e) nenhum mineiro é atleta.

5) Se tem sol faz calor. Nessas condições é correto afirmar que

(a) Ter sol é condição necessária para fazer calor.

(b) Fazer calor é condição suficiente para ter sol.

(c) Fazer sol é condição necessária e suficiente


(d) Fazer sol é condição suficiente para fazer calor.

(e) Fazer calor é condição necessária e suficiente para ter sol.

6) Represente por diagrama de Venn-Euler

a) Algum A é B

b) Algum A não é B

c) todo A é B

d) nenhum A é B

7) Considere as seguintes proposições

I – 4+3 = 7 e 2 + 6 = 8

II – 5 > 2 e 10 < 12

III – 4 = 7 e 5 < 1

Em relação a elas é correto afirmar que

a) todas são falsas.

b) I e II são falsas

c) somente III é falsa

d) Somente I é verdadeira.

e) somente II é falsa.

8) Considere as proposições

I – 2 + 3 = 5 ou 4 + 5 = 9

II – 8 < 3 e 6 < 5
III – 3 < 0 ou 2 = 8

Em relação a elas é correto afirmar que

a) todas as proposições são falsas

b) somente III é falsa

c) somente II é falsa

d) I e II são falsas.

e) I é falsa ou II é falsa.

9) Assinale a afirmativa falsa.

a) Se 2 é ímpar, então 5 é ímpar.

b) Se 4 ímpar, então 1 é menor que 5.

c) Se 6 é par, então 5 é menor que 2.

d) Se 3 é maior que 2, então 8 é menor que 9.

e) Se 5 é par, então 3 é maior que 7

10) A negação da proposição “Todas as mulheres são vaidosas” é

a) todos os homens são vaidosos.

b) algumas mulheres são vaidosas.

c) nenhuma mulher é vaidosa.

d) todos os homens não são vaidosos.

e) nenhum homem é vaidoso


11) Considere as proposições

P1: Todos os bebês são pequenos

P2: Pessoas pequenas têm baixa estatura

P3: Quem sabe jogar vôlei não tem baixa estatura.

Assinale a única alternativa que é uma conseqüência lógica das três proposições apresentadas.

a) Bebês não sabem jogar vôlei.

b) Pessoas de baixa estatura são bebês.

c) Pessoas de baixa estatura não sabem jogar vôlei.

d) Pessoas pequenas não sabem jogar vôlei.

As questões 12 e 13, a seguir referem-se ao seguinte texto: “Os sobrenomes de Ana, Beatriz
e Carla são Arantes, Braga e Castro, mas não necessariamente nesta ordem. A de sobrenome
Braga, que não é Ana, é mais velha que Carla e a de sobrenome Castro é a mais velha das
três.”

12) (Apostila MRE/2009 - Vestcon) Os sobrenomes de Ana, Beatriz e Carla são respectivamente

a) Arantes, Braga e Castro.

b) Arantes, Castro e Braga

c) Castro, Arantes e Braga

d) Castro, Braga e Arantes.

e) Braga, Arantes e Castro

13) (Apostila MRE/2009 - Vestcon) Nomeando-as em ordem crescente de idade, teremos

a) Ana, Beatriz e Carla.


b) Carla, Ana e Beatriz.

c) Beatriz, Carla e Ana.

d) Ana, Carla e Beatriz

e) Carla, Beatriz e Ana

14) (AFC/96) Se Beto briga com Glória, então Glória vai ao cinema. Se Glória vai ao cinema,
então Carla fica em casa. Se Carla fica em casa, então Raul briga com Carla. Ora, Raul não briga
com Carla, logo

a) Carla não fica em casa e Beto Não Briga com Glória.

b) Carla fica em casa e Glória vai ao cinema.

c) Carla não fica em casa e Glória vai ao cinema.

d) Glória vai ao cinema e Beto briga com Glória.

e) Glória não vai ao cinema e Beto briga com Glória.

15) (AFC/96) Três irmãs – Ana, Maria e Cláudia – foram a uma festa com vestidos de cores
diferentes. Uma vestiu azul, a outra branco e a terceira preto. Chegando à festa, o anfitrião
perguntou qual era uma delas. A de azul respondeu: “Ana é a que está de branco” A de branco
falou: “Eu sou Maria”E a de preto disse “Cláudia é quem está de branco” Como o anfitrião
sabia que Ana sempre diz a verdade, que Maria às vezes diz a verdade e que Cláudia nunca diz
a verdade, ele foi capaz de identificar corretamente quem era cada pessoa. As cores dos
vestidos de Ana, Maria e Cláudia eram, respectivamente,

a) preto, branco, azul.

b) preto, azul, branco.

c) azul, preto, branco.

d) azul, branco, preto.

e) branco, azul, preto.


16) (Apostila MRE/2009 - Vestcon) Dizer que é verdade que “para todo x, se x é rã e se x é
verde, então x está saltando” é logicamente equivalente a dizer que não é verdade que

a) algumas rãs que não são verdes estão saltando.

b) algumas rãs verdes estão saltando.

c) nenhuma rã verde não está saltando.

d) existe uma rã verde que não está saltando.

e) algo que não seja uma rã verde está saltando.

17) “Se você não estudar, então será reprovado. Sobre essa proposição é correto afirmar que

a) não estudar é condição suficiente para ser reprovado.

b) não estudar é condição necessária para ser reprovado.

c) se você estudar então será aprovado.

d) você será reprovado só se não estudar.

e) mesmo que você não estude você não será reprovado

18) Se os pais de professores são sempre professores, então é correto afirmar que

a) os filhos de não professores nunca são professores.

b) os filhos de não professores sempre são professores.

c) os filhos de professores sempre são professores

d) os filhos de professores quase sempre são professores.

e) alguns filhos de professores são professores.

19) Sejam x e y dois números reais quaisquer. Sendo assim, assinale a alternativa correta.

a) Se é verdade que x ≠ y então é falso que x ≥ y.


b) Se é verdade que x > y e então é verdade que x ≥ y.

c) Se é verdade que x ≠ y, então é falso que x ≤ y.

d) Se é verdade que x < y, então é falso que x ≤ y

e) Se é verdade que x ≥ y, então é verdade que x ≠ y

20) Sejam x e y dois números reais quaisquer e as afirmativas

I – Se é falso que x < y, então é verdadeiro que x > y.

II – Se é falso que x < y, então é verdade que x ≥ y.

III – Se é falso que x = y, então é verdade que ou x < y ou x > y

Em relação as essas afirmativas é correto dizer que

a) Todas as afirmativas são falsas.

b) As afirmativas I e III são falsas

c) As afirmativas I e II são verdadeiras.

d) As afirmativas II e III são verdadeiras.

e) Todas as afirmativas são verdadeiras

21) (VUNESP) Todos os marinheiros são republicanos. Assim sendo:

a) o conjunto de marinheiros contém o conjunto dos republicanos.

b) o conjunto dos republicanos contém o conjunto dos marinheiros.

c) todos os republicanos são marinheiros.

d) algum marinheiro não é republicano.

e) nenhum marinheiro é republicano.


22) (VUNESP) Assinale a afirmativa que apresenta uma contradição.

a) Todo espião não é vegetariano e algum vegetariano é espião.

b) Todo espião é vegetariano e algum vegetariano é não é espião.

c) Nenhum espião é vegetariano e algum espião não é vegetariano

d) Algum espião é vegetariano e algum espião não é vegetariano.

e) Todo vegetariano é espião e algum espião não é vegetariano.

23) (VUNESP) Todos os que conhecem João e Maria admiram Maria. Alguns que conhecem
Maria não a admiram. Logo:

a) todos que conhecem Maria a admiram.

b) ninguém admira Maria.

c) Alguns que conhecem Maria não conhecem João.

d) quem conhece João admira Maria.

e) só quem conhece João e Maria conhece Maria.

24) (VUNESP) Valter tem inveja de quem é mais rico do que ele. Geraldo não é mais rico do
que quem o inveja. Logo:

a) quem não é mais rico do que Valter é mais pobre que Valter.

b) Geraldo é mais rico do que Valter.

c) Valter não tem inveja de quem é mais rico do ele.

d)Valter inveja só quem é mais rico do que ele.

e) Geraldo não é mais rico que Valter


25) (VUNESP) Em uma avenida reta, a padaria fica entre o posto de gasolina e a banca de
jornal, e o posto de gasolina fica entre a banca de jornal e a sapataria. Logo:

a) a sapataria fica entre a banca de jornal e a padaria.

b) a banca de jornal fica entre o posto de gasolina e a padaria.

c) o posto de gasolina fica entre a padaria e a banca de jornal.

d) a padaria fica entre a sapataria e o posto de gasolina.

e) o posto de gasolina fica entre a sapataria e a padaria.

26) (VUNESP) Marta corre tanto quanto Rita e menos do que Juliana. Fátima corre tanto
quanto Juliana. Logo:

a) Fátima corre menos do que Rita.

b) Fátima corre mais que Marta.

c)Juliana corre menos do que Rita

d) Marta corre mais do que Juliana.

e) Juliana corre menos do que Marta.

27) (BACEN – Analista) Aldo, Benê e Caio receberam uma proposta para executar um projeto.
A seguir estão registradas as declarações dadas pelos três, após a conclusão do projeto.

- Aldo: Não é verdade que benê e Caio executaram o projeto.

- Benê: Se Aldo não executou o projeto, então Caio o executou.

- Caio: Eu não executei o projeto, mas Aldo e Benê o executaram.

Se somente a afirmação de Benê é falsa, então o projeto foi executado APENAS por

a) Aldo

b) Benê

c) Caio

d) Aldo e Benê
e) Aldo e Caio

28) (BACEN – Analista) Sejam as proposições:

p: atuação compradora de dólares por parte do Banco Central.

q: fazer frente ao fluxo positivo.

Se p implica q, então

a) A atuação compradora de dólares por parte do Banco Central é condição necessária para
fazer frente ao fluxo positivo.

b) Fazer frente ao fluxo positivo é condição suficiente para a atuação compradora de dólares
por parte do Banco Central.

c) A atuação compradora de dólares por parte do Banco Central é condição suficiente para
fazer frente ao fluxo positivo.

d) Fazer frente ao fluxo positivo é condição necessária e suficiente para a atuação compradora
de dólares por parte do Banco Central.

e) A atuação compradora de dólares por parte do Banco Central não é condição suficiente e
nem necessária para fazer frente ao fluxo positivo.

29) (IPER – Técnico) Quando não vejo Lúcia, não passeio e fico deprimido. Quando chove, não
passeio e fico deprimido. Quando não faz calor e passeio, não vejo Lúcia. Quando chove e
estou deprimido, não passeio.

Hoje passeio. Portanto, hoje

a) vejo Lúcia, e não estou deprimido, e não chove e faz calor.

b) não vejo Lúcia, e estou deprimido, e chove e faz calor.

c) não vejo Lúcia, e estou deprimido, e não chove, e não faz calor.

d) vejo Lúcia, e não estou deprimido, e chove, e faz calor.

e) vejo Lúcia, e estou deprimido, e não chove, e faz calor.


30) (IPER – Técnico) Considerando “toda prova de Lógica é difícil” uma proposição verdadeira,
é correto inferir que

a) “nenhuma prova de Lógica é difícil” é uma proposição necessariamente verdadeira.

b) “alguma prova de Lógica é difícil” é uma proposição necessariamente verdadeira.

c) “alguma prova de Lógica é difícil” é uma proposição falsa e verdadeira.

d) “alguma prova de Lógica não é difícil” é uma proposição necessariamente verdadeira.

e) “alguma prova de Lógica não é difícil” é uma proposição verdadeira e falsa.

1.12 - GABARITO I

Questão Questão

1e 2c 3e 4e

5d 6a 6b 6c

B
A B B
A
A

6d 7c 8e 9b

B
A

10 c 11 a 12 d 13 e

14 a 15 b 16 a 17a

18 a 19b 20 d 21b
22 a 23 c 24 e 25 e

26 b 27 e 28 c 29 d

30 b

2 – ANÁLISE COMBINATÓRIA

2.1 - PRICÍPIO ADITIVO E MULTIPLICATIVO

Em análise Combinatória há dois princípios fundamentais – o Princípio Aditivo e o Princípio


Multiplicativo ou Princípio Fundamental da Contagem

Vejamos um exemplo de um problema em que se usa o princípio aditivo para resolvê-lo.

Em uma escola foi feita uma enquete para saber quem prefere futebol ou vôlei. O resultado foi
o seguinte: 230 alunos gostam de futebol, 150 gostam de vôlei e 80 alunos gostam dos dois
esportes. Quantos alunos tem essa escola?

Em princípio parecem ser 230 + 150 + 80 = 460 alunos. Entretanto, há que se observar que
entre os alunos que gostam de futebol podem existir alunos que também gostam de vôlei,
portanto, o número de alunos que gostam somente de futebol é 230 – 80 = 150. Da mesma
maneira, o número de alunos que gostam somente de vôlei é 150 – 80 = 70. Sendo assim, o
número de alunos da escola será:

Número de alunos que gostam só de futebol + número de alunos que gostam só de vôlei +
número de alunos que gostam de futebol e vôlei, ou seja, 150 + 70 + 80 = 300 alunos.

Isto porque, segundo o teorema:

Sendo A e B conjuntos finitos, o número de elementos da união de A e B é dado por:

n(A B) = n(A) + n(B) - n(AnB);

Você também pode gostar